cours de probabilité

266
EH - PROBABILITES Sommaire Chapitre 1 - Ensembles : généralités ............................................................... 3 Chapitre 2 - Ensembles dénombrables ou finis .................................................... 15 Chapitre 3 - Espaces probabilisables - Probabilités ................................................ 23 Chapitre 4 - Espaces probabilisés finis ............................................................ 33 Chapitre 5 - Variables aléatoires ................................................................. 45 Chapitre 6 - Variables discrètes .................................................................. 59 Chapitre 7 - Variables continues ................................................................. 73 Chapitre 8 - Lois de probabilité classiques ........................................................ 85 Résumé sur les variables aléatoires ............................................................... 97

Upload: jerome-younan

Post on 12-Aug-2015

76 views

Category:

Documents


0 download

TRANSCRIPT

EH - PROBABILITES

Sommaire

Chapitre 1 - Ensembles : généralités . . . . . . . . . . . . . . . . . . . . . . . . . . . . . . . . . . . . . . . . . . . . . . . . . . . . . . . . . . . . . . . 3

Chapitre 2 - Ensembles dénombrables ou finis . . . . . . . . . . . . . . . . . . . . . . . . . . . . . . . . . . . . . . . . . . . . . . . . . . . . 15

Chapitre 3 - Espaces probabilisables - Probabilités . . . . . . . . . . . . . . . . . . . . . . . . . . . . . . . . . . . . . . . . . . . . . . . . 23

Chapitre 4 - Espaces probabilisés finis . . . . . . . . . . . . . . . . . . . . . . . . . . . . . . . . . . . . . . . . . . . . . . . . . . . . . . . . . . . .33

Chapitre 5 - Variables aléatoires . . . . . . . . . . . . . . . . . . . . . . . . . . . . . . . . . . . . . . . . . . . . . . . . . . . . . . . . . . . . . . . . . 45

Chapitre 6 - Variables discrètes . . . . . . . . . . . . . . . . . . . . . . . . . . . . . . . . . . . . . . . . . . . . . . . . . . . . . . . . . . . . . . . . . . 59

Chapitre 7 - Variables continues . . . . . . . . . . . . . . . . . . . . . . . . . . . . . . . . . . . . . . . . . . . . . . . . . . . . . . . . . . . . . . . . . 73

Chapitre 8 - Lois de probabilité classiques . . . . . . . . . . . . . . . . . . . . . . . . . . . . . . . . . . . . . . . . . . . . . . . . . . . . . . . .85

Résumé sur les variables aléatoires . . . . . . . . . . . . . . . . . . . . . . . . . . . . . . . . . . . . . . . . . . . . . . . . . . . . . . . . . . . . . . . 97

EH 3

Chapitre 1 - Ensembles : généralités

Ce premier chapitre n’est pas un exposé méthodique de la théorie des ensembles. On y rappelle sim-plement quelques définitions et propriétés dont l’utilisation est importante en théorie des probabilités.Les résultats sont donnés sans démonstrations, qui pourront être faites à titre d’exercices.

Un ensemble Ω étant donné une fois pour toute, on rappelle les différentes opérations dans l’ensembleP(Ω) des parties de Ω et leurs propriétés élémentaires.

1- Opérations dans P(Ω)

Dans les dessins suivants, l’ensemble cherché est en grisé.

(1) Complémentaire de A dans Ω

Si A est une partie de Ω, on note AC ou ∁ΩA (et parfois A), l’ensemble des éléments x de Ω qui nesont pas dans A.

Ω

A

(2) Intersection

A ∩ B = x |x ∈ A et x ∈ B .

Ω

A

B

EH 4

Plus généralement si (Ai)i∈I est une famille de parties de Ω,

i∈I

Ai = x | ∀i ∈ I, x ∈ Ai .

(3) Réunion

A ∪ B = x |x ∈ A ou x ∈ B .

(Il s’agit du « ou » inclusif).

Ω

A

B

Plus généralement si (Ai)i∈I est une famille de parties de Ω,

i∈I

Ai = x | ∃i ∈ I, x ∈ Ai .

(4) Différence

A \ B = x |x ∈ A et x /∈ B = A ∩ BC = A \ (A ∩ B) .

Ω

A

B

EH 5

(5) Différence symétrique

A∆ B = x |x ∈ A ou x ∈ B et x /∈ (A ∩ B) .

(Il s’ agit donc ici d’un « ou » exclusif).

On a également

A∆ B = (A \ B) ∪ (B \ A) = (A ∪ B) \ (A ∩ B) .

Ω

A

B

Formulaire important concernant les trois premières opérations

1)∅C = Ω , ΩC = ∅ , (AC)C = A

(le passage au complémentaire est une opération involutive dans P(Ω)).

2) Formules de Morgan

(

i∈I

Ai

)C

=⋃

i∈I

ACi ,

(

i∈I

Ai

)C

=⋂

i∈I

ACi .

3) Les lois ∪ et ∩ sont des lois internes dans P(Ω) et ont les propriétés suivantes :

associativité :(A ∪ B) ∪ C = A ∪ (B ∪ C) , (A ∩ B) ∩ C = A ∩ (B ∩ C) .

commutativité :A ∪ B = B ∪ A , A ∩ B = B ∩ A .

EH 6

distributivité d’une loi sur l’autre :

A ∪ (B ∩ C) = (A ∪ B) ∩ (A ∪ C) , A ∩ (B ∪ C) = (A ∩ B) ∪ (A ∩ C) .

4) Eléments neutres et absorbants

− ∅ est élément neutre pour ∪

A ∪ ∅ = A .

− Ω est absorbant pour ∪

A ∪ Ω = Ω .

− Ω est élément neutre pour ∩

A ∩ Ω = A .

− ∅ est absorbant pour ∩

A ∩ ∅ = ∅ .

De plus

A ∩ A = A ∪ A = A .

Exercice 1 Démontrer les formules suivantes.

a) (A ∪ B) \ C = (A \ C) ∪ (B \ C) ;

b) (A ∩ B) \ C = (A \ C) ∩ (B \ C) = (A \ C) ∩ B = (B \ C) ∩ A ;

c) (A \ B) \ C = A \ (B ∪ C) ;

d) A∆ BC = AC ∆ B = (A∆ B)C , AC ∆ BC = A∆ B , A∆ Ω = AC .

Exercice 2 La loi ⋆ désignant la réunion, l’intersection, ou la différence, exprimer, A ∪ B, A ∩ B,A \ B, A∆ B, en utilisant A, B, ⋆ et le passage au complémentaire.

Peut-on faire de même si ⋆ désigne ∆ ?

EH 7

Définitions 1) Deux ensembles A et B sont disjoints, si A ∩ B = ∅.(On note parfois A + B au lieu de A ∪ B si A et B sont disjoints).

2) Si (Ai)i∈I est une famille de parties de Ω on dira que la réunion⋃

i∈I

Ai est disjointe, si les ensembles

Ai sont deux à deux disjoints, c’est-à-dire si quels que soient i et j distincts dans I,

Ai ∩ Aj = ∅ .

(On note parfois∑

i∈I

Ai la réunion dans ce cas).

3) Une partition de Ω, est une famille (Ai)i∈I de parties non vides de Ω, telle quei) les Ai soient deux à deux disjoints,

ii)⋃

i∈I

Ai = Ω.

(Tout élément x de Ω appartient à un ensemble Ai et à un seul).

Exercice 3 Ecrire Ω comme la réunion de quatre sous-ensembles disjoints, construits à partir de Aet B à l’aide des opérations élémentaires sur les ensembles. Quand a-t-on une partition ?

2- Relation d’ordre dans P(Ω)

Définition Si A et B sont dans P(Ω), on dit que A est inclus dans B, si tout élément x de Aappartient aussi à B, et l’on note

A ⊂ B .

La relation binaire ainsi définie dans P(Ω) est

− réflexiveA ⊂ A ,

− antisymétrique

(A ⊂ B et B ⊂ A) implique A = B ,

− transitive

(A ⊂ B et B ⊂ C) implique A ⊂ C .

Cette relation est donc une relation d’ordre.

EH 8

C’est cette relation d’ordre qui sera toujours utilisée dans P(Ω). Quand on parlera de plus petit en-semble vérifiant une propriété, ce sera au sens de cette relation d’ordre. En particulier on dira qu’unesuite (An)n∈N de parties de Ω est croissante (resp. décroissante) si i ≤ j implique Ai ⊂ Aj (resp.Aj ⊂ Ai).

Formulaire

1) ∅ ⊂ A ⊂ Ω.2) A ∩ B ⊂ A ⊂ A ∪ B.3) Compatibilité de l’inclusion avec les lois ∩ et ∪ :

(A ⊂ B et A′ ⊂ B′) implique A ∩ A′ ⊂ B ∩ B′ et A ∪ A′ ⊂ B ∪ B′ .

4) Si A ⊂ B, alors BC ⊂ AC .

Exercice 4a) Soit (Ai)i∈N une suite de parties de Ω.

Ecrire⋃

i∈N

Ai comme réunion d’une suite croissante de parties de Ω.

b) Soit (Ai)i∈N une suite croissante de parties de Ω, écrire⋃

i∈N

Ai comme réunion disjointe de parties

de Ω.

Exercice 5 Est-ce que la relation d’inclusion est compatible avec la loi ∆ ?

Exercice 6 Montrer que les propriétés suivantes sont équivalentes :

i) A ⊂ B , ii) A ∩ B = A , iii) A ∪ B = B , iv) A \ B = ∅ , v) A∆ B = B \ A .

3- Fonction indicatrice (ou caractéristique) d’une partie de P(Ω)

Définition Si A appartient à P(Ω), on note 1lA et on appelle fonction indicatrice de A,l’application de Ω dans R, définie par

1lA(x) =

1 si x ∈ A0 si x /∈ A

.

Remarque : en théorie des ensembles cette fonction s’appelle fonction caractéristique de A, mais enprobabilité, la fonction caractéristique désigne une autre fonction. Nous utiliserons donc le terme de

EH 9

fonction indicatrice.

La fonction indicatrice de A caractérise exactement A au sens suivant

1lA = 1lB si et seulement si A = B ,

carA = x ∈ Ω | 1lA(x) = 1 .

Il est facile d’obtenir les relations suivantes :1) 1lΩ = 1 , 1l∅ = 02) 1lAC = 1 − 1lA3) 1lA∩B = 1lA · 1lB4) 1lA∪B = 1lA + 1lB − 1lA · 1lB.

En remarquant de plus que(1lA)2 = 1lA

puisque la fonction indicatrice ne prend que les valeurs 0 et 1, on a un moyen très simple de vérifierdes formules ensemblistes, en calculant les fonctions indicatrices des deux membres.

Propriété Les ensembles A et B sont disjoints si et seulement si

1lA∪B = 1lA + 1lB .

Exercice 7a) Trouver les fonctions indicatrices de A \ B et A∆ B en fonction de celles de A et B.b) Montrer la formule

(A∆ B) ∪ (A∆ C) = (A ∪ B ∪ C) \ (A ∩ B ∩ C) .

c) Montrer que la loi ∆ est associative.d) Montrer que ∩ est distributive sur la loi ∆ .e) Montrer, en prenant des exemples, que la réunion n’est pas distributive sur ∆ et que ∆ n’ estdistributive ni sur ∪ ni sur ∩.

Exercice 8 Montrer que l’application de P(Ω) dans l’ensemble des applications de Ω dans 0, 1 quià A associe 1lA est bijective et croissante.

Exercice 9 Montrer que si (Ai)i∈I est une partition de Ω, alors

1lA =∑

i∈I

1lA∩Ai,

EH 10

(cette somme ne contient jamais qu’un seul terme non nul au plus pour une valeur de x donnée dans Ω).

Exercice 10 On suppose que A et B sont des parties de Ω telles que A 6⊂ B, B 6⊂ A, A ∩ B 6= ∅,A ∪ B 6= Ω.

Soit C une partie de Ω construite à partir de A et B et des opérations élémentaires. Montrer qu’ilexiste un quadruplet (α, β, γ, δ) dans R4 et un seul, tel que

(1) 1lC = α + β 1lA +γ 1lB +δ 1lA 1lB .

(Pour l’existence, on pourra faire une récurrence sur le nombre d’opérations permettant de calculer C,et pour l’unicité, utiliser l’exercice 3).Trouver toutes les fonctions caractéristiques de la forme (1) et les ensembles associés.

4- Applications de Ω dans Ω′

On se donne deux ensembles Ω et Ω′, ainsi qu’une application f de Ω dans Ω′. On définit

− l’image réciproque f−1(B) d’une partie B de Ω′ par

f−1(B) = x ∈ Ω | f(x) ∈ B .

− l’image (directe) f(A) d’une partie A de Ω par

f(A) = f(x) |x ∈ A = y ∈ Ω′ | ∃x ∈ A, f(x) = y .

Formulaire

l’opération f−1 est compatible avec

1) le passage au complémentaire :(f−1(B))C = f−1(BC) .

(Le complémentaire de B est pris dans Ω′, celui de f−1(B) dans Ω).

2) la réunion

f−1

(

i∈I

)

=⋃

i∈I

f−1(Ai) .

3) l’intersection

f−1

(

i∈I

)

=⋂

i∈I

f−1(Ai) .

Exercice 11 Montrer que1lf−1(A) = 1lA f .

EH 11

(Cette formule permet de démontrer facilement les relations précédentes).

Exercice 12a) Quelle relation a-t-on entre f(A ∪ B) et f(A) ∪ f(B) ?

b) Quelle relation a-t-on entre f(A ∩ B) et f(A) ∩ f(B)− dans le cas général ?− si f est injective ?

c) Quelle relation a-t-on entre f(AC) et f(A)C

− dans le cas général ?− si f est injective ?− si f est surjective ?− si f est bijective ?(La fonction x 7→ x2, utilisée convenablement donne de nombreux contrexemples).

Exercice 13a) Soit f : x 7→ x2 de R dans R. Déterminer les ensembles suivants

f−1( ]−3, 1 ] ) , f−1( [ 1, 4 ] ) , f−1( ]−∞, 0 ] ) , f−1(1) .

b) Soit f : x 7→ arctan x de R dans R. Déterminer les ensembles suivants

f−1( [π/4, 2 ] ) , f−1( ] 0, π/3 [ ) , f−1( [−π/3, π/6 ] ) .

5- Tribu de parties d’un ensemble

Soit T un sous-ensemble de P(Ω). (Les éléments de T sont donc des parties de Ω et non des élémentsde Ω).

Définition On dira que T est une tribu sur Ω, si elle possède les propriétés suivantes :

(1) ∅ appartient à T

(2) Si A appartient à T , alors AC appartient à T ,(T est stable par passage au complémentaire).

(3) Si (An)n∈N est une suite d’éléments de T , alors⋃

n∈NAn appartient à T ,

(T est stable par réunion dénombrable).

Remarque : si l’on prend A1, . . . , An dans T et Ai = ∅ pour i ≥ n + 1, on déduit de (2) queA1 ∪ · · · ∪ An est dans T , donc T est stable par réunion finie.

EH 12

Conséquences

(4) Ω appartient à T .

(5) Si (An)n∈N est une suite d’éléments de T , alors⋂

n∈NAn appartient à T ,

(T est stable par intersection dénombrable, et aussi par intersection finie).

(6) Si A et B sont dans T , alors A \ B est dans T ,(T est stable par différence).

(4) En effet Ω = ∅C .

(5) Il suffit d’appliquer les formules de Morgan.

(6) Car A ∩ B, puis (A ∩ B)C sont dans T .

Exemple 1 L’ensemble P(Ω) est une tribu sur Ω. C’est la plus grande possible.

Exemple 2 Si A est une partie de Ω distincte de ∅ et de Ω, l’ensemble ∅, A, AC , Ω est une tribusur Ω. C’est la plus petite tribu contenant A.

Exercice 14 Montrer que toute tribu est stable par ∆ .

Exercice 15 Trouver la plus petite tribu sur R contenant A et B. (Voir exercice 10).

Exercice 16 Soit T une tribu sur R contenant les demi-droites ]−∞, a [ pour tout a réel. Montrerque T contient tous les intervalles.

(On montrera successivement que T contient les intervalles [ a, b [ , ] a, b [ , [ a, b ] , ]−∞, a ] , [ a, ∞ [ ,] a, ∞ [ ).

Exercice 17 Soit T1 un sous-ensemble de P(Ω) tel que

(1) ∅ appartient à T1,

(2) si A appartient à T1 et si B est inclus dans A, alors B appartient à T1

(3) T1 est stable par réunion dénombrable.

Soit T1 = A |AC ∈ T1 .

Montrer que si A est dans T1, et B dans T1 alors A ∪ B est dans T1. En déduire que T = T1 ∪ T1

est une tribu sur Ω.

Exercice 18 Soit T une tribu sur Ω, et A dans T . Soit

T′ = A ∩ B |B ∈ T .

EH 13

Montrer que T est une tribu sur A.

Exercice 19 Soit Ω et Ω′ deux ensembles, T ′ une tribu sur Ω′ et f une application de Ω dans Ω′.On pose

T = f−1(B) |B ∈ T′ .

Montrer que T est une tribu sur Ω.

Exercice 20 Soit S inclus dans P(Ω). Montrer que l’intersection des tribus sur Ω contenant S estune tribu sur Ω contenant S . (Plus petite tribu sur Ω contenant S ).

EH 15

Chapitre 2 - Ensembles dénombrables ou finis

1- Définitions et propriétés

Définitions Un ensemble Ω est dit dénombrable, s’il existe une bijection de Ω sur N∗. L’en-semble Ω est constitué d’une suite (xn)n∈N∗ d’éléments distincts

Ω = xn |n ∈ N∗ .

Un ensemble Ω est dit fini s’il existe une bijection de Ω sur une partie de N∗ de la forme 1, . . . , n.On dit alors que le cardinal de Ω vaut n, et l’on note

card Ω = n .

Si Ω est l’ensemble vide, on posecard Ω = 0 .

Montrer qu’un ensemble est dénombrable revient à trouver un moyen d’écrire la suite de ses éléments.

Remarque : si Ω n’est pas dénombrable, il contient nécessairement un sous-ensemble dénombrable.

Propriétés

(1) Si Ω est dénombrable, toute partie de Ω est dénombrable ou finie.(2) Soit I dénombrable ou fini, et pour tout i de I, soit Ωi dénombrable ou fini. Alors ∪i∈IΩi estdénombrable ou fini.(3) Si Ω1 et Ω2 sont dénombrables ou finis, le produit cartésien Ω1 × Ω2 est dénombrable ou fini.Plus généralement, si Ωi (1 ≤ i ≤ n) est dénombrable ou fini, le produit cartésien Ω1 × · · · × Ωn

l’est aussi.En particulier, si Ω est dénombrable ou fini, l’ensemble Ωp l’est aussi, pour tout entier p positif.(4) Si f est une bijection de Ω sur Ω′, et si Ω′ est dénombrable (resp. si card Ω = p), alors Ω estdénombrable (resp. card Ω = p).(5) Si f est une application de Ω dans Ω′, injective, et si Ω′ est dénombrable (resp. card Ω′ = p),alors Ω est dénombrable ou fini (resp. card Ω′ ≤ p).(6) Si f est une application de Ω sur Ω′, surjective, et si Ω est dénombrable, (resp. card Ω = p),alors Ω′ est dénombrable ou fini (resp. card Ω′ ≤ p).(7) Si Ω et Ω′ sont finis, et si card Ω = card Ω′, une application f de Ω dans Ω′ est bijective, dèsqu’elle est injective, ou surjective.

EH 16

(2) Idée de la démonstration. On écrit

Ωi = xi1 . . . , xik, . . . ,

et l’on forme la matrice (infinie) dont les éléments sont les éléments xik. On parcourt les diagonalessuccessives i + k = r, pour r ≥ 1, (en omettant les éléments que l’on aurait déjà obtenus).

x11 // x12

||yyyy

yyyy

x13 · · ·

x21

x22

<<yyyyyyyyx23 · · ·

x31

<<yyyyyyyyx32 x33 · · ·

Les premiers termes sont : x11, x12, x21, x31, x22, x13, . . . Tous les éléments de la matrice, donc de laréunion, sont obtenus par ce procédé au bout d’un nombre fini d’opérations.

(3) Même démonstration que dans (2) en formant la matrice des couples (xi, yj) de Ωi × Ωj. Ensuiteon procède par récurrence.

(4) Si Ω est dénombrable, soit ϕ une bijection de Ω′ sur N∗. Alors f ϕ est une bijection de Ω sur N∗,et Ω est dénombrable. Si Ω′ est fini, on remplace N∗ par 1, . . . , p.

(5) L’application f est une bijection de Ω sur f(Ω), et f(Ω) est un sous-ensemble de Ω′.Si Ω′ est dénombrable, alors f(Ω) est dénombrable ou fini, donc Ω aussi.

Si card Ω′ = p , alors card f(Ω) = card Ω ≤ p.

(6) Pour tout x de Ω′, soit g(x) un antécédent dans Ω de x par f . L’application g de Ω′ dans Ω estalors injective, et on applique (5).

(7) Si f est une application injective de Ω dans Ω′, alors f est une application bijective de Ω sur f(Ω),donc

card f(Ω) = card Ω = card Ω′ .

Comme f(Ω) est inclus dans Ω′ et a même cardinal que Ω′ ils sont égaux. Il en résulte que f estsurjective. Donc f est bijective.

Si f est une application surjective de Ω dans Ω′, pour tout y de Ω′ soit g(y) un antécédent. Alors g estune application injective de Ω′ dans Ω, donc g est bijective. Mais par construction, pour tout y de Ω′,on a

f g(y) = y ,

donc f = g−1 est aussi bijective.

Remarque : la propriété (7) est fausse si Ω et Ω′ sont seulement dénombrables :

EH 17

− l’application n 7→ 2n de N dans N est injective, mais pas surjective ;

− l’application n 7→ |n| de Z dans N est surjective mais pas injective.

Exercice 1

a) Montrer que Zp est dénombrable pour tout entier positif p.

b) Montrer que Qp est dénombrable pour tout entier positif p.

Exercice 2 Soit Z[x] l’ensemble des polynômes à coefficients entiers. Montrer que l’ensemble Zp[x]des polynômes de degré p à coefficients entiers est isomorphe à Z∗ × Zp. En déduire que Zp[x] estdénombrable.

Montrer que l’ensemble des nombres réels qui sont racines d’un élément au moins de Zp[x] (autre quele polynôme nul), est dénombrable. (Nombres algébriques).

Exercice 3 Montrer que R n’est pas dénombrable. (Faire un raisonnement par l’absurde :

si R = xi | i ≥ 1, en écrivant les nombres xi sous forme décimale, trouver un nombre x différent detous les xi).

Montrer par un procédé analogue que l’ensemble des suites de nombres entiers n’est pas dénombrable.

L’ensemble des suites de nombres entiers qui sont nulles à partir d’un certain rang, est-il dénombrable ?

Exercice 4 Soit Ω un ensemble, et T1 = A |A ⊂ Ω, A dénombrable ou fini. En utilisant le résultatdu chapitre 1 exercice 17, montrer que T1 ∪ T1 est une tribu sur Ω, et que c’est la plus petite tribucontenant les singletons.

2- Ensembles finis

Un certain nombre d’exercices de probabilité, consistent à déterminer le nombre d’éléments d’un en-semble fini. On rappelle ici quelques formules et propriétés à connaître.

Dans ce qui suit E et F sont deux ensembles finis tels que card E = p et card F = n.

Ensemble F (E,F ) des applications de E dans F

card F (E,F ) = np .

En effet, à tout élément de E on peut associer n valeurs possibles dans F . Comme il y a p élémentsdans E, on a donc n · n · · ·n possibilités, soit np.

Ensemble P(F ) des parties de F

card P(F ) = 2n .

EH 18

En effet, d’après ce qui précède card F (F, 0, 1) = 2n, or cet ensemble est l’ensemble des fonctionsindicatrices des parties de F . Donc card P(F ) = card F (F, 0, 1).

Arrangement de n objets p à p, où p ≤ n

Un tel arrangement est un p−uplet (x1, . . . , xp) formé d’éléments distincts de F .L’ordre des termes est donc important ((x1, x2) 6= (x2, x1)). Le nombre de ces arrangements est

Apn = n(n − 1)...(n − p + 1) =

n!

(n − p)!.

En effet, il y a n choix possibles pour x1, puis n−1 choix pour x2, n−2 choix possibles pour x3 etc. . . .

Si p ≤ n, le nombre d’arrangements de n objets p à p est aussi le nombre d’applications injectives deE dans F .

Permutation de n objets

Une permutation de n objets est un arrangement de n objets pris n à n. Le nombre de permutationsde n objets est donc

Ann = n! .

Si n = p, c’est aussi le nombre d’applications bijectives de E dans F .

Combinaisons de n objets p à p, où p ≤ n

Une combinaison de n objets p à p est un sous-ensemble de F à p éléments x1, . . . , xp.

L’ordre des termes n’a pas d’importance. (x1, x2 = x2, x1). Le nombre de combinaisons de n objetsp à p est

(

n

p

)

= Cpn =

n!

p!(n − p)!=

n(n − 1) · · · (n − p + 1)

p(p − 1) · · · 1 .

En effet, il y a un nombre p! de p−uplets (x1 . . . , xp) associés au même ensemble x1 . . . , xp, donc

(

n

p

)

=Ap

n

p!.

Rappel de formules

(

n

p

)

=

(

n

n − p

)

,

(

n

p

)

=

(

n − 1

p

)

+

(

n − 1

p − 1

)

(

n

1

)

=

(

n

n − 1

)

= n ,

(

n

0

)

=

(

n

n

)

= 1 .

EH 19

Formule du binôme de Newton

(a + b)n =

n∑

p=0

(

n

p

)

apbn−p ,

en particulier :n∑

p=0

(

n

p

)

= 2n ,

(on retrouve card P(F )),n∑

p=0

(−1)p(

n

p

)

= 0 .

Produit cartésien de deux ensembles

card(E × F ) = card E · card F .

Pour terminer voici quelques relations liant les cardinaux d’ensembles finis obtenus grâce aux opéra-tions élémentaires.

card(A ∪ B) + card(A ∩ B) = card A + card B

card(B \ A) = card B − card(A ∩ B)

si A ⊂ B, card(B \ A) = card B − card A

card(AC) = card Ω − card A .

Exercice 5 Calculer les sommes suivantes où E(x) désigne la partie entière de x.

E(n/2)∑

p=0

(

n

2p

)

,

E((n−1)/2)∑

p=0

(

n

2p + 1

)

,

E(n/2)∑

p=0

(−1)p(

n

2p

)

,

E((n−1)/2)∑

p=0

(−1)p(

n

2p + 1

)

,

E(n/4)∑

p=0

(

n

4p

)

.

Exercice 6 On suppose p ≤ n. Montrer que les nombres suivants sont égaux et trouver leur valeur.

− nombre d’éléments de (N∗)p dont la somme des termes vaut n.

− nombre de monômes de degré n de la forme xn1

1 xn2

2 · · · xnpp où ni 6= 0.

Exercice 7 Montrer que les nombres suivants sont égaux et trouver leur valeur commune (en utilisantl’exercice 6)

− nombre d’éléments de Np dont la somme des termes vaut n.

− nombre de monômes de degré n de la forme xn1

1 xn2

2 · · · xnpp où ni ≥ 0.

EH 20

− nombre de combinaisons de n objets pris p à p avec répétition (on prend p objets parmi n, sans tenircompte de l’ordre, mais en prenant éventuellement plusieurs fois le même objet).

Exercice 8 Trouver le nombre de façons de choisir p éléments d’un n−uplet donné, de telle sorte quedeux d’entre eux ne soient pas consécutifs.

Exercice 9 Soit Ω = 1, 2, . . . , n. On note xn le nombre de bijections de Ω sur lui même, sans pointfixe, c’est-à-dire de bijections f telles que, pour tout i,

f(i) 6= i .

a) Trouver une relation liant xn, xn−1, xn−2.b) On pose yn = (−1)n(xn − nxn−1). Montrer que (yn) est constante et en déduire une relation entrexn et xn−1.c) On pose zn = xn/n!. Trouver une relation entre zn et zn−1. En déduire la valeur de zn puis de xn.d) Que vaut lim

n→∞zn ?

Exercice 10 Soit Ω = x1, . . . , x10 où xi est un entier tel que 0 ≤ xi ≤ 106. Montrer qu’il existedeux ensembles disjoints A = a1, . . . , ar et B = b1, . . . , bq inclus dans Ω, tels que

a1 + · · · + ar = b1 + · · · + bq .

(On considérera l’application f de P(Ω) dans N, qui à une partie de Ω associe la somme de ses élé-ments).

Exercice 11 Trouver− Le nombre maximal de diagonales d’un polygone convexe à n côtés.− Le nombre maximal de points d’intersection de ces diagonales autres que les sommets.

Exercice 12 Soit Ω un ensemble de n lettres. On forme la somme

a∈Ω,b∈Ω,a6=b

a2b .

Combien y a-t-il de termes dans cette somme ?

Même question avec les sommes∑

a3b2c et∑

a2b2c.

Exercice 13

a) Calculer la somme

n∑

p=1

p

(

n

p

)

.

b) Soit Ω tel que card Ω = n. Soit A dans P(Ω) tel que card A = a. Chercher le nombre de parties Bde Ω telles que card(A ∩ B) = i, où 0 ≤ i ≤ a.En déduire la somme

S =∑

(A,B)∈P(Ω)2

card(A ∩ B) .

EH 21

Exercice 14 Soit Ω un ensemble fini, T un sous-ensemble de P(Ω) tel que(1) ∅ appartient à T ,(2) si A est dans T , alors AC est dans T ,(3) si A et B sont dans T , alors A ∪ B est dans T .Montrer que T est une tribu sur Ω.En prenant Ω = N, T1 = A ⊂ Ω |A fini, T1 = A ⊂ Ω |AC fini et T = T1 ∪ T1, montrer que T

satisfait les conditions (1) (2), (3) mais n’est pas une tribu sur Ω.

Exercice 15 Trouver une relation simple entre card(A ∪ B), card(A ∩ B) et card(A∆ B). Exprimercard(A∆ B) en fonction de card(A ∩ B), card A et card B.

3- Complément

De nombreux problèmes de probabilité font appel à des sommes :− somme de séries : les séries entières classiques sont à connaître ;− intégrales : les intégrales et les méthodes d’intégration classiques sont à connaître également ;− sommes finies : en plus de la formule du binôme de Newton, il est bon de connaître les sommes despremières puissances d’entiers. On note

Sk(n) =n∑

p=1

pk .

Les premières sommes sont les suivantes :

S0(n) = n , S1(n) = 1 + · · · + n =n(n + 1)

2

S2(n) = 12 + · · · + n2 =n(n + 1)(2n + 1)

6

S3(n) = 13 + · · · + n3 =

(

n(n + 1)

2

)2

= S1(n)2 .

Pour démontrer ces formules, on peut faire apparaître une relation permettant de calculer Sk(n) enfonction de S0(n), . . . , Sk−1(n) grâce à la formule du binôme, en partant du développement de (p+1)k+1.

(p + 1)k+1 =

k+1∑

i=0

(

k + 1

i

)

pi = pk+1 + (k + 1)pk +

k−1∑

i=0

(

k + 1

i

)

pi ,

on tire

pk =1

k + 1

(

(p + 1)k+1 − pk+1 −k−1∑

i=0

(

k + 1

i

)

pi

)

.

En sommant ces égalités pour p variant de 1 à n, on obtient

Sk(n) =1

k + 1

(

(n + 1)k+1 − 1 −k−1∑

i=0

(

k + 1

i

)

Si(n)

)

.

EH 22

En partant de S0(n) = n, on obtient successivement les valeurs S1(n), S2(n) etc . . .

Exercice 16 Calculer 12 + 32 + · · · + (2n + 1)2 et 14 + 24 + · · · + n4.

EH 23

Chapitre 3 - Espaces probabilisables - Probabilités

1- Espace probabilisable

Définition On appelle espace probabilisable, le couple (Ω,T ), formé d’un ensemble Ω nonvide et d’une tribu T sur Ω.

Rappelons quelques exemples d’espaces probabilisables rencontrés dans les chapitres précédents.

− (Ω,P(Ω)) en particulier si Ω est dénombrable ou fini.

− Si Ω n’est ni dénombrable, ni fini, on peut prendre T = T1 ∪ T1 où

T1 = A ⊂ Ω |A dénombrable ou fini(ex.4 Ch.2). C’est la plus petite tribu sur Ω contenant les singletons.

− Si A est une partie de Ω distincte de ∅ et de Ω, alors T = A,AC , ∅,Ω est la plus petite tribucontenant A.

Dans R, il existe une tribu importante, la tribu de Lebesgue, notée T0. C’est la plus petite tribu surR contenant les demi-droites ]−∞, a [ (ex.20 Ch.1). Elle contient donc aussi tous les intervalles.(ex.16Ch.1).

Les demi-droites (ou les intervalles) « engendrent » T0 en ce sens que de nombreuses propriétés sontvraies pour tout élément A de T0 dès qu’elles sont vraies pour les demi-droites ]−∞, a [ , ou les inter-valles. Nous admettrons par la suite ces propriétés.

2- Vocabulaire

Si (Ω,T ) est un espace probabilisable, les éléments de T sont appelés événements– ∅ est l’événement impossible– Ω est l’événement certain– AC est l’événement contraire de A– A ∪ B est l’événement A ou B– A ∩ B est l’événement A et B– Si A ∩ B = ∅, on dit que A et B sont incompatibles– Un système exhaustif Ai | i ∈ I est une partition de Ω.

3 - Probabilité

EH 24

Définition On appelle probabilité sur un espace probabilisable (Ω,T ), une application P deT dans [ 0, 1 ] telle que(1) P(Ω) = 1 .(2) pour toute suite (An)n≥0 d’éléments de T disjoints 2 à 2

P

n≥0

An

=

∞∑

n=0

P(An) .

On dit alors que (Ω,T , P) est un espace de probabilité ou espace probabilisé.

Propriétés

(1) P(∅) = 0(2) Si A1, . . . , An est une famille finie de parties 2 à 2 disjointes de Ω

P(A1 ∪ · · · ∪ An) = P(A1) + · · · + P(An) .

(3) P(AC) = 1 − P(A)(4) P(A \ B) = P(A) − P(A ∩ B)En particulier si B ⊂ A, on a P(A \ B) = P(A) − P(B).(5) Si B ⊂ A, P(B) ≤ P(A)(6) P(A ∪ B) + P(A ∩ B) = P(A) + P(B)(7) Si (An)n≥0 est une suite croissante d’éléments de T

P

n≥0

An

= limn→∞

P(An) .

(8) Si (An)n≥ est une suite décroissante d’éléments de T

P

n≥0

An

= limn→∞

P(An) .

(9) Pour toute suite (An)n≥0 d’éléments de T

P

n≥0

An

≤∑

n≥0

P(An) .

(10) Si (An)n≥0 est un système exhaustif, et si A est dans T ,

P(A) =∑

n≥0

P(An ∩ A) .

EH 25

(1) On prend An = ∅ pour tout n.

(2) On prend Ak = ∅ si k ≥ n + 1.

(3) AC ∪ A = Ω et AC ∩ A = ∅.

(4) (A \ B) ∪ (A ∩ B) = A et (A \ B) ∩ (A ∩ B) = ∅.

(5) P(A) − P(B) = P(A \ B) ≥ 0.

(6) A ∪ B = (A \ B) ∪ B.

(7) On écrit⋃

n≥0

An comme réunion disjointe des ensembles An+1 \ An. En supposant A0 = ∅ on a

P(⋃

n≥0

An) = P (⋃

n≥0

(An+1 \ An)) =∑

n≥0

(P(An+1) − P(An))

= limk→∞

k−1∑

n=0

(P(An+1) − P(An)) = limk→∞

P(Ak) .

(8) On applique (7) à (ACn ) grâce aux formules de Morgan.

(9) L’inégalité est vraie pour une réunion de deux ensembles, d’après (6), puis pour une réunion finie,par récurrence. Alors

P

(

k⋃

n=0

An

)

≤k∑

n=0

P(An) ≤∑

n≥0

P(An) .

Comme la suite

(

k⋃

n=0

An

)

est croissante, il résulte de (7) que la suite

(

P

(

k⋃

n=0

An

))

converge vers

P

n≥0

An

d’où le résultat.

(10) A est la réunion disjointe des An ∩ A.

Définition Un événement A est dit presque sûr si P(A) = 1 et négligeable, si P(A) = 0.

Exemple 1 Soit Ω non dénombrable et T la plus petite tribu sur Ω contenant les singletons. Cherchonsles probabilités sur (Ω,T ). On posera

p(x) = P(x) .

SoitEk = x ∈ Ω | 1/(k + 1) < p(x) ≤ 1/k .

EH 26

Si cet ensemble n’était pas fini, il existerait une suite (xn) dans Ek, mais

P(xn |n ≥ 0) =∑

n≥0

P(xn) ,

et comme p(xn) est supérieur à 1/(k +1) la série divergerait. On a une contradiction. Donc Ek est fini.Alors

E = x ∈ Ω | p(x) > 0 =⋃

k≥1

Ek

est dénombrable ou fini. DoncE = xn |n ∈ I

où I est inclus dans N. De plus la série∑

n∈I

P(xn) converge puisque sa somme vaut P(E).

La probabilité P est alors parfaitement définie par la fonction p.

Si D est dénombrable ou fini,

P(D) =∑

x∈D∩E

p(x) et P(DC) = 1 − P(D) .

Exemple 2 Si Ω est dénombrable ou fini, comme dans 1’exemple 1, les probabilités sont entièrementdéfinies par la donnée de la fonction p. Si Ω = xi | i ≥ 0, et si l’on pose

pi = p(xi) = P(xi) ,

on doit avoir∑

i≥0

pi = P

i≥0

xi

= P(Ω) = 1 .

La probabilité est entièrement déterminée dès que l’on se donne une série convergente positive desomme 1. On a alors

P(D) =∑

i | xi∈D

pi .

Le cas des ensembles finis sera repris plus en détail dans le chapitre suivant.

Exemple 3 Sur (R,T0), on peut définir des probabilités de plusieurs manières. En particulier, si fest une fonction continue par morceaux, positive, telle que

∞∫

−∞

f(t) dt = 1 ,

on peut définir une probabilité P telle que

P( [ a, b [ ) =

b∫

a

f(t) dt .

EH 27

On peut remarquer que les formules concernant les fonctions indicatrices ressemblent à celles concernantles probabilités :

− si A et B sont éléments deT ,

1lA\B = 1lA − 1lA∩B , P(A \ B) = P(A) − P(A ∩ B) ;

− si A et B sont disjoints

1lA∪B = 1lA + 1lB , P(A ∪ B) = P(A) + P(B) ,

− si A ⊂ B

1lA ≤ 1lB , P(A) ≤ P(B) .

etc...

Cette ressemblance n’est pas fortuite et trouve son explication dans la théorie de la mesure de Lebesgue.On pourra noter :

P(A) =

1lA dP ,

et remarquer que cette notation est cohérente avec ce que l’on peut attendre d’une intégrale. Parexemple, si A et B sont disjoints

P(A ∪ B) =

1lA∪B dP =

(1lA + 1lB) dP =

1lA dP +

1lB dP = P(A) + P(B) ,

et ceci est donc lié à la linéarité de l’intégrale. De même, si A est inclus dans B

P(A) =

1lA dP ≤∫

1lB dP = P(B) ,

et ceci est lié à la croissance de l’intégrale.

Nous ne chercherons pas à savoir, à ce niveau, quelle est la nature exacte de cette intégrale. Ellereprésentera uniquement une notation commode. Par exemple

dP = 1 .

Exercice 1 Calculer P(A∆ B) en fonction de P(A), P(B), P(A ∩ B).

Exercice 2 Trouver toutes les probabilités sur (Ω,T ) où T = ∅,Ω, A,AC.

Exercice 3 Soit (A1, . . . , An) n événements de T . Montrer la formule suivante (formule de Poincaré)

P(A1 ∪ . . . ∪ An) =n∑

k=1

(−1)k+1∑

1≤i1<i2<···<ik≤n

P(Ai1 ∩ · · · ∩ Aik) .

EH 28

Application : calculer P(A1∪A2∪A3) si P(Ai) = 1/2, P(Ai∩Aj) = 1/3 (i 6= j), et P(A1∩A2∩A3) = 1/4.

Exercice 4 Soit (Ai)i≥1 une suite d’éléments de T tels que

P(Ai) =1

i + 1, P(Ai ∩ Ai+1) =

1

i + 3,

etAi ∩ Aj = ∅ si |i − j| 6= 0 et 1 .

Calculer la probabilité de la réunion des Ai.

Exercice 5 Soit (An)n≥0 une suite d’événements de T tels que

limn→∞

P(An) = 0 .

Montrer que l’intersection des éléments An, est négligeable.Que peut-on dire de la réunion des An si lim

n→∞P(An) = 1 ?

4- Probabilité conditionnelle

Si (Ω,T , P) est un espace de probabilité, et si B est un élément de T de probabilité non nulle, ondéfinit une probabilité P′ sur (Ω,T ) en posant, si A appartient à T ,

P′(A) =P(A ∩ B)

P(B).

On vérifie facilement que les conditions définissant une probabilité sont satisfaites. Cette probabilitéest appelée probabilité de A sachant B, et se note P(A/B). On a donc

P(A ∩ B) = P(B) P(A/B) .

Cette notion a une interprétation intuitive qui se traduit par le nom qu’on lui a donné :sachant que l’événement B est réalisé, la probabilité que A se réalise est justement P(A/B).

Il faut faire bien attention de ne pas confondre P(A/B) et P(A ∩ B).

Remarque : ce qui précède permet de définir une probabilité sur (B,T ′) où

T′ = A ∩ B |A ∈ T .

Formule importante Soit (A1, . . . , An) un système exhaustif d’événements non négligeables,et B un événement de T . On a

P(B) =

n∑

i=1

P(Ai) P(B/Ai) .

EH 29

Cette somme n’est autre quen∑

i=1

P(B ∩ Ai) c’est-à-dire

P

(

n⋃

i=1

(B ∩ Ai)

)

= P(B) ,

puisque le système est exhaustif.

D’autres formules utiles sont données dans les exercices suivants.

Exercice 6 Soit (A1, . . . , An) des événements de T tels que, pour tout i, l’ensemble A1 ∩ · · · ∩Ai nesoit pas négligeable. Montrer que

P(A1 ∩ · · · ∩ An) = P(A1) P(A2/A1) · · ·P(Ai/A1 ∩ · · · ∩ Ai−1) · · · P(An/A1 ∩ · · · ∩ An−1) .

(Formule utile dans le cas de tirages exhaustifs).

Exercice 7 Formule de Bayes (probabilité des causes).

Soit (B1, . . . , Bn) un système exhaustif d’événements non négligeables, et A dans T non négligeable.Montrer que

P(Bi/A) =P(Bi) P(A/Bi)

n∑

j=1

P(Bj) P(A/Bj)

.

Application : une machine M1 fabrique 100 pièces dont 2% sont ratées. Une machine M2 fabrique 200pièces dont 3% sont ratées. On tire au hasard un objet parmi les 300 pièces. Cet objet est raté. Quelleest la probabilité pour qu’il vienne de M1 ?

5- Indépendance en probabilité

Définition Si A et B appartiennent à T on dit que A et B sont indépendants, si

P(A ∩ B) = P(A) P(B) .

Si B n’est pas négligeable, les événements A et B sont indépendants si et seulement si

P(A/B) = P(A) .

EH 30

Propriétés

(1) Si A et B sont indépendants, il en est de même de A et BC .(2) Si P(B) = 0, quel que soit A, les événements A et B sont indépendants.

(1)

P(A ∩ BC) = P(A \ B) = P(A) − P(A ∩ B) = P(A) − P(A) P(B) = P(A)(1 − P(B)) = P(A) P(BC) .

(2) Comme A ∩ B est inclus dans B, on a

0 ≤ P(A ∩ B) ≤ P(B) = 0 ,

alors

P(A ∩ B) = P(A) P(B) = 0 .

Exercice 8 Montrer que si P(B) = 1, alors, quels que soit A, les événements A et B sont indépendants.

Exercice 9 Soit A et B deux événements tels que P(A) = 1/5 et P(B) = 3/5. Calculer P(A ∪ B),P(A ∩ B), P(B/A) dans les cas suivants :

a) A et B sont incompatibles

b) A et B sont indépendants

c) P(A ∪ B) = 0, 7.

Exercice 10 Pour l’année 87/88 les étudiants de SM 1 du centre d’Epinal sont constitués de 69% degarçons et de 31% de filles. D’autre part 66% de garçons et 59% de filles on choisit la filière PC et lesautres la filière MP.

On choisit deux étudiants au hasard, l’un en MP, l’autre en PC. Déterminer la probabilité que les deuxétudiants soient de sexes différents.

Définition De manière plus générale, si (Ai)i∈I est une famille d’événements de T on dit queles Ai sont indépendants (globalement), si, pour toute partie finie J de I, on a

P

j∈J

Aj

=∏

j∈J

P(Aj) .

Cette propriété est globale. Elle implique que les événements Ai soient 2 à 2 indépendants, mais laréciproque est fausse.

EH 31

Remarque

La notion d’indépendance traduit déjà l’indépendance « physique » de deux événements :

si on lance un dé deux fois de suite, la probabilité d’obtenir deux fois le chiffre 6 est le produit desprobabilités d’amener 6 au premier jet, et 6 au second. Les deux lancements sont « physiquement »indépendants, et la probabilité est 1/6 × 1/6 = 1/36.

Mais la notion d’indépendance est plus générale que l’indépendance physique. Considérons les événe-ments suivants :− A est l’événement « amener une face paire avec un dé »,− B est l’événement « amener un chiffre supérieur ou égal à 5 »,− C est l’événement « amener un chiffre supérieur ou égal à 4 ».L’événement A ∩ B est : « amener un 6 », et l’on

P(A) =1

2, P(B) =

1

3, P(A ∩ B) =

1

6= P(A) P(B) ,

et les événements A et B sont indépendants.

Par contre l’événement A ∩ C est « amener 4 ou 6 » et l’on a

P(A) =1

2, P(C) =

1

2, P(A ∩ C) =

1

36= P(A) P(C) .

Les événements A et C ne sont pas indépendants.

Les deux situations sont « physiquement » semblables, alors qu’elles ne sont pas les mêmes du pointde vue de l’indépendance en probabilité.

Exercice 11 Montrer que si A, B, C sont indépendants, alors A, BC , C, sont indépendants, et queA est indépendant de B ∩ C, de B ∪ C, de B \ C et de B ∆ C.

Le résultat de l’exercice 11 se généralise et montre que la notion d’indépendance est stable pour lesopérations sur les ensembles :

Proposition Soit (A1, . . . , An, B1, . . . , Bm) des événements indépendants. Soit A construit àpartir de A1,. . . ,An et soit B construit à partir de B1,. . . ,Bm à l’aide des opérations élémentairessur les ensembles. Alors A et B sont indépendants.

Remarque : l’indépendance en probabilité peut s’écrire∫

1lA 1lB dP =

(∫

1lA dP

) (∫

1lB dP

)

.

EH 32

Exercice 12 Si A, B, C sont indépendants, calculer P(U) où

U = (A∆ B) ∩ (A∆ C) .

EH 33

Chapitre 4 - Espaces probabilisés finis

1- Rappels

L’univers Ω étant un ensemble fini x1, . . . , xn, on considère la tribu des parties de Ω. C’est unensemble fini de 2n éléments. Sur l’espace probabilisable (Ω,P(Ω)), on définit une probabilité P en sedonnant n nombres positifs (p1, . . . , pn) tels que

p1 + · · · + pn = 1 ,

et en posantP(xi) = pi .

Alors pour toute partie A de Ω,

P(A) =∑

i | xi∈A

pi .

2- Cas particulier : tirage au hasard

On peut prendre comme probabilité particulière, une probabilité P telle que

p1 = p2 = · · · = pn = λ .

On dit alors que les événements élémentaires sont équiprobables. On doit avoir

1 = p1 + · · · + pn = nλ ,

et donc

pi =1

n=

1

card Ω.

Alors, pour toute partie A de Ω,

P(A) =∑

i |xi∈A

1

card Ω=

card A

card Ω.

C’est le type de probabilité qui apparaît, dans le cas où l’on choisit « au hasard » parmi les élémentsd’un ensemble fini.

De telles probabilités se calculent donc en comptant les éléments de A et de Ω, et l’on est ramené à unproblème de dénombrement. On formule souvent cette probabilité en disant que

P(A) =nombre de cas favorables

nombre de cas possibles.

D’autre part, le fait que l’on définisse une probabilité en posant

P(A) =card A

card Ω,

EH 34

explique la ressemblance qui existe entre les formules de probabilité et celles des cardinaux d’ensembles.Par exemple,

P(A ∪ B) = P(A) + P(B) − P(A ∩ B)

card(A ∪ B) = card A + card B − card(A ∩ B) .

La seconde formule n’est qu’un cas particulier de la première, si l’on divise par card Ω.

Enfin, les probabilités conditionnelles s’expriment aussi très simplement dans ce cas.

P(A/B) =P(A ∩ B)

P(B)=

card(A ∩ B)

card B.

Exercice 1 Soit (Ω,P(Ω), P) où card Ω = n, et P est définie comme ci-dessus. Montrer que si A etB sont indépendants et distincts de Ω et de ∅, le nombre card Ω divise le produit card A × card B.

Que peut-on dire si card Ω, est un nombre premier ?

3- Quelques remarques

1) De nombreux exercices sont liés à des tirages « au hasard » de boules de cartes etc... On est doncamené à chaque fois à se placer dans un ensemble Ω dont les événements élémentaires sont équipro-bables. Le problème peut en général se traiter à plusieurs niveaux :

− soit en calculant des cardinaux d’ensembles et en divisant à la fin par card Ω pour avoir la probabilitécherchée,

− soit en raisonnant directement sur les probabilités.

Exemple 1 On jette deux dés. Quelle est la probabilité d’obtenir un nombre pair avec le premier dé,et un nombre divisible par 3 avec le second.

− Avec les cardinaux, on prend Ω = 1, . . . , 6 × 1, . . . , 6, de cardinal 36, et l’on cherche les couplessatisfaisant aux conditions imposées :

le premier dé peut prendre les valeurs : 2, 4 ou 6, le second 3 ou 6. Il y aura donc 6 couples possibles et

P(A) =6

36=

1

6.

− Avec les probabilités, on commence par calculer les probabilités des deux événements suivants :

A1 « tirer un nombre pair avec un dé »,

A2 « tirer un nombre divisible par trois avec un dé ».

L’ensemble Ω′ dans lequel on se place est 1, . . . , 6, de cardinal 6. On a bien sûr card A1 = 3 etcard A2 = 2, d’où

P(A1) =1

2et P(A2) =

1

3.

EH 35

Lorsque l’on lance les deux dés, on est ramené à calculer P(A1 ∩ A2) où A1 et A2 sont indépendants,et donc

P(A) = P(A1) P(A2) =1

2× 1

3=

1

6.

Cette seconde méthode à l’avantage d’utiliser des ensembles plus petits que dans la première.

2) Il faut de toute manière s’assurer que les événements élémentaires sont équiprobables. Sinon il faututiliser les formules générales et non celles avec les cardinaux.

Exemple 2 On jette deux dés. Quelle est la probabilité pour que les deux chiffres soient inférieurs auégaux à 2 ?

Comme le résultat ne fait pas intervenir l’ordre de lancement des dés, on pourrait penser utiliserl’ensemble des combinaisons avec répétitions, c’est-à-dire les ensembles de la forme i, j avec éven-tuellement i = j. Il y a 21 combinaisons de ce type. Mais les événements élémentaires ainsi obtenus nesont pas équiprobables. Il suffit de se placer dans l’ensemble des couples (i, j) pour voir que

P(i, j) =1

18si i 6= j et P(i, i) =

1

36,

car i, j, provient de deux couples (i, j) et (j, i), alors que i, i ne provient que du couple (i, i).

La probabilité cherchée est celle de 1, 1, 1, 2, 2, 2. Elle ne sera pas de 3/21, mais de 4/36 = 1/9.

3) Une question que l’on se pose souvent est « faut-il additionner ou faut-il multiplier certaines pro-babilités trouvées pour avoir le résultat ? »

L’addition correspond à la réunion d’événements incompatibles, c’est-à-dire d’ensembles disjoints.

On additionne donc lorsque l’on envisage des cas qui ne peuvent se produire simultanément : A seproduit ou B se produit, mais A et B ne peuvent se produire ensemble.

On a alors la formuleP(A ∪ B) = P(A) P(B) .

En particulier, il peut être intéressant de décomposer un événement en réunion d’événements incom-patibles. C’est l’application de la formule

P(A) =∑

i

P(A ∩ Ai)

où les Ai constituent un système exhaustif, c’est-à-dire une partition de Ω.

Exemple 3 On lance trois dés. Quelle est la probabilité d’obtenir au plus une fois le nombre 1 ?

On décompose l’événement en deux événements incompatibles :− A1 « obtenir une fois exactement le chiffre 1 »,

EH 36

− A2 « ne pas obtenir le chiffre 1 ».

L’ensemble Ω choisi est l’ensemble des triplets formés de nombres de 1 à 6, et donc card Ω = 63.

Pour former les triplets de A1, on choisit la position du chiffre 1 : il y a trois possibilités, et pourchacune de ces possibilités, il reste deux chiffres autres que 1 à placer, soit 52 possibilités. D’où

card A1 = 3 × 52 .

Pour A2 on choisit trois chiffres autres que 1, d’où

card A2 = 53 .

Alors

P(A) = P(A1) + P(A2) =3.52 + 53

63=

25

27.

La multiplication correspond à deux situations possibles.

a) L’intersection de deux événements indépendants.

Si A et B doivent avoir lieu en même temps et sont indépendants

P(A ∩ B) = P(A) P(B) .

L’exemple 1 montre qu’au niveau des ensembles, cela revient à calculer le cardinal d’un produit car-tésien. En effet, si A1 est inclus dans Ω1 et A2 dans Ω2, l’événement A = A1 × Ω2 et l’événementB = Ω1 × A2, sont tels que

A ∩ B = A1 × A2 ,

et donc

card(A ∩ B) = card A1 card A2 .

Comme

card(Ω1 × Ω2) = card Ω1 card Ω2 ,

on a donc

P(A ∩ B) =card(A ∩ B)

card(Ω1 × Ω2)=

card A1

card Ω1

card A2

card Ω2= P(A1) P(A2) .

Par ailleurs

P(A) =card(A1 × Ω2)

card(Ω1 × Ω2)=

card A1 card Ω2

card Ω1 card Ω2= P(A1) ,

et de même

P(B) = P(A2) ,

On voit donc que

P(A ∩ B) = P(A) P(B) .

EH 37

-

6

Ω2

A2

A1 Ω1

A ∩ B

=

A1 × A2

B = Ω1 × A2

A = A1 × Ω2

b) L’intersection de deux événements non indépendants.

Si A et B doivent avoir lieu en même temps, on a toujours, si B n’est pas négligeable,

P(A ∩ B) = P(B) P(A/B) .

Exemple 4 On jette deux dés. Soit A l’événement « la somme des deux chiffres est inférieure ou égaleà 4 », et B « le premier chiffre vaut 2 ». Quelle est la probabilité de A ∩ B ?

De manière évidente P(B) = 1/6. Par ailleurs, sachant B, la somme des deux chiffres est inférieure ouégale à 4 si et seulement si le second chiffre est inférieur ou égal à 2, et donc

P(A/B) =2

6=

1

3,

d’où

P(A ∩ B) =1

6× 1

3=

1

18.

(On aurait pu bien sûr, voir dans le produit cartésien 1, . . . , 62 qu’il n’y a dans A ∩ B que les deuxcouples possibles (2, 1) et (2, 2) ce qui donnait directement le résultat).

La généralisation de la formule utilisée est donnée dans l’exercice 6 du chapitre 3 :

P(A1 ∩ · · · ∩ An) = P(A1) P(A2/A1) · · ·P(Ai/A1 ∩ · · · ∩ Ai−1) · · · P(An/A1 ∩ · · · ∩ An−1) .

4- Quelques méthodes de calcul

1) Comptage direct des éléments.

EH 38

En écrivant explicitement l’ensemble Ω, (quand Ω n’est pas trop gros), il suffit de compter les élémentspour avoir la probabilité.

Remarque : si AC est plus petit que A, on a intérêt à calculer card AC . De même on a intérêt àcalculer card(A ∩ B) pour avoir card(A ∪ B) par la formule

P(A ∪ B) = P(A) + P(B) − P(A ∩ B) .

Au besoin, on peut s’aider d’un arbre pour ne pas oublier de cas.

Exemple 5 Une urne contient 4 boules : 2 rouges, 1 blanche et 1 noire. On tire les 4 boules au hasard,sans remise (tirage exhaustif). Quelle est la probabilité que l’une des deux dernières boules tirées aumoins soit rouge ?

On forme l’arbre des différents tirages possibles :

N // B

R

AAA

AAAA

A

>>

B // N

R // B

R

***

****

****

****

****

**//

IIN

AAA

AAAA

A

>>

B // R

R // N

B

AAA

AAAA

A

>>

N // R

R // N

R

AAA

AAAA

A

>>

B

AAA

AAAA

A

>>N // R

N // R // R

R // B

R

AAA

AAAA

A

>>

N

AAA

AAAA

A

>>B // R

B // R // R

EH 39

On obtient 12 cas possibles, et 10 cas favorables, d’où la probabilité cherchée de 10/12 = 5/6.

2) Obtention d’une formule grâce aux cardinaux d’ensembles finis (combinaisons, arrangements, pro-duits cartésiens etc...)

Il faut bien faire attention si l’on tient compte de l’ordre ou non (les deux situations peuvent intervenirdans le même calcul).

Exemple 6 Les billets de la loterie nationale ont 6 chiffres, tirés au moyen de 6 sphères. Calculer laprobabilité que le numéro gagnant soit formé de 3 chiffres distincts, l’un figurant 4 fois.

L’ensemble Ω est 0, 1, . . . , 96 et a pour cardinal 106. Pour trouver les nombres constituant l’événementA cherché, on choisit− les trois chiffres distincts parmi 10. On tient compte de l’ordre c’est donc le nombre d’arrangementsde 10 objets 3 à 3 : A3

10.− la position des quatre chiffres identiques. On tire donc 4 places parmi 6 sans tenir compte de l’ordre.C’est le nombre de combinaisons de 6 objets 4 à 4 : C4

6. D’où

card A = A310 C4

6 et P(A) =A3

10 C46

106.

3) Lorsque le résultat cherché dépend d’un paramètre entier n, établissement d’une relation de récur-rence permettant de calculer la probabilité pn soit explicitement, soit de proche en proche.

En général on essaye de voir comment former l’ensemble cherché au rang n en fonction des ensemblesaux rangs précédents.

Exemple 7 Soit E un ensemble à n = p.q éléments et Ω l’ensemble des partitions de E formées de psous-ensembles. On choisit au hasard une telle partition.Quelle est la probabilité que cette partition soit formée de p sous-ensembles à q éléments chacun ?Application : n = 6, p = 3, q = 2.

Il y a donc à déterminer deux cardinaux. Celui de Ω et celui de A, ensemble des partitions formées dep sous-ensembles à q éléments.

Pour Ω, notons N(n, p) le nombre de partitions d’un ensemble à n éléments en p sous-ensembles, ainsique

E = x1, . . . , xn et F = x1, . . . , xn−1et cherchons comment former une partition de E en p éléments à partir d’une partition de F . Il y adeux possibilités :

a) ajouter xn à une partition de F en p− 1 sous-ensembles : il y a donc N(n− 1, p− 1) façons de lefaire ;b) ajouter l’élément xn à l’un des p ensembles d’une partition de F en p sous-ensembles : il y a doncp N(n − 1, p) façons de le faire.

EH 40

On obtient donc la relation

N(n, p) = pN(n − 1, p) + N(n − 1, p − 1) ,

avec

N(n, 1) = N(n, n) = 1 .

On peut donc calculer de proche en proche tous les nombres N(n, p) (nombres de Stirling de deuxièmeespèce) grâce à un triangle analogue au triangle de Pascal

11 11 3 11 7 6 11 15 25 10 11 31 90 65 15 1

Par exemple N(6, 3) = 90.

Pour calculer le cardinal de A, notons P (p, q) le nombre de partitions d’un ensemble a p.q élémentsen p sous-ensembles de q éléments. Soit E′ un ensemble à (p − 1)q éléments. Si l’on a une partie à qéléments de E, on obtiendra un élément de A en lui ajoutant une partition quelconque de E′ en p− 1sous-ensembles à q éléments. Mais, ce faisant, on obtiendra p fois la même partition. Le nombre dechoix d’une partie à q éléments étant Cq

n on aura donc

P (p, q) =1

pCq

nP (p − 1, q) ,

avec de plus P (1, q) = 1. On en déduit facilement

P (p, q) =(pq)!

p! (q!)p,

et en particulier, P (3, 2) = 15. Donc dans ce cas

P(A) =15

90=

1

6.

Remarque : il n’est pas interdit de vérifier « à la main » que les formules obtenues sont exactes pourdes petites valeurs des paramètres.

4) Utilisation de formules de récurrence matricielles.

Dans certains cas où l’on a k probabilités dépendant d’un paramètre entier n, on peut trouver desrelations linéaires permettant de lier ces probabilités à celles obtenues au rang n − 1. Si l’on écrit lamatrice Un des probabilités au rang n, il apparaît une matrice A telle que

Un = A.Un−1 ,

EH 41

et donc, si A est une matrice carrée d’ordre k, indépendante de n,

Un = An. U0 .

On est ramené ainsi à un classique problème de calcul de puissance de matrice (ce qui peut se faire endiagonalisant par exemple, si c’est possible).

Exemple 8 On déplace une bille sur une barre contenant 3 positions avec la règle suivante :− si la bille est en 1 ou en 3 elle va en 2− si la bille est en 2, on choisit au hasard, une des positions 1 ou 3, et on y place la bille.

On répète l’opération n fois. On note Pn(i) la probabilité pour que la bille soit en i après n opérations.On suppose au départ, la bille en 2. On peut poser

P0(2) = 1 et P0(1) = P0(3) = 0 .

On exprime les Pn(i) en fonction des Pn−1(j).

La bille est en 2 à l’instant n si, ou bien elle était en 1 à l’instant n − 1, ou bien elle était en 3. D’où

Pn(2) = Pn−1(1) + Pn−1(3) .

La bille est en 1 à l’instant n, si elle était en 2 à l’instant n− 1. Or sachant qu’elle est en 2. elle va en1 avec probabilité 1/2, d’où

Pn(1) =1

2Pn−1(2) ,

et par symétrie

Pn(3) =1

2Pn−1(2) .

Si l’on note

Un =

Pn(1)Pn(2)Pn(3)

et A =

0 1/2 01 0 10 1/2 0

,

les trois relations ci-dessus s’écriventUn = A.Un−1 .

Or, on vérifie facilement queA3 = A ,

doncA2n+1 = A et A2n = A2 .

On en déduit, puisque

A2 =

1/2 0 1/20 1 0

1/2 0 1/2

,

les relationsP2n(1) = P2n(3) = 0 , P2n(2) = 1

EH 42

P2n+1(1) = P2n+1(3) = 1/2 , P2n+1(2) = 0 .

(Ces résultats étaient prévisibles).

Exercice 2 On jette deux dés. Soit les événements suivants :

A « la somme des faces est inférieure ou égale à 8 »

B « la somme des faces est divisible par 3 ».

Déterminer les probabilités suivantes :

P(A), P(B), P(A ∩ B), P(A ∪ B), PP (AC), P(B/A), P(A/B) .

Les événements A et B sont-ils : incompatibles ? indépendants ?

Exercice 3 Dans une tombola il y a 20 lots. On a distribué 1000 billets. Une personne en achète 5.Quelle est la probabilité de gagner un lot exactement ?

Exercice 4 Dans un jeu de 32 cartes, on choisit au hasard 4 cartes (tirages sans remise). Déterminerla probabilité des événements suivants :

A « l’une des cartes au moins est un as »

B « les quatre cartes sont de la même couleur » (il y a quatre couleurs)

C « il n’y a pas deux cartes ayant la même couleur »

D « il y a exactement deux cœurs et un as »

Exercice 5 On répartit au hasard 4 jetons sur un damier de 16 cases (4 lignes et 4 colonnes), chaquecase pouvant contenir un jeton au plus. Déterminer la probabilité des événements suivants :

A « il y a exactement un jeton par ligne et par colonne »

B « il y a exactement une colonne sans jeton »

C « il y a au moins une colonne sans jeton »

Exercice 6 Une personne à n clés dans sa poche et veut ouvrir sa porte dans l’obscurité. Elle prendau hasard les clés les unes après les autres et les essaye. On suppose que, lorsqu’une clé a été essayée,elle est mise de côté. Quelle est la probabilité de tirer la bonne clé à la k−ième tentative ?

Exercice 7 On dispose de n bâtonnets que l’on brise en 2 morceaux de longueurs inégales. Les 2nparties obtenues sont alors combinées au hasard en n paires (sans tenir compte de l’orientation ou dela position des bâtonnets dans la paire formée). Trouver la probabilité

a) d’avoir reconstitué les bâtonnets primitifs

b) que tous les nouveaux bâtonnets soient composés d’une partie longue et d’une courte .

Exercice 8 Une urne contient n boules noires et p blanches (n+p = k). On tire les boules au hasard,l’une après l’autre, sans remise. Trouver la probabilité des événements suivants :

A « la première et la k−ième boule tirées sont de couleurs différentes »

B « les boules noires sont tirées l’une à la suite de l’autre »

EH 43

C « deux boules noires sont tirées les dernières »

Exercice 9 Dans une assemblée, n personnes ont déposé leurs chapeaux à un vestiaire et repartent enprenant au hasard un des chapeaux. Quelle est la probabilité Pn pour qu’aucune des personnes n’aitson chapeau ? (Utiliser ex.9 chap. 2). Quelle est la limite de Pn lorsque n tend vers l’infini ?

Exercice l0 On dispose de trois dés que l’on lance. On considère que l’on a gagné dans l’un des cassuivants :a) l’ensemble des chiffres obtenus est 4, 2, 1b) sur les trois chiffres obtenus, on a tiré deux des trois chiffres 4, 2, 1, puis on a relancé un dé etobtenu le troisième des trois chiffres 4, 2, 1c) sur les trois chiffres obtenus, on a tiré un seul des trois chiffres 4, 2, 1, puis on a relancé deux déset obtenu les deux chiffres manquants parmi 4, 2, 1d) sur les trois chiffres obtenus, on a tiré un seul des trois chiffres 4, 2, 1, puis on a relancé deux déset obtenu un seul des deux chiffres manquants. Enfin on a relancé un dé et obtenu le dernier chiffre.Quelle est la probabilité de gain à ce jeu ?

Exercice 11 I - Préliminaire. Soit

A =

0 1 11 0 11 1 0

.

a) Montrer qu’il existe deux suites (xn)n≥0 et (yn)n≥0 d’entiers, telles que pour tout n

An = xnA + ynI .

b) Trouver deux relations de récurrence, liant xn+1 et yn+1 à xn et yn.

c) Trouver deux suites géométriques (un)n≥0 et (vn)n≥0 qui soient combinaisons linéaires de (xn)n≥0

et (yn)n≥0. En déduire un, vn, puis xn, yn, et enfin An.

II - Soient trois points placés sur un cercle. On se déplace avec la règle suivante :si l’on se trouve en un point à l’instant n, on se déplace vers l’un des deux autres points qui sontatteints avec la même probabilité 1/2.Trouver la probabilité Pn(i) d’être en i à l’instant n.Quelle est la limite de cette probabilité quand n tend vers l’infini ?

EH 45

Chapitre 5 - Variables aléatoires

1- Définition et exemples

Soit (Ω,T , P) un espace de probabilité.

Définition On appelle variable aléatoire (réelle) une application X de Ω dans R, telle que,X−1( ]−∞, x [ ) appartienne à T pour tout x réel.

Comme il l’a été signalé au chapitre 1, nous admettrons que cette définition entraine que, X−1(A)appartient à T , pour tout élément A de la tribu de Lebesgue T0. En particulier, X−1(I) appartient àT pour tout intervalle I de R.

Cette définition semble compliquée, mais en fait elle est naturelle si l’on veut, à partir de X, définirune probabilité sur (R,T0). En effet, on a la propriété suivante.

Propriété Si X est une variable aléatoire sur (Ω,T , P), on définit une probabilité p sur (R,T0),appelée probabilité image de P par X, en posant, pour tout élément A de T0,

p(A) = P(X−1(A)) .

(Vérification immédiate).

En quelque sorte, on transporte par X la probabilité P sur R et, dans beaucoup de cas, on « oublie »l’espace (Ω,T , P) pour ne plus voir que (R,T0, p).

On voit aussi que la définition donnée, dépend de la tribu choisie.

Par exemple, si T = P(Ω), toute fonction X sur Ω est une variable aléatoire.

Par contre si T = ∅,Ω, seules les fonctions constantes sont des variables aléatoires. En effet, si Xn’est pas constante, soit x1 et x2 deux réels de X(Ω) tels que x1 < x2. Alors soit ω1 un antécédent dex1 et ω2 un antécédent de x2. Si l’on prend x entre x1 et x2, l’ensemble X−1( ]−∞, x [ ) contient ω1,mais pas ω2, donc cet ensemble n’est ni Ω, ni vide, et n’appartient pas à la tribu.

Inversement, toute fonction constante est toujours une variable aléatoire, quelle que soit la tribu. En

EH 46

effet, si X = a, on a

X−1( ]−∞, x [ ) =

∅ si x ≤ aΩ si x > a

,

et ∅ et Ω se trouvent dans toutes les tribus sur Ω.

De même, si A appartient à T la fonction indicatrice de A est une variable aléatoire sur (Ω,T , P). Eneffet

1l−1A ( ]−∞, x [ ) =

∅ si x ≤ 0AC si 0 < x ≤ 1Ω si x > 1

.

Exercice 1 Soit A une partie de Ω, distincte de Ω et de ∅, et T = ∅,Ω, A,AC. Montrer que lesvariables aléatoires sur (Ω,T , P) sont les fonctions qui sont constantes sur A et sur AC .

Exercice 2 Soit Ω ni dénombrable, ni fini, et T = T1 ∪ T1 où

T1 = A ⊂ Ω |A dénombrable ou fini et T1 = A ⊂ Ω |AC ∈ T1 .

On rappelle que T est une tribu. On veut trouver les variables aléatoires sur (Ω,T , P).a) Soit I une partie de R ni dénombrable, ni finie. On pose

I0 = a ∈ R | ]−∞, a [∩ I dénombrable ou fini ,

I1 = a ∈ R | [ a, ∞ [∩ I dénombrable ou fini .

Montrer que I0 et I1 sont des demi-droites fermées, et que

sup I0 < inf I1 .

En déduire qu’il existe a dans R tel que ]−∞, a [∩ I et [ a, ∞ [∩ I ne soient ni dénombrables, ni finis.

b) Déduire de a) par l’absurde que X(Ω) est dénombrable ou fini, et que si X(Ω) = x1, . . . , xn, . . .,il existe un indice i et un seul tel que X−1(xi) soit le complémentaire d’une partie A dénombrableou finie.

Inversement montrer que toute fonction de ce type est une variable aléatoire sur (Ω,T , P).

Notations

Si X est une variable aléatoire, on notera

(X ∈ A) l’événement X−1(A)

(a ≤ X < b) l’événement X−1( [ a, b [ ) ,

EH 47

etc. . .

Cette notation est d’un emploi très agréable. Par exemple l’événement (X ∈ A) et (X ∈ B) estsimplement (X ∈ A ∩ B). Cela résulte de la relation

X−1(A) ∩ X−1(B) = X−1(A ∩ B) .

De même

(X ∈ A) ou (X ∈ B) = (X ∈ A ∪ B)

(X /∈ A) = (X ∈ A)C .

On notera également

(X ∈ A,Y ∈ B) = (X ∈ A) et (Y ∈ B) = (X ∈ A) ∩ (Y ∈ B) .

Remarques : on conviendra de ne pas distinguer deux variables aléatoires X et Y telles que l’événe-ment (X 6= Y ) soit négligeable, c’est-à-dire deux variables qui diffèrent seulement sur un ensemble deprobabilité nulle.

De même on pourra autoriser X à être infinie en certains points, à condition que l’ensemble (X = ∞)soit négligeable.

L’étude des variables aléatoires ne peut se faire de manière générale que dans le cadre de la théorie dela mesure de Lebesgue. Dans ce cadre, on peut démontrer que si X et Y sont des variables aléatoiressur (Ω,T , P) il en est de même de X + Y , X.Y , X/Y si (Y = 0) est négligeable.

De même, si f est une fonction continue sur R, la fonction f X est une variable aléatoire. Par exemple,Xn (si n est entier), |X|a si a est un réel positif, eX , ln |X| si (X = 0) est négligeable, sont des variablesaléatoires.

Exercice 3 Soit X une variable aléatoire sur (Ω,T , P) et λ et µ deux nombres réels. Montrer, enrevenant à la définition, que λX + µ est une variable aléatoire.

Exercice 4 Soit X et Y deux variables aléatoires sur (Ω,T , P) et A dans T . On pose

Z(ω) =

X(ω) si ω ∈ AY (ω) si ω ∈ AC .

Montrer que Z est une variable aléatoire.Montrer en particulier que sup(X,Y ) et inf(X,Y ) sont des variables aléatoires.

2- Fonction de répartition

EH 48

Définitions Si X est une variable aléatoire sur (Ω,T , P), on définit la fonction de répartitionFX de la variable X en posant, pour tout x réel,

FX(x) = P(X < x) .

Lorsque deux variables aléatoires X et Y ont la même fonction de répartition, on dira dans ce casqu’elles ont même loi.

Propriétés La fonction FX est une application de R dans [ 0, 1 ] croissante, continue à gauche,possédant une limite à droite en tout point, et telle que

limx→+∞

FX(x) = 1 et limx→−∞

FX(x) = 0 .

De plus en tout point a de R,

P(X = a) = limx→a

x>a

FX(x) − FX(a) .

Le nombre P(X = a) représente le saut de la fonction FX en a. Donc FX est continue en a, si etseulement si P(X = a) est nul.

La croissance résulte du fait que, si x < x′, on a l’inclusion

(X < x) ⊂ (X < x′) .

Pour les autres propriétés, on utilise un lemme d’analyse du type suivant.

Lemme Soit f est une application de R dan R. On a

limx→a

x<a

f(x) = s

si et seulement si, pour toute suite croissante (xn)n≥0 qui converge vers a, la suite (f(xn))n≥0

converge vers s.

(Ce résultat est vrai que a soit fini ou non, et se transpose facilement pour une limite par valeursupérieure).

En admettant ce lemme, démontrons par exemple la continuité à gauche de FX en un point a.

Si (xn)n≥0 est une suite croissante qui converge vers a, les événements (X < xn) constituent une suite

EH 49

croissante dont la réunion est (X < a). Donc, d’après une propriété des probabilités

limn→∞

P(X < xn) = P(X < a) ,

c’est-à-direlim

n→∞FX(xn) = F (a) ,

et, d’après le lemme, cela prouve que

limx→a

x>a

FX(x) = F (a) .

Si a = +∞, la suite (X < xn)n≥0 a pour réunion Ω, et donc

limx→+∞

P(X < xn) = P(Ω) = 1 ,

ce qui prouve quelim

x→+∞FX(x) = 1 .

Pour la limite à droite en un point a, on prend cette fois une suite (xn)n≥0 décroissante et de limite a.Alors la suite (X < xn)n≥0 est décroissante, donc la suite (P(X < xn))n≥0 converge vers la probabilitéde l’intersection de ces événements, c’est-à-dire vers

P(X ≤ a) = P(X < a) + P(X = a) .

On en déduit quelimx→ax>a

FX(x) = FX(a) + P(X = a) .

Enfin si a = −∞ l’intersection des événements (X < xn) est vide, et

limx→−∞

P(X < xn) = P(∅) = 0 ,

et donclim

x→−∞FX(x) = 0 .

Formules Si a < b, on aP(a ≤ X < b) = FX(b) − FX(a) ,

et pour tout réel aP(X ≥ a) = 1 − FX(a) .

Pour la première formule(a ≤ X < b) = (X < b) \ (X < a)

et (X < a) est inclus de (X < b).

EH 50

Pour la seconde(X ≥ a) = (X < a)C .

AttentionP(a < X < b) = FX(b) − FX(a) − P(X = a) ,

et ce n’est que dans le cas où P(X = a) est nul que l’on obtient

P(a < X < b) = FX(b) − FX(a) .

Exercice 5 A l’aide de la fonction de répartition, écrire les probabilités suivantes :

P(a ≤ X ≤ b) , P(a < X ≤ b) , P(X > a) .

Exercice 6 Montrer que si a < b, on a

limx→a

x>a

FX(x) ≤ FX(b) .

Exercice 7 Montrer que pour tout entier n > 0, l’ensemble

En = x ∈ R | limy→x

y>x

FX(y) − FX(x) > 1/n

est fini. En déduire que l’ensemble E des points de R où FX n’est pas continue est dénombrable oufini.

Exercice 8 Montrer que si X ≤ Y on a FX ≥ FY .

Exercice 9 Soit X une variable aléatoire. Exprimer en fonction de FX la fonction de répartition desvariables suivantes :

λx + µ (λ et µ réels) , |X| ,√

X (si P(X < 0) = 0) , X2 , eX .

3- Vecteur aléatoire

On peut considérer des variables aléatoires X1, . . . ,Xn sur (Ω,T , P) et former le n−uplet (X1, . . . ,Xn)c’est donc une application de Ω dans Rn. On définit la fonction de répartition de ce n−uplet en posant

F(X1,...,Xn)(x1, . . . , xn) = P(X1 < x1, . . . ,Xn < xn) .

C’est une application de Rn dans [ 0, 1 ] .

Cette fonction permet de retrouver la fonction de répartition des variables aléatoires qui composent len−uplet (lois marginales). Par exemple, pour un couple

FX(x) = limy→+∞

F(X,Y )(x, y) et FY (y) = limx→∞

F(X,Y )(x, y) .

EH 51

(Démonstration analogue à celle faite pour la continuité de FX).

Remarque : si l’on a deux variables aléatoires, on peut définir

P(X ∈ A,Y ∈ B) = P((X,Y ) ∈ A × B)

quels que soient A et B dans T0. Mais on peut définir également P((X,Y ) ∈ U) pour des parties U deR2 qui ne sont pas de la forme A×B, en fait pour tous les éléments d’une tribu sur R2 qui est la pluspetite tribu contenant les éléments A × B où A et B sont dans T0 (tribu produit).

Exemple L’ensemble

U = (x, y) | 0 < x < y < 1s’écrit comme réunion des ensembles

Un =

n−1⋃

i=1

] i/n, (i + 1)/n [× ] 0, i/n [ .

Donc U est réunion dénombrable d’éléments de la forme A×B. Cela permet de définir des événementsdu type

(X2 + Y 2 ≤ 1) = ((X,Y ) ∈ (x, y) |x2 + y2 ≤ 1) = ω ∈ Ω |X(ω)2 + Y (ω)2 ≤ 1 .

4- Variables aléatoires indépendantes

Définition Deux variables X et Y sont dites indépendantes, si, quels que soient A et B dansT0, on a

P(X ∈ A,Y ∈ B) = P(X ∈ A) P(Y ∈ B) ,

c’est-à-dire si les événements (X ∈ A) et (Y ∈ B) sont indépendants.

En fait, comme toujours pour la tribu T0, il suffit de vérifier cette propriété pour les demi-droites. Cecis’écrit

P(X < x, Y < y) = P(X < x) P(Y < y) ,

et l’on voit réapparaître les fonctions de répartition. On a donc une autre caractérisation des variablesindépendantes.

Proposition Deux variables X et Y sont indépendantes, si et seulement si, quel que soit (x, y)dans R2

F(X,Y )(x, y) = FX(x)FY (y) .

EH 52

Ces définitions se généralisent pour une famille (Xi)i∈I de variables aléatoires.

Définition Une famille (Xi)i∈I de variables aléatoires sera indépendante si pour toute famillefinie (Xj)j∈J et tout ensemble (Aj)j∈J d’éléments de T0, on a

P

j∈J

(Xj ∈ Aj)

=∏

j∈J

P(Xj ∈ Aj) ,

et l’on peut là aussi se contenter des demi-droites.

Remarquons que, comme pour les événements indépendants, cette notion est globale. Il ne suffit pasque les variables aléatoires soient deux à deux indépendantes pour qu’elles le soient globalement.

Exercice 10 Soit X, Y et Z trois variables indépendantes, et A dans T0. On pose

T (ω) =

X(ω) si Z(ω) ∈ AY (ω) si Z(ω) ∈ AC .

Déterminer la fonction de répartition de T .

Exercice 11 Soit X et Y deux variables indépendantes. Trouver la fonction de répartition desup(X,Y ), et de inf(X,Y ).

5- Espérance mathématique

La notion d’espérance est importante en probabilité. Malheureusement elle ne peut se définir en toutegénéralité que dans le cadre de la théorie de Lebesgue. En effet il s’agit de donner un sens à uneintégrale de la forme

XdP pour certaines variables aléatoires X.

Nous supposerons dans ce paragraphe qu’il existe un sous-espace vectoriel, noté L1(dP) de l’espacevectoriel des variables aléatoires sur Ω, et une forme linéaire E sur L1(dP), possédant les propriétéssuivantes :

1) si X ∈ L1(dP) et X ≥ 0 alors E(X) ≥ 0 ;

2) si 0 ≤ X ≤ Y et Y ∈ L1(dP) alors X ∈ L1(dP)

3) pour tout élément A de T , la fonction 1lA appartient à L1(dP) et

E(1lA) = P(A) .

EH 53

(En particulier E(1) = 1 ).

Remarque : en fait la définition que l’on donne de L1(dP) en théorie de Lebesgue est telle que Xappartient à L1(dP) si et seulement si |X| est dans L1(dP).

On notera

E(X) =

X dP .

On remarquera que cette notation est cohérente avec celle utilisée dans le chapitre 3, puisque, pourune fonction indicatrice, nous supposons que

E(1lA) = P(A) .

Nous verrons dans les chapitres suivants comment donner un sens à E dans deux cas précis. Dansl’immédiat, nous donnons quelques propriétés générales déduites des propriétés supposées ci-dessus,ainsi que des définitions qui seront réutilisées dans la suite.

Propriétés

(1) L’application E est croissante sur L1(dP).(2) Si |X| est dans L1(dP), alors il en est de même de X et

|E(X)| ≤ E(|X|) .

(3) Inégalité de Markov. Soit X dans L1(dP) et positive, alors, pour tout nombre réel α > 0, on a

P(X ≥ α) ≤ 1

αE(X) .

(4) Si X ≥ 0, et si E(X) = 0, alors X = 0 presque sûrement.

(1) Si X et Y sont dans L1(dP), avec X ≤ Y , alors Y − X est dans L1(dP) et est positive, doncd’après la propriété 1), le nombre E(Y − X) est positif et, par linéarité de E, le nombre E(Y )− E(X)également. On a bien

E(X) ≤ E(Y ) .

(2) PosonsX+ = sup(X, 0) et X− = − inf(X, 0) .

On voit facilement queX = X+ − X− et |X| = X+ + X− ,

et que0 ≤ X+ ≤ |X| et 0 ≤ X− ≤ |X| .

Alors d’après la propriété 2) les variables X+ et X− sont dans L1(dP), et

|E(X)| = |E(X+) − E(X−)| ≤ |E(X+)| + |E(X−)| = E(X+) + E(X−) = E(|X|) .

EH 54

(3) Posons A = (X ≥ α). On a les inégalités

0 ≤ α 1lA ≤ X 1lA ≤ X ,

donc0 ≤ E(α 1lA) ≤ E(X) ,

c’est-à-direα P(A) ≤ E(X) .

(4) D’après (3), en prenant α = 1/n, on obtient

0 ≤ P(X ≥ 1/n) ≤ nE(X) = 0 ,

doncP(X ≥ 1/n) = 0 .

Or l’événement (X > 0) est réunion croissante des événements (X ≥ 1/n), donc

P(X > 0) = limn→∞

P(X ≥ 1/n) = 0 ,

et, par suite,P(X = 0) = 1 ,

c’est-à-direX = 0 p.s. .

Notation

Si p est un entier supérieur ou égal à 1, on note Lp(dP) l’ensemble des variables aléatoires X telles que|X|p soit dans L1(dP).

Proposition-définition

Si X appartient à L2(dP), alors X est dans L1(dP), X − E(X) est dans L2(dP), et l’on appellevariance de X, le nombre

Var X = E((X − E(X))2) .

On a égalementVar X = E(X2) − E(X)2 .

Puisque (|X| − 1)2 est positif, on a toujours

|X| ≤ 1

2(X2 + 1) ,

et comme X2 et 1 sont dans L1(dP), il en sera de même de |X|. Alors

(X − E(X))2 = X2 − 2E(X)X + E(X)2

EH 55

se trouve dans L1(dP). De plus, en raison de la linéarité,

E((X − E(X))2) = E(X2) − 2E(X)E(X) + E(X)2 = E(X2) − E(X)2 .

La variance de X est donc un nombre positif, et d’après la propriété (4) ci-dessus, Var X = 0 si etseulement si (X − E(X))2 est nul p.s. c’est-à-dire si et seulement si

X = E(X) p.s. ,

donc uniquement dans le cas des variables presque sûrement constantes.

Définition Si X est dans L2(dP), on appelle écart-type de X, le nombre

σX =√

Var X .

Propriété Soit X dans L2(dP). Alors, quels que soient a et b réels, la variable aléatoire aX + best dans L2(dP), et

Var(aX + b) = a2 Var X .

En effetaX + b − E(aX + b) = a(X − E(X)) ,

d’oùE((aX + b − E(aX + b))2) = E(a2(X − E(X))2) = a2E((X − E(X))2 .

Conséquence

Si X est dans L2(dP), et si l’on pose

E(X) = m et σX = σ ,

alors la variable aléatoire Y définie par

Y =X − m

σ

est telle queE(Y ) = 0 et σY = 1 .

Inversement, si Y est une variable aléatoire telle que

E(Y ) = 0 et σY = 1 ,

alors, quels que soient m réel, et σ réel positif, la variable X définie par

X = σY + m

EH 56

est telle que

E(X) = m et σX = σ .

On dira qu’une variable aléatoire X est centrée lorsque

E(X) = 0

et réduite lorsqueσX = 1 .

Inégalité de Bienaymé-Tchebychev

Propriété Soit X dans L2(dP) alors, pour tout t > 0, on a, avec les notations précédentes,

P

(∣

X − m

σ

≥ t

)

≤ 1

t2.

Cette inégalité résulte de celle de Markov appliquée à la variable aléatoire

(

X − m

σ

)2

avec α = t2.

On obtient alors

P

(

(

X − m

σ

)2

≥ t2

)

≤ 1

t2E

(

(

X − m

σ

)2)

=1

t2Var

X − m

σ=

1

t2

puisque (X − m)/σ est centrée réduite.

Remarque : en posant ε = σ t, on obtient une antre formulation de l’inégalité

P(|X − m| ≥ ε) ≤ σ2

ε2.

Définition Si X appartient à Lp(dP), on appelle moment d’ordre p, le nombre E(Xp).

Exercice 12 Montrer que, pour tout x ≥ 0, on a, pour p ≥ 2,

xp−1 ≤ p − 1

p(xp + 1) .

En déduire que si X est dans Lp(dP), alors X appartient à Lk(dP) pour tout entier k compris entre 1et p.

EH 57

Proposition-définition Soit X et Y dans L2(dP). Alors la variable X.Y est dans L1(dP), ainsique (X − E(X))(Y − E(Y )), et l’on définit la covariance de (X,Y ) par

Cov(X,Y ) = E((X − E(X))(Y − E(Y ))) ,

et l’on a aussiCov(X,Y ) = E(X.Y ) − E(X) E(Y ) .

On a l’inégalité

|X.Y | ≤ 1

2(X2 + Y 2) ,

ce qui montre que X.Y est dans L1(dP). D’autre part

(X − E(X))(Y − E(Y )) = X.Y − E(X)Y − E(Y )X + E(X) E(Y )

et toutes ces variables sont dans L1(dP). En prenant l’espérance de cette égalité on trouve facilementque

E((X − E(X))(Y − E(Y ))) = E(X.Y ) − E(X) E(Y ) .

Remarque : on aCov(X,X) = Var X .

Propriétés Les relations suivantes sont équivalentes :

Cov(X,Y ) = 0 ,

E(X.Y ) = E(X) E(Y ) ,

Var(X + Y ) = Var X + Var Y .

Vérification immédiate.

Exercice 13 Calculer Cov(aX + b, cY + d) en fonction de Cov(X,Y ).

Exercice 14 Montrer que L2(dP) est un espace vectoriel, et que l’application qui à (X,Y ) associeCov(X,Y ) est une forme bilinéaire, symétrique positive. Quels sont les vecteurs isotropes ?

EH 59

Chapitre 6 - Variables discrètes

Dans ce chapitre, nous reprenons dans un cas particulier, les notions introduites au chapitre 5. Aupa-ravant, le paragraphe 1 donne quelques résultats sur les séries et les sommes multiples.

1- Séries : rappels et compléments

Si (xn)n≥0 est une suite de nombres réels ou complexes on note

Sn =

n∑

p=0

xp ,

et l’on dira que la série de terme général xn converge si la suite (Sn)n≥0 admet une limite finie S,appelée somme de la série. On note alors

∞∑

p=0

xp = S .

Si la suite n’a pas de limite, ou si elle a une limite infinie, on dit que la série diverge.

Dans le cas où la suite (xn)n≥0 est positive, la suite (Sn)n≥0 est croissante et possède donc une limitefinie ou non. Dans ce cas

∞∑

p=0

xp = +∞ si la série diverge

ou∞∑

p=0

xp < +∞ si la série converge.

On dira que la série de terme général xn converge absolument si la série de terme général |xn| converge.

Théorème 1 Si la série de terme général xn converge absolument, alors elle converge, et

∞∑

p=0

xp

≤∞∑

p=0

|xp| .

Théorème 2 Pour une série à termes positifs ou pour une série qui converge absolument, la sommede la série ne dépend pas de la façon dont on regroupe les termes, ni de l’ordre de ces termes.

EH 60

Par exemple, on peut sommer les termes de rang pair, et ceux de rang impair. En particulier

∞∑

p=0

xp =

∞∑

p=0

x2p +

∞∑

p=0

x2p+1 .

Cas des sommes multiples.

Si l’on a une famille (xij)i≥0,j≥0 dépendant de deux indices il n’existe plus de façon « naturelle » d’eneffectuer la somme. Il est en effet possible s’imaginer plusieurs façons de sommer :− sommer d’abord en i puis en j,

− sommer d’abord en j puis en i,− sommer les termes tels que i + j = n, puis faire varier n, etc. . .

L’exemple des séries montre qu’il y a peu d’espoir que ces diverses sommes donnent le même résultat,sans condition supplémentaire.

Théorème 3 Pour une somme multiple à termes positifs, on obtient toujours le même résultat,fini ou non, quelle que soit la manière dont on effectue la sommation.

Définition On dira que la famille (xij)i≥0,j≥0 est sommable, si la famille (|xij |)i≥0,j≥0 a unesomme finie.

Théorème 4 Pour une famille sommable, on obtient le même résultat (nécessairement fini) quelleque soit la manière dont on effectue la sommation.

Exercice 1 Soit q dans ]−1, 1 [ . Montrer que la famille ((−1)nqn+m+2nm)m≥0,n≥0 est sommable, eten déduire l’égalité

∞∑

n=0

qn

1 + q2n+1=

∞∑

n=0

(−1)nqn

1 − q2n+1.

Exercice 2 Soit q dans ] 0, 1 [ . Calculer

∞∑

n=0

(

n∑

m=0

qn+m

(2m)!

)

.

Exercice 3 Si |p| + |q| < 1, calculer∑

(i,j)∈N2

(

i + j

i

)

piqj .

EH 61

2- Définition

Définition Soit X une variable aléatoire sur (Ω,T , P). On dit que X est discrète, si l’ensembleX(Ω) est une partie dénombrable ou finie de R.

On a doncX(Ω) = xi | i ∈ I ,

où I est une partie de N.

Remarque 1 : il suffit en fait qu’il existe une partie dénombrable ou finie A de R telle que l’on aitP(X ∈ A) = 1.

Remarque 2 : dans les exemples classiques, X(Ω) est une suite (x1, x2, . . . , xn, . . .) strictement crois-sante et tendant vers l’infini (N par exemple). Certaines propriétés seront données uniquement dans cecas.

Définition On appelle distribution de probabilité de la variable discrète X, l’ensemble descouples (xi, pi) , où xi décrit X(Ω), et où

pi = P(X = xi) .

Cette suite (pi)i∈I est donc telle que∑

i∈I

pi = 1 .

En effet∑

i∈I

P(X = xi) = P

(

X ∈⋃

i∈I

xi)

= P(X ∈ X(Ω)) = P(Ω) = 1 .

Inversement, toute suite (xi, pi)i∈I pourra être considérée comme la distribution de probabilité d’unevariable discrète X, si et seulement si les pi sont des nombres positifs dont la somme vaut 1.

Dans ce cas, pour toute partie A de T0,

P(X ∈ A) =∑

i |xi∈A

pi .

En particulier

FX(x) = P(X < x) =∑

i | xi<x

pi .

EH 62

Exercice 4 Soit X 1a variable discrète de distribution (1, 1/2), (2, 1/3), (5, k). Déterminer k.Calculer P(x ∈ ] 1, 5 [ ), P(X < 3), P(X > 2).

Proposition Lorsque la suite xi est croissante la fonction de répartition FX de X est en escalieret

pi = FX(xi+1) − FX(xi) .

De plus, si X(Ω) = x1, . . . , xn est fini,

pn = 1 − FX(xn) .

Si xi < x ≤ xi+1, on a

FX(x) = P(X < x) = P(X < xi+1) = FX(xi+1) ,

donc FX est constante sur ]xi, xi+1 ] , et en particulier

limx→xix>xi

FX(x) = FX(xi+1) ,

d’où

pi = P(X = xi) = limx→xix>xi

FX(x) − FX(xi) = FX(xi+1) − FX(xi) .

Exemple 1 Variable certaine X = α de distribution (α, 1).

-

6

1

α

Exemple 2 Fonction indicatrice X = 1lA où A 6= ∅, Ω, de distribution (0, P(AC )), (1, P(A)).

EH 63

-

6

1

1

P(AC)

Exercice 5 On tire une carte d’un jeu de 52 cartes. A un as on associe 4, à une dame 2, à un roi 3, àun valet 1 et 0 à une autre carte. Déterminer la fonction de répartition de la variable aléatoire X ainsidéfinie, et représenter cette fonction.

Remarque : deux variables aléatoires discrètes ont la même loi si et seulement si elles ont mêmedistribution de probabilité, car la fonction de répartition est parfaitement déterminée par la distributionet réciproquement.

pi = limx→xix>xi

FX(x) − FX(xi) et FX(x) =∑

i |xi<x

pi .

3- Couple de variables aléatoires discrètes

On peut généraliser la définition de la distribution de probabilité à un vecteur aléatoire. Par exemple,pour un couple (X,Y ), on pose si X(Ω) = xi | i ∈ I et Y (Ω) = yj | j ∈ J

pij = P(X = xi, Y = yj) .

On a alors la distribution ((xi, yj), pij) avec la condition

i∈I,j∈J

pij = 1 .

Notons (xi, pi.) la distribution de X, et (yj , p.j) celle de Y , (lois marginales). On a alors

pi. = P(X = xi) =∑

j∈J

P(X = xi, Y = yj) =∑

j∈J

pij

et

p.j = P(Y = yj) =∑

i∈I

P(X = xi, Y = yj) =∑

i∈I

pij

EH 64

(en effet l’événement (X = xi) est réunion disjointe des événements (X = xi, Y = yj) lorsque j décritJ).

Proposition Soit (X,Y ) un couple de variables discrètes et f une fonction définie sur l’ensembleX(Ω) × Y (Ω). Alors Z = f(X,Y ) est une variable discrète.

On a

Z(Ω) = f(xi, yj) | i ∈ I, j ∈ J .

C’est donc un ensemble dénombrable ou fini. Par ailleurs

(Z < z) =⋃

(i,j) | f(xi,yj)<z

(X = xi) ∩ (Y = yj)

appartient à T puisque (X = xi) et (Y = yj) se trouvent dans T et qu’une tribu est stable parintersection et réunion dénombrable.

Corollaire 1) Si X et Y sont des variables discrètes, il en est de même de : λX + µY , X.Y ,X/Y (si P(Y = 0) = 0), sup(X,Y ), inf(X,Y ) etc. . .

2) Si X est une variable discrète, et f une fonction définie sur X(Ω), alors f(X) est une variablediscrète. En particulier, Xn, |X|, λX, eX , sont discrètes, et si P(X = 0) = 0, les variables 1/X,ln |X| sort discrètes.

Les variables discrètes forment donc un ensemble stable par les différentes opérations. En particulierelles constituent un espace vectoriel et aussi un anneau.

Exercice 6 Montrer que l’ensemble des variables discrètes dont l’image X(Ω) est finie, constitue unsous-espace vectoriel et un sous-anneau de l’ensemble des variables discrètes. Est-ce encore vrai pourl’ensemble des variables discrètes telles que X(Ω) soit dénombrable ?

Exercice 7 Soit (X,Y ) un couple de variables à valeurs dans N, telles que

P(X = i, Y = j) = pij = kaibj

où a et b sont dans ] 0, 1 [ . Déterminer k et les lois marginales. Trouver la distribution (n, pn) de

Z = X + Y (distinguer les cas a = b et a 6= b), et vérifier que l’on a bien∞∑

n=0

pn = 1.

EH 65

4- Espérance

Nous allons définir l’espérance d’une variable discrète X et montrer qu’elle satisfait aux propriétésindiquées au chapitre 5.

Définition Si (xi, pi) (i ∈ I) est la distribution de probabilité d’une variable discrète X sur(Ω,T , P), on définit E(X) par

E(X) =∑

i∈I

pixi ,

si cette série converge absolument (c’est toujours le cas si I est fini). Dans le cas contraire, X n’apas d’espérance.

Remarque : si X ≥ 0, c’est-à-dire si xi ≥ 0, pour tout i, on a bien

E(X) ≥ 0 .

Cas particuliers

1) Si X = a (variable certaine), de distribution (a, 1), on a

E(X) = E(a) = a .

2) Si X = 1lA où A appartient à T , de distribution (0, P(AC), (1, P(A)), on a

E(X) = 0.P(AC) + 1.P(A) = P(A) .

Exercice 8 Soit X une variable aléatoire de distribution (i, k i) (1 ≤ i ≤ n). Déterminer k, et trouverE(X).

Proposition Soit (X,Y ) un couple de variables aléatoires discrètes de distribution ((xi, yj), pij)(i, j) ∈ I × J , et f une application définie sur X(Ω) × Y (Ω) à valeurs dans R. Soit Z = f(X,Y ).Alors

E(Z) =∑

(i,j)∈I×J

f(xi, yj) pij ,

si cette série converge absolument.

Considérons la somme doubleS =

(i,j)∈I×J

f(xi, yj) pij .

EH 66

Si cette famille converge absolument, on peut sommer dans n’importe quel ordre. On somme d’abordsur tous les couples (i, j) tels que f(xi, yj) = zk. On a donc

S =∑

k∈K

(i,j) [ f(xi,yj)=zk

f(xi, yj) pij

=∑

k∈K

(i,j) [ f(xi,yj)=zk

zk pij

=∑

k∈K

zk

(i,j) [ f(xi,yj)=zk

pij

=∑

k∈K

zk P

(i,j) [ f(xi,yj)=zk

(X = xi, Y = yj)

=∑

k∈K

zk P(Z = zk) = E(Z) .

Corollaire 1 Soit X une variable discrète et f définie sur X(Ω). Si (xi, pi) (i ∈ I) est ladistribution de X et Z = f(X) alors

E(Z) =∑

i∈I

f(xi) pi

si la série converge absolument.

Il suffit de prendre pour Y la variable certaine Y = 1.

Corollaire 2 Si X et Y ont une espérance, alors λX + µY a une espérance et

E(λX + µY ) = λE(X) + µE(Y ) .

On prend d’abord

f(x, y) = |λx + µy| .

E(|λX + µY |) =∑

i,j

|λxi + µyj| pij ≤ |λ|∑

i,j

|xi| pij + |µ|∑

i,j

|yj | pij

≤ |λ|∑

i

|xi| pi. + |µ|∑

j

|yj| p.j

≤ |λ| |E(X)| + |µ| |E(Y )| .

EH 67

On en déduit queE(|λX + µY |) < ∞ .

Donc λX + µY a une espérance. On reprend alors le calcul avec

f(x, y) = λx + µy ,

et l’on a partout des égalités, ce qui montre la linéarité de E.

Corollaire 3 Si 0 ≤ X ≤ Y , et si E(Y ) existe, alors E(X) existe et

0 ≤ E(X) ≤ E(Y ) .

On a sup(X,Y ) = Y et inf(X,Y ) = X. Par ailleurs

i,j

inf(xi, yj) pij ≤∑

i,j

sup(xi, yj) pij .

En prenant f(x, y) = sup(x, y), le membre de droite vaut E(Y ), qui est fini. Alors la somme de gaucheconverge absolument (elle est positive) et en prenant cette fois f(x, y) = inf(x, y), le membre de gauchevaut E(X). Cela prouve que E(X) existe et que

E(X) ≤ E(Y ) .

On constate donc que l’ensemble des variables discrètes telles que E(X) existe constitue un espacevectoriel, et que E est une forme linéaire sur cet espace, vérifiant les conditions demandées dans lechapitre 5.

Exercice 9 On choisit au hasard, quatre nombres a, b, c, d dans I = −1, 0, 1, et l’on forme lamatrice

A =

[

a bc d

]

.

Déterminer la distribution de probabilité, l’espérance et la variance des variables aléatoires X = det Aet Y = tr A. Trouver la probabilité pour que A soit diagonalisable.

Exercice 10 Soit a > 1. Déterminer k pour que (i, k/ai) (i ≥ 0) soit la distribution de probabilitéd’une variable X. Calculer E(X) et Var X.

EH 68

5- Variables discrètes indépendantes

Proposition Deux variables discrètes X et Y de distribution conjointe ((xi, yj), pij) (i, j) ∈ I×Jsont indépendantes, si et seulement si, pour tout couple (i, j) ∈ I × J , on a

pij = pi. p.j .

Si X et Y sont indépendantes, on a en particulier

P(X = xi, Y = yj) = P(X = xi) P(Y = yj)

c’est-à-direpij = pi. p.j .

Inversement, si l’on a pour tout couple (i, j) de I × J

pij = pi. p.j ,

on a, pour tout couple (A,B) de T0 × T0

P(X ∈ A,Y ∈ B) =∑

(i,j) |xi∈A , yj∈B

pij

=∑

(i,j) |xi∈A , yj∈B

pi. p.j

=

i | xi∈A

pi.

j |xj∈B

p.j

= P(X ∈ A) P(Y ∈ B) ,

et donc X et Y sont indépendantes.

Proposition Soit X et Y deux variables discrètes ayant une espérance. Si X et Y sont indépen-dantes, alors X.Y a une espérance et

E(X.Y ) = E(X) E(Y ) ,

soit encoreCov(X,Y ) = 0 .

On utilise une proposition du paragraphe 4. En prenant tout d’abord 1a fonction

f(x, y) = |x y| ,

EH 69

on obtient

i,j

|xi yj | pij =∑

i,j

|xi| pi.|yj| p.j

=

(

i

|xi| pi.

)

j

|yj| p.j

= E(|X|) E(|Y |) < ∞ .

La famille de gauche est donc sommable, ce qui prouve que X.Y a une espérance.

En reprenant le calcul sans les valeurs absolues, on en déduit alors que

E(X.Y ) = E(X) E(Y ) .

Remarque : on peut avoirE(X.Y ) = E(X) E(Y )

sans que X et Y soient indépendantes.

Corollaire Si X et Y sont des variables discrètes indépendantes ayant un moment d’ordre 2,alors

Var(X + Y ) = Var X + Var Y .

Exercice 11 Soit Ω = a, b, c, d, e probabilisé par

p(a) = p(b) = p(c) = p(d) = 1/6 .

1) Soit X et Y définies sur Ω par

X(a) = 1, X(b) = 2, X(c) = 3, X(d) = X(e) = 4 ,

Y (a) = 2, Y (b) = 1, Y (c) = 4, Y (d) = 3, Y (e) = 0 .

Calculer E(X), Var X , E(Y ), Var Y .Déterminer la distribution de probabilité de Z = X + Y . Calculer E(Z), Var Z. Les variables X et Ysont-elles-indépendantes ?

2) Soit X ′ et Y ′ deux variables aléatoires indépendantes sur un espace Ω′, ayant mêmes lois respecti-vement que X et Y . Déterminer la distribution de probabilité de Z ′ = X ′ + Y ′. Calculer E(Z ′), Var Z ′

et vérifier que l’on a bien

E(Z ′) = E(X ′) + E(Y ′) et Var(Z ′) = Var(X ′) + Var(Y ′) .

EH 70

Exercice 12 Soit X et Y deux variables aléatoires telles que

X(Ω) = Y (Ω) = 1, . . . , n + 1 .

On poseaij = P(X = i, Y = j) , bij = P(Y = i/X = j) ,

ainsi que

A =[

aij

]

1≤i,j≤n+1et B =

[

bij

]

1≤i,j≤n+1.

Montrer que si A est une matrice symétrique, le vecteur

V = (P(X = 1), ..., P(X = n + 1))

est vecteur propre de B. Quelles sont les valeurs propres associées ?

On prend

aij =

1/(2n) si |i + j − (n + 2)| = 10 sinon

.

Ecrire la matrice A. Déterminer les lois de X et de Y . Calculer E(X) et Var X. Les variables X et Ysont-elles indépendantes ?

Exercice 13 Soit X et Y telles que

P(X = 1, Y = 2) = P(X = 2, Y = 1) = P(X = 2, Y = 3) = P(X = 3, Y = 2)

et P(X = i, Y = j) = 0 dans tous les autres cas. Calculer E(X), E(Y ), E(X.Y ). Les variables X et Ysont-elles indépendantes ?

Exercice 14 Une urne renferme n boules numérotées de 1 à n. On choisit une boule, puis, aprèsremise, une seconde boule, et l’on appelle X le plus grand des nombres portés par les deux boules etY le plus petit. Enfin Z = X − Y .a) Montrer que si 1 ≤ j ≤ n, on a

FX(j) =(j − 1)2

n2.

En déduire la distribution de probabilité de X. Calculer E(X), Var X.b) Comparer la distribution de probabilité de Y et celle de n + 1 − X. En déduire E(Y ) et Var Y .c) Déterminer la distribution de probabilité de Z. Calculer E(Z), Var Z.

Exercice 15 Soit deux urnes U1 et U2. La première contient k1 boules noires et une blanche, etla seconde contient k2 boules noires. On effectue des tirages successifs dans les urnes de la manièresuivante :− on choisit une des deux urnes, U1 avec probabilité p, U2 avec probabilité q = 1 − p ;− on choisit alors au hasard une boule dans l’urne choisie : si la boule tirée est noire, on la replacedans la même urne, si elle est blanche, on la place dans l’autre urne.

EH 71

On note Zn la variable aléatoire donnant le numéro de l’urne contenant la boule blanche après len−ième tirage. Si n ≥ 1, on pose

xn = P(Zn = 1) , yn = P(Zn = 2) ,

et aussix0 = 1 et y0 = 0 .

Exprimer xn et yn linéairement en fonction de xn−1 et yn−1. Ecrire matriciellement les relations obte-nues, et en déduire la valeur de xn et yn. On remarquera que la matrice obtenue est de la forme

A =

[

1 − a ba 1 − b

]

.

Calculer les limites des suites (xn)n≥0 et (yn)n≥0. Calculer E(Zn) et Var Zn et les limites de ces ex-pressions quand n tend vers l’infini.

Exercice 16 Un pion ne peut prendre que deux positions 0 ou 1 sur l’axe Ox. On effectue, de manièreindépendante, des tirages successifs d’un direction : la probabilité de déplacement vers la droite étant p,et vers la gauche q = 1−p. (p ∈ ] 0, 1 [ ). Le jeu s’arrête lorsque le pion ne peut effectuer le déplacementprévu (s’il est en 1 et doit se déplacer vers la droite, ou en 0 et doit déplacer vers la gauche). Au départle pion est en 0.a) Calculer la probabilité pk pour que le jeu se termine au k−ième tirage. Montrer que le jeu s’arrêtepresque sûrement.b) Soit X la variable aléatoire donnant le numéro du tirage auquel le jeu s’arrête. Calculer E(X),Var X.c) Soit Y donnant la position en laquelle le jeu s’arrête. Calculer E(Y ), Var Y .d) Calculer E(X.Y ). Les variables X et Y sont-elles indépendantes ?

Exercice 17 Une urne contient trois boules de couleurs différentes. On effectue des tirages successifsavec remise, et on s’arrête dès que l’on a obtenu 3 boules consécutives de même couleur.Soit X donnant le nombre de tirages effectués. Si n ≥ 1, on pose

pn = P(X = n) et cn = P(X ≤ n) .

a) Que valent pn et cn pour 1 ≤ n ≤ 4 ?b) Ecrire cn en fonction des pk dans le cas général.c) Montrer que pour tout n ≥ 1,

pn+3 =2

27(1 − cn) ,

et en déduire la relation

pn+3 − pn+2 +2

27pn = 0 .

d) Déterminer explicitement pn. Que vaut la limite de pn quand n tend vers l’infini ?e) Montrer que X est une variable aléatoire et calculer E(X).

EH 73

Chapitre 7 - Variables continues

Dans ce chapitre, nous rencontrons un autre type de variable aléatoire, dont on définit l’espérance aumoyen d’une intégrale classique.

1- Rappels et compléments sur les intégrales

a) Fonction d’une variable réelle

Si f est continue sur [ a, b [ (b fini ou non), on définit l’intégrale

b∫

a

f(t) dt = limx→b

x∫

a

f(t) dt

si la limite existe. On dit dans ce cas que l’intégrale converge. On a des définitions analogues pour desintégrales sur des segments ] a, b ] et ] a, b [ .

Si la fonction f est continue sur R sauf en un nombre fini de points x1, . . . , xn, on posera

∞∫

−∞

f(t) dt =

x1∫

−∞

f(t) dt + · · · +xi+1∫

xi

f(t) dt + · · · +∞∫

xn

f(t) dt ,

si toutes les intégrales du membre de droite convergent.

En particulier, si f est nulle sur ]−∞, x1 ] , on aura

∞∫

−∞

f(t) dt =

∞∫

x1

f(t) dt .

Théorème Si l’intégrale∞∫

−∞f(x) dx converge absolument, c’est-à-dire si

∞∫

−∞|f(x)| dx converge,

alors∞∫

−∞f(x) dx converge et

∞∫

−∞

f(x) dx

≤∞∫

−∞

|f(x)| dx .

EH 74

b) Fonction de plusieurs variables

Si f est une application de R2 dans R, il n’y a plus de moyen naturel de définir l’intégrale double∫∫

f(x, y) dxdy. En particulier on peut commencer par intégrer en x puis en y, ou faire l’inverse, etl’on n’obtient pas nécessairement le même résultat.

Nous considérerons des fonctions de type suivant :

elles sont continues sur une partie A de R2 limitée par une courbe régulière (droite, cercle, etc. . . ) etnulles en dehors de A. Le théorème de Fubini donne une condition suffisante pour que l’on ait

∫ ∞

−∞

(∫ ∞

−∞f(x, y) dx

)

dy =

∫ ∞

−∞

(∫ ∞

−∞f(x, y) dy

)

dx .

Théorème de Fubini

1) Si f est positive l’égalité est vérifiée : les deux intégrales sont toutes deux infinies ou toutes deuxfinies et égales.2) Si f est de signe quelconque, on dit que l’intégrale

∫∫

f(x, y) dxdy converge absolument si l’inté-grale

∫∫

|f(x, y)| dxdy est finie. Alors, si l’intégrale converge absolument, l’égalité est encore vérifiée,et l’on posera

∫∫

A

f(x, y) dxdy =

∫∫

R2

f(x, y) dxdy =

∫ ∞

−∞

(∫ ∞

−∞f(x, y) dy

)

dx .

De plus∣

∫∫

A

f(x, y) dxdy

≤∫∫

A

|f(x, y)| dxdy .

Remarque : on constatera la ressemblance de ce théorème avec celui donné au chapitre 7 sur lesfamilles sommables.

Exercice 1 Soit f définie par f(x, y) = x−1/4y−1/2 sur A = (x, y) | 0 < x < 1 , 0 < y < 1/x et nulleailleurs. Calculer

∫∫

A

f(x, y) dxdy de deux manières.

La formule de changement de variables dans une intégrale double

Soit Φ = (g, h) une application bijective de B sur A où B et A sont des parties de R2. On suppose queg et h ont des dérivées partielles continues sur B (on dit que Φ est de classe C1).

EH 75

On note

x = g(u, v)y = h(u, v)

,

et l’on pose

dxdy

dudv=

∂g

∂u(u, v)

∂g

∂v(u, v)

∂h

∂u(u, v)

∂h

∂v(u, v)

.

Ce déterminant s’appelle le déterminant jacobien de Φ au point (u, v), et se notera égalementdetJΦ(u, v).

Alors si f est une fonction définie comme dans le théorème de Fubini, on a∫∫

A

f(x, y) dxdy =

∫∫

B

f(g(u, v), h(u, v))

dxdy

dudv

dudv

=

∫∫

B

f Φ(u, v) |det JΦ(u, v)| dudv .

Remarque : cette formule se généralise à n variables.

Exercice 2 Calculer l’intégrale de l’exercice 1 en utilisant le changement de variables x = u , y = v/u.

2- Définitions et propriétés

Définition Une variable aléatoire X sur (Ω,T , P) sera dite continue, s’il existe une fonctionfX positive, continue par morceaux sur R telle que

∞∫

−∞

fX(t) dt = 1 ,

et, pour tout x réel,

FX(x) =

x∫

−∞

fX(t) dt .

La fonction fX est appelée fonction de densité de la variable X.

Remarque 1 : la définition nécessite donc en particulier que l’intégrale∞∫

−∞fX(t) dt converge.

EH 76

Remarque 2 : la valeur de fX aux points de discontinuité n’intervient pas dans le calcul de l’intégrale ;nous conviendrons de ne pas distinguer deux fonctions qui différeraient en un nombre fini de points.

Conséquences

1) La fonction FX est continue sur R. Elle est dérivable, sauf éventuellement, aux points de discon-tinuité de fX et, en tout point où f est continue,

F ′X(x) = fX(x) .

2) On a donc en particulier pour tout x réel

P(X = x) = 0 .

3) Si a < b

P(a ≤ X < b) = FX(b) − FX(a) =

b∫

a

fX(t) dt .

Cette probabilité est aussi égale à P(a < X < b), P(a ≤ X ≤ b) ou P(a < X ≤ b) puisque P(X = a)et P(X = b) sont nulles.

Remarque 1 : de manière plus générale, pour toute partie A de T0,

P(x ∈ A) =

A

f(t) dt .

Remarque 2 : le nombre P(a ≤ X < b) représente l’aire limitée par la courbe représentative de fX ,l’axe des x et les droites x = a et x = b.

Remarque 3 : dire que fX est nulle sur une partie A revient à dire que P(X ∈ A) est nulle c’est-à-dire que X n’appartient pas à A (presque sûrement).

Remarque 4 : la fonction FX est définie par fX et réciproquement. Deux variables aléatoires conti-nues auront donc même loi, si et seulement si elles on même fonction de densité.

Exercice 3 Soit f définie sur R par

f(x) =k

1 + x2.

Déterminer k pour que f soit la densité d’une variable continue X (loi de Cauchy). Calculer P(X > 1),P(1 < X <

√3).

EH 77

Cas particulier : Variables symétriques

Définition Une variable continue est dite symétrique si sa fonction de densité est paire.

On a dans ce cas

P(X < 0) = P(X > 0) =1

2,

FX(−a) = P(X < −a) = P(X > a) = 1 − FX(a) ,

P(a < X < b) = P(−b < X < −a) ,

P(|X| < a) = 2P(0 < X < a) = 2FX(a) − 1 ,

P(|X| > a) = 2(1 − FX(a)) .

Par exemple, une variable aléatoire de Cauchy, définie dans l’exercice 3, est symétrique.

3- Changement de variables

Si X est une variable continue, et g une fonction dérivable, définie au voisinage de X(Ω), on chercheà déterminer la fonction de densité de la variable Y = g(X), en fonction de celle de X.

Par exemple, si g est une fonction croissante sur R

FY (y) = P(Y < y) = P(g(X) < y) = P(X < g−1(Y )) = FX g−1(y) .

Il ne reste plus qu’à dériver pour avoir la densité :

fY = fX g−1 (g−1)′ .

Remarque : cette formule n’est autre que celle du changement de variables dans une intégrale.

On peut traiter ce problème dans d’autres cas où g n’est pas monotone.

Exemple Si g(x) = |x|, on a alors

FY (y) = P(|X| < y) = P(−y < X < y) = FX(y) − FX(−y) ,

et en dérivant, on obtient, si y > 0,

fY (y) = fX(y) + fX(−y) .

par contre fY (y) = 0 si y ≤ 0.

En particulier si X est une variable symétrique, on a, pour y > 0,

fY (y) = 2fX(y) .

EH 78

Exercice 4 Si X est une variable continue, trouver la densité de

λX + µ (λ 6= 0) , X2 , eX ,√

X (si X > 0) .

Remarque : contrairement aux variables discrètes, l’ensemble des variables continues ne constitue pasun espace vectoriel. En effet, la fonction constante nulle est discrète et non continue ; donc si X estcontinue, −X est continue, mais X − X ne l’est pas.

4- Vecteurs aléatoires continus

On peut généraliser ce qui précède pour un couple (ou un n−uplet).

Définition Un vecteur aléatoire (X,Y ) sera continu, s’il existe une fonction f(X,Y ) de R2 dansR, positive, continue par morceaux, telle que

∫∫

R2

f(X,Y ) (u, v)dudv = 1 ,

et, quels que soient x et y dans R,

F(X,Y )(x, y) =

x∫

−∞

y∫

−∞

f(X,Y )(u, v) dv

du .

La fonction f(X,Y ) est appelée, fonction de densité du couple (X,Y ), et, pour des ensembles A deR2, assez réguliers

P((X,Y ) ∈ A) =

∫∫

A

f(X,Y )(u, v) dudv .

Aux points de continuité de f(X,Y ), on a

∂2F(X,Y )

∂x∂y= f(X,Y ) .

Remarque : si X et Y sont des variables continues, le couple (X,Y ) n’est pas nécessairement continu.

Exercice 5 Soit X une variable continue. Trouver la fonction de répartition du couple (X,−X). Cevecteur aléatoire est-il continu ?

EH 79

Proposition La densité des éléments du couple (lois marginales) est obtenue par les formules

fX(x) =

∞∫

−∞

f(X,Y )(x, y) dy et fY (y) =

∞∫

−∞

f(X,Y )(x, y) dx .

On a

FX(x) = limy→∞

F(X,Y )(x, y) = limy→∞

y∫

−∞

x∫

−∞

f(X,Y )(u, v) du

dv

=

∞∫

−∞

x∫

−∞

f(X,Y )(u, v) du

dv ,

et grâce au théorème de Fubini, ceci est encore égal à

x∫

−∞

∞∫

−∞

f(X,Y )(u, v) dv

du ,

d’où

fX(x) =

∞∫

−∞

f(X,Y )(x, y) dy .

On a alors une caractérisation simple des variables continues indépendantes.

Proposition Deux variables continues X et Y sont indépendantes si et seulement si

f(X,Y )(x, y) = fX(x) fY (y) .

En effet, en intégrant on passe à

F(X,Y )(x, y) = FX(x)FY (y) ,

et inversement, en dérivant cette dernière relation, on retrouve la première.

Exercice 6 Soit (X,Y ) un couple de densité

f(X,Y )(xy) =

k(x + y) sur le carré ] 0, 1 [× ] 0, 1 [0 sinon

.

EH 80

Déterminer k, les densités des variables X et Y . Ces variables sont-elles indépendantes ?

Exercice 7 Soit X et Y deux variables continues indépendantes. Montrer que

P(X > Y ) =

∞∫

−∞

fX(x)FY (x) dx .

Que vaut cette probabilité si X et Y ont la même loi ?

5- Espérance d’une variable continue

Définition Si X est une variable continue de densité fX , on définit l’espérance de X par

E(X) =

∞∫

−∞

xfX(x) dx

si cette intégrale converge absolument. Dans le cas contraire X n’a pas d’espérance.

Remarque : si X est symétrique et si E(X) existe, alors E(X) = 0 (intégrale d’une fonction impaire).

Nous admettrons les résultats suivants.

Proposition 1) Si X est une variable continue, et g une fonction continue définie sur X(Ω), alors

E(g(X)) =

∞∫

−∞

g(x)fX(x) dx ,

si cette intégrale converge absolument.2) Si (X,Y ) est un vecteur aléatoire continu, et g une fonction continue définie sur (X,Y )(Ω), alors

E(g(X,Y )) =

∫∫

R2

g(x, y)f(X,Y )(x, y) dxdy ,

si cette intégrale converge absolument.

Remarque : Si g est croissante et de classe C1, le résultat 1) peut se démontrer simplement par lechangement de variable g(x) = y).

EH 81

Corollaire L’espérance ainsi définie, satisfait aux conditions du chapitre 5.

1) Si X ≥ 0, alors P(X < x) est nul si x < 0, donc fX(x) est nul également. Alors

E(X) =

∞∫

0

xfX(x) dx ≥ 0

(intégrale d’une fonction positive).

2) On a

E(λX + µY ) =

∫∫

R2

(λx + µy)f(X,Y )(x, y) dxdy

= λ

∞∫

−∞

x

∞∫

−∞

f(X,Y )(x, y) dy

dx + µ

∞∫

−∞

y

∞∫

−∞

f(X,Y )(x, y) dx

dy .

En utilisant les fonctions de densité des lois marginales, on a alors

E(λX + µY ) = λ

∞∫

−∞

xfX(x) dx + µ

∞∫

−∞

yfY (y) dy = λE(X) + µE(Y ) .

Remarque : pour montrer la convergence absolue de l’intégrale, il suffit de refaire le calcul précédentavec |λ| |X| + |µ| |Y |.

3) Si 0 ≤ X ≤ Y , on a

∫∫

R2

inf(x, y)f(X,Y )(x, y) dxdy ≤∫∫

R2

sup(x, y)f(X,Y )(x, y) dxdy

≤ E(sup(X,Y )) = E(Y ) .

Donc l’intégrale de gauche converge et cette intégrale n’est autre que E(inf(X,Y )) = E(X).

Exercice 8 Déterminer k pour que la fonction f définie

f(x) =

k

1 + xsi x ∈ ] 0, 1 [

0 sinon

,

soit la densité de probabilité d’une variable aléatoire X. Représenter FX . Calculer E(X), Var X.

EH 82

Exercice 9 Soit f définie sur R par

f(x) =k

x4 + 1.

a) Monter, sans le calculer, qu’il existe k, tel que f soit la densité d’une variable aléatoire X.b) Montrer que cette variable est centrée réduite.c) Calculer de deux manières E(X2 +

√2X + 1). En déduire k.

Exercice 10 Déterminer k pour que la fonction f définie par

f(x) =

k ln x si x ∈ ] 0, 1 [0 sinon

,

soit la densité de probabilité d’une variable X. Calculer E(X), Var X, le moment d’ordre n de X.Déterminer la densité de probabilité de Y = ln X.

Exercice 11 Redémontrer, dans le cadre des variables continues, l’inégalité de Markov.

6- Loi d’une somme de variables indépendantes

Soit (X,Y ) un vecteur continu de densité f(X,Y ), et soit Φ une application bijective de classe C1. Onconsidère le couple

(U, V ) = Φ−1(X,Y ) .

Alors (U, V ) est un vecteur aléatoire continu, de densité

f(U,V )(u, v) = f(X,Y ) Φ(u, v)|det JΦ(u, v)| .

Cette formule provient de la formule de changement de variables dans une intégrale double. En effet

P((U, V ) ∈ B) = P((X,Y ) ∈ Φ(B)) =

∫∫

Φ(B)

f(X,Y )(x, y) dxdy ,

et, en posant(u, v) = Φ−1(x, y) ,

on obtient

P((U, V ) ∈ B) =

∫∫

B

f(X,Y ) Φ(u, v) |det JΦ(u, v)| dudv =

∫∫

B

f(U,V )(u, v) dudv .

De ce résultat on déduit le théorème suivant

Théorème Si X et Y sont des variables continues indépendantes, alors U = X + Y est continueet

fU (u) =

∞∫

−∞

fX(u − v)fY (v) dv .

EH 83

Cette fonction s’appelle produit de convolution de fX et fY et se note fX ⋆ fY .

On pose

u = x + yv = y

soit

x = u − vy = v

.

On a donc la bijection Φ(u, v) = (u − v, v) de jacobien

det JΦ(u, v) =

1 −10 1

= 1 .

La densité du couple (X,Y ) estf(X,Y )(x, y) = fX(x) fY (y) ,

puisque X et Y sont indépendantes. Alors celle de (U, V ) est

f(U,V )(u, v) = f(X,Y ) Φ(u, v) |det FΦ(u, v)|= fX(u − v) fY (v) .

On obtient la densité de U comme loi marginale du couple (U, V ) :

fU(u) =

∞∫

−∞

f(U,V )(u, v) dv =

∞∫

−∞

fX(u − v) fY (v) dv .

Remarque : cette technique peut être utilisée pour trouver la densité d’un produit ou d’un quotient.

Exercice 12 Déterminer k pour que la fonction f définie par

f(x) =

0 si x ∈ ]−∞, 0 [ke−ax si x ∈ ] 0, ∞ [

,

soit la densité de probabilité d’une variable aléatoire X (loi exponentielle de paramètre a). Si X suitune telle loi de paramètre a, et Y une loi de paramètre b, déterminer la densité de X + Y lorsque Xet Y sont indépendantes.

Exercice 13 Déterminer la densité d’un quotient X/Y de deux variables continues indépendantes.Appliquer ce résultat aux variables de l’ex.12

Exercice 14 Soit Z une variable continue positive. On pose X = [Z] (partie entière de Z), etY = Z − X. Déterminer la fonction de répartition de Y et la distribution de X. Montrer que X et Ysont indépendantes.

EH 85

Chapitre 8 - Lois de probabilité classiques

I - Lois discrètes

1- Loi uniforme ou équirépartie sur x1, . . . , xpC’est la loi du tirage au hasard dans x1, . . . , xp. On a

X(Ω) = x1, . . . , xp et pi = P(X = xi) =1

p.

Alors

E(X) =

p∑

i=1

pixi =1

p

p∑

i=1

xi ;

L’espérance est la moyenne arithmétique des nombres xi. On a aussi

Var X =1

p

p∑

i=1

x2i −

1

p2

(

p∑

i=1

xi

)2

.

Exercice 1 Déterminer la moyenne et la variance d’une loi X uniforme sur 1, 2, . . . , n.

2- Loi de Bernoulli de paramètre p

C’est la loi d’une variable X qui ne peut prendre que les valeurs 0 et 1.

X(Ω) = 0, 1 et P(X = 1) = p , P(X = 0) = q = 1 − p .

On a immédiatement

E(X) = p et Var X = p − p2 = p q .

Exercice 2 Soit X une variable aléatoire ne prenant que les valeurs a et b, et telle que P(X = a) = p.Montrer qu’il existe une variable de Bernoulli Y de paramètre p et deux nombres λ et µ tels queX = λY + µ.

Exercice 3 Soit X une variable aléatoire à valeurs dans N, et Y une variable de Bernoulli de paramètrep. On suppose X et Y indépendantes.Exprimer la distribution de probabilité de la variable Z = X + Y en fonction de celle de X. Quelle loisuit T = X.Y ?

EH 86

Exercice 4 Soit (X,Y ) un couple de variables de Bernoulli. On pose

A =

[

P(X = 0, Y = 0) P(X = 0, Y = 1)P(X = 1, Y = 0) P(X = 1, Y = 1)

]

.

Montrer que E(X.Y ) = E(X) E(Y ) si et seulement si det A est nul.

3- Loi binomiale de paramètres n et p : B(n, p)

On a

X(Ω) = 0, 1, . . . , n et P (X = i) =

(

n

i

)

piqn−i (où q = 1 − p) .

(La somme des nombres P(X = i) n’est autre que le développement de (p + q)n et vaut 1).

Théorème Toute variable X suivant une loi B(n, p) peut être considérée comme la somme de nvariables de Bernoulli indépendantes de paramètre p.

Soit X1, . . . ,Xn des variables de Bernoulli de paramètre p indépendantes et X leur somme. L’événement(X = i), c’est-à-dire (X1 + · · · + Xn = i) s’écrit comme la réunion disjointe des événements

(Xα1= · · · = Xαi

= 1,Xαi+1= · · · = Xαn = 0)

pour tout choix possible de (α1, . . . , αi) dans 1, . . . , n. Comme les variables sont indépendantes, laprobabilité de l’événement ci-dessus est

P(Xα1= 1) · · · P(Xαi

= 1) P(Xαi+1= 0) · · · P(Xαn = 0) = piqn−i .

Cette probabilité ne dépend pas du choix de α1, . . . , αn, et comme il y a(n

i

)

façons de faire ce choix,on a finalement

P(X = i) =

(

n

i

)

piqn−i .

Ce qui montre que X suit une loi binomiale B(n, p).

Si X suit une loi B(n, p)

E(X) = n p et Var X = n p q .

En effet, à cause de la linéarité de E,

E(X) = E(X1) + · · · + E(Xn) = n p ,

et puisque les variables sont indépendantes

Var X = Var X1 + · · · + Var Xn = n p q .

EH 87

D’autre part si X suit une loi B(n, p) et Y une loi B(m, p) et sont indépendantes, alors X + Y suitune loi B(n + m, p).

Remarque : il est important de savoir reconnaître quand on a une loi binomiale de paramètres n etp. Une telle loi apparaît lorsque l’on effectue n fois une épreuve ne présentant que deux possibilités :− réussite avec probabilité p− échec avec probabilité q = 1 − p.

Si les différentes épreuves sont indépendantes les unes des autres, la variable X qui donne le nombrede réussites, suit une loi B(n, p).

Exercice 5 On lance une pièce de monnaie. Quelle est la probabilité− d’obtenir 3 faces en 7 épreuves− d’obtenir au moins 4 faces en 6 épreuves− d’obtenir au plus 3 faces en 6 épreuves.

Exercice 6 Huit composants électroniques identiques sont mis en service simultanément. La proba-bilité pour que l’un quelconque d’entre eux soit encore en fonctionnement au bout d’un an est 0, 7.Soit X la variable aléatoire donnant le nombre de composants en fonctionnement au bout d’un an.Quelle loi suit X ?Sachant qu’il y a au moins 4 composants en fonctionnement, quelle est la probabilité pour qu’il y enait au plus 6 ?

Exercice 7 On considère un groupe de n personnes. Soit X la variable aléatoire donnant le nombrede jours de l’année (de 365 jours) ou exactement k personnes sont nées. Trouver E(X).

Exercice 8 Soit X et Y indépendantes suivant respectivement les lois B(n, p) et B(m, p). En calculantP(X + Y = i) lorsque 0 ≤ i ≤ min(n, p) montrer que

(

n + m

i

)

=

i∑

j=0

(

n

j

)(

m

i − j

)

.

Exercice 9 Soit X et Y deux variables aléatoires à valeurs dans 1, . . . , n + 1. On pose

P(X = i, Y = j) = pij = k

(

n

j − 1

)(

n

i − 1

)

.

a) Déterminer k.b) Trouver la distribution de probabilité de X et Y . Quelle loi suit X − 1 ?c) Calculer E(X), Var X.d) Etudier l’indépendance de X et Y .

EH 88

4- Loi de Poisson de paramètre p : P(p)

Cette loi repose sur le fait que, pour tout x réel

∞∑

k=0

xn

n!= ex .

On a

X(Ω) = N et P(X = i) = e−p pi

i!.

Alors

E(X) =

∞∑

i=1

e−p pi

(i − 1)!= p ,

puis

E(X2) = e−p∞∑

i=0

i2pi

i!= e−p

∞∑

i=0

(i(i − 1) + i)pi

i!

= e−p∞∑

i=2

pi

(i − 2)!+ e−p

∞∑

i=1

pi

(i − 1)!

= p2 + p .

On en déduit donc

E(X) = Var X = p .

Proposition Si X suit une loi P(p) et Y une loi P(q), et si X et Y sont indépendantes, alorsX + Y suit une loi P(p + q).

Si Z = X + Y , on a

P(Z = i) = P(X + Y = i) = P

(

i⋃

k=0

(X = k, Y = i − k)

)

=i∑

k=0

P(X = k)P(Y = i − k) .

En remplaçant P(X = k) et P(Y = i − k) par leur valeur, on obtient

P(Z = i) =

i∑

k=0

e−pe−q pkqi−k

k!(i − k)!,

ce qui s’écrit encore

P(Z = i) =e−(p+q)

i!

(

i∑

k=0

i!

k!(i − k)!pkqi−k

)

,

EH 89

et la parenthèse n’est autre que le développement de (p + q)i, d’où

P(Z = i) = e−(p+q) (p + q)i

i!.

Proposition Soit Xn une variable aléatoire suivant une loi binomiale de paramètres (n, p/n) oùn > p. Alors, lorsque n tend vers l’infini, P(Xn = i) tend vers e−ppi/i! .

On a

P(Xn = i) =

(

n

i

)

( p

n

)i (

1 − p

n

)n−i

=n(n − 1)..(n − i + 1)

ni

(

1 − p

n

)n−i pi

i!.

La fraction en n tend vers 1, et(

1 − p

n

)n−itend vers e−p d’où le résultat.

Cette proposition veut dire, qu’en pratique, on peut remplacer une loi binomiale B(n, p) par une loide Poisson P(n p) lorsque n est grand et p petit (p < 0, 1, n > 20 par exemple).

Exercice 10 Soit X et Y deux variables aléatoires indépendantes suivant respectivement des loisP(n) et P(m). Si 0 ≤ i ≤ k, calculer

pi = P(X = i/X + Y = k)

et montrer que (i, pi)0≤i≤k est la distribution de probabilité d’une variable aléatoire T . Quelle loi suitla variable T ?

II - Lois continues

1- Loi uniforme sur un intervalle ] a, b [

Il n’est pas possible de choisir au hasard un point d’un intervalle donné : puisqu’il y a une infinité depoints dans un intervalle, cette probabilité ne peut être que nulle. Si l’on veut donner un sens à untel tirage au hasard, on ne peut que choisir un intervalle inclus dans ] a, b [ en demandant que deuxintervalles de même longueur ait la même chance d’être tirés, et que la probabilité soit proportionnelleà la longueur de l’intervalle. En d’autres termes

P(x < X < x + h) = k h ,

avec bien sûrP(a < X < b) = 1 = k(b − a) .

EH 90

Cela détermine la constante k. On trouve

k =1

b − a.

Une variable X sera dite uniforme sur ] a, b [ si sa fonction de répartition possède le graphe suivant :

-

6

1

ba

c’est-à-dire si sa densité est constante sur ] a, b [ et vaut 1/(b − a) sur cet intervalle, et est nulle endehors de ] a, b [ . Alors

E(X) =

b∫

a

x

b − adx =

b + a

2

c’est le milieu de l’intervalle ] a, b [ .

On obtient également facilement

Var X =(b − a)2

12.

Exercice 11 Déterminer la densité de probabilité de la somme de deux variables indépendantesuniformes sur ] 0, 1 [ .

2- Loi normale (ou de Laplace-Gauss) de paramètres m et σ : N (m, σ)

La densité de probabilité est

f(x) =1

σ√

2πexp

(

−1

2

(

x − m

σ

)2)

,

dont la courbe représentative est la courbe en cloche bien connue, symétrique par rapport à la droited’équation x = m. Le coefficient σ traduit le resserrement de la « bosse ». En effet, si l’on calcule la

EH 91

dérivée seconde, on obtient des points d’inflexion pour x = m ± σ. La distance entre les deux pointsd’inflexion est donc de 2σ.

m − σ m m + σ

Le fait que l’on obtienne ainsi une densité de probabilité se démontre en faisant le changement devariable

u =1√2

x − m

σ.

On obtient alors∞∫

−∞

f(x) dx =1√π

∞∫

−∞

e−u2

du = 1 ,

en utilisant le résultat classique∞∫

−∞

e−u2

du =√

π .

Proposition Si X suit une loi N (m,σ), et si a et b sont des constantes, où a est non nulle,alors aX + b suit une loi N (am + b, |a|σ).

Posons Y = aX + b. On aFY (y) = P(Y < y) = P(aX + b < y) .

Si a > 0,FY (y) = P(X < (y − b)/a) = FX((y − b)/a) ,

et donc, en dérivant

fY (y) =1

afY ((y − b)/a) =

1

aσ√

2πexp

(

−1

2

(

y − b − am

)2)

.

EH 92

Donc Y suit une loi N (am + b, σa).

Si a < 0

FY (y) = P(X > (y − b)/a) = 1 − FX((y − b)/a) ,

et en dérivant on obtient le même résultat que ci-dessus mais avec un signe opposé. Donc Y suit uneloi N (am + b,−σa).

En particulier (X − m)/σ suit une loi N (0, 1).

On aE(Y ) = m et Var X = σ2 .

Démontrons le pour une loi N (0, 1). Si Y suit une telle loi

E(Y ) =1√2π

∞∫

−∞

ye−y2/2 dy .

Cette intégrale converge absolument, et est nulle à cause de la parité. On a aussi

E(Y 2) =1√2π

∞∫

−∞

y2e−y2/2 dy ,

et en intégrant par parties

E(Y 2) =1√2π

∞∫

−∞

e−y2/2 dy = 1 .

Alors, si X suit une loi N (m,σ), on a X = σY + m, où Y suit une loi N (0, 1) et donc

E(X) = m et Var X = σ2 Var Y = σ2 .

Remarque : les tables numériques donnent la fonction de répartition de la loi N (0, 1) pour des valeurspositives ; pour une loi normale quelconque, on se ramènera toujours à cette loi normale N (0, 1) quiest une variable symétrique.

Par exemple, si X suit une loi N (2; 0, 1),

P(X < 1) = P

(

X − 2

0, 1< −10

)

= 1 − P

(

X − 2

0, 1< 10

)

.

Rappelons que, pour toutes les variables symétriques, on a

FX(−a) = P(X < −a) = 1 − P(X < a) = 1 − FX(a) .

EH 93

Exercice 12 Soit F la fonction de répartition d’une variable X suivant une loi normale centréeréduite. Soit a, b, c trois réels, tels que

F (a) =1

3, F (b) =

3

4, F (c) =

5

8.

Classer en ordre croissant les nombres a, b, c, 0.Calculer

P(−a < X < b) , P(|X| > c) , P(|X| > c/(X > a)) .

Exercice 13 : Soit X suivant une loi normale centrée réduite. Trouver la densité de Y = X2.Calculer P(Y < 1), P(|Y − 1| < 0, 5). (On donne les valeurs approchées suivantes :

FX(1) = 0, 841 , FX(1, 22) = 0, 888 , FX(0, 70) = 0, 758) .

Calculer E(Y ), Var Y .Trouver a tel que P(Y > a) = 0, 05. (On donne F−1

X (0, 975) = 1, 96).

Exercice 14 Déterminer le quotient X/Y de deux lois normales indépendantes centrées réduites.

Proposition Si X et Y sont des variables aléatoires indépendantes suivant respectivement deslois N (m,σ) et N (m′, σ′), alors X + Y suit une loi N (m + m,

√σ2 + σ′2).

Pour montrer cela, calculons tout d’abord l’intégrale∞∫

−∞e−(at2+bt+c) dt si a > 0.

En mettant le trinôme sous forme réduite et en effectuant le changement de variable

u =√

a(t + b/2a) ,

on trouve

1√a

expb2 − 4ac

4a

∞∫

−∞

e−u2

du =

π

aexp

b2 − 4ac

4a.

Soit alors X et Y indépendantes suivant respectivement des lois N (0, σ) et N (0, σ′), et Z = X + Y .On a donc

fZ(z) = fX ⋆ fY (z) =1

2πσσ′

∞∫

−∞

exp

(

−1

2

[

(

z − t

σ

)2

+t2

σ′2

])

dt .

En développant, on applique alors le calcul ci-dessus avec

a =1

2

σ2 + σ′2

σ2 σ′2, b = − z

σ2, c =

z2

2σ2

et on obtientb2 − 4ac

4a= − z2

2(σ2 + σ′2).

EH 94

Donc Z suit une loi N (0,√

σ2 + σ′2).

Si maintenant X et Y sont indépendantes et suivent des lois N (m,σ) et N (m′, σ′) respectivement,X −m et X ′ −m′ sont indépendantes et suivent des lois N (0, σ) et N (0, σ′), donc X + X ′ −m−m′

suit une loi N (0,√

σ2 + σ′2) et finalement X + Y suit une loi une loi N (m + m′,√

σ2 + σ′2).

Pour terminer, indiquons que la loi normale s’obtient comme limite de nombreuses autres lois. Celarésulte du théorème de la limite centrale.

Théorème Soit (Xn)n≥0 une suite de variables aléatoires indépendantes et de même loi, demoyenne m, et d’écart type σ. Alors X1 + · · · + Xn est une variable aléatoire de moyenne n m etd’écart-type σ

√n et la variable aléatoire

Zn =X1 + · · · + Xn − n m

σ√

n

est centrée réduite.Alors, si Fn désigne la fonction de répartition de Zn, la suite (Fn(x))n≥1, converge pour tout x réelvers la fonction de répartition FX(x) d’une variable X suivant une loi N (0, 1). (On dit que Zn

converge en loi vers X).

Applications

Prenons pour Xi des variables de Bernoulli de même paramètre p. Alors X1 + · · · + Xn suit une loiB(n, p), qui peut, pour n assez grand (n p et n q ≥ 10) être remplacée par une loi N (n p,

√n p q).

Prenons pour Xi des variables de Poisson de paramètre a, alors X1 + · · · + Xn suit une loi de Poissonde paramètre n a, et peut, pour n assez grand, être remplacée par une loi N (n a,

√n a).

Le théorème de la limite centrale signifie de manière générale que l’on peut remplacer, pour n assezgrand, la loi de X1 + · · · + Xn par une loi N (n m, σ

√n).

Exercice 15 Sur une autoroute, la proportion des camions sur l’ensemble des véhicules est de 7%.

a) A quelle variable aléatoire peut-on assimiler le nombre de camions parmi 100 véhicules pris au ha-sard ? Quelle est la probabilité pour que ce nombre soit au moins égal à 5 ?

b) A quelle variable aléatoire peut-on assimiler le nombre de camions parmi 1000 véhicules pris auhasard ? Déterminer la probabilité pour qu’il y ait entre 65 et 75 camions.

c) On choisit n véhicules au hasard. Déterminer, en utilisant l’inégalité de Bienaymé- Tchebychev, pourquelles valeurs de n, on peut affirmer que la proportion des camions sera comprise entre 6% et 8% avec

EH 95

un risque d’erreur inférieur à 0, 05.Reprendre le calcul en admettant que la fréquence des camions suit une loi normale. Si F est la fonctionde répartition d’une loi N (0, 1), on a

F (0, 62) = 0, 732 , F−1(0, 975) = 1, 96 .

Remarque : il existe des variables aléatoires qui ne sont ni discrètes, ni continues (la somme d’unevariable discrète et d’une variable continue n’est en général ni l’un ni l’autre). Donnons un exemple detelle variable arrivant « naturellement ».

Temps d’attente à un feu rouge.

On suppose que le feu ne puisse être que vert ou rouge, la durée d’un cycle étant l’unité de temps, lefeu est vert pendant p unités et rouge pendant q = 1 − p unités.

Cherchons la fonction de répartition. On a évidement

FX(x) =

0 si x ≤ 01 si x > q

.

Si 0 < x ≤ q, on a

FX(x) = P(X < x) = P(X = 0) + P(0 < X < x) = p + x .

La fonction de répartition n’est ni continue, ni en escalier.

-

6

1

q

p

EH 97

Résumé sur les variables aléatoires

EH 98

DISCRETES CONTINUES

X(Ω) = x1, . . . , , xn, . . . X(Ω) est un intervalle de R

Distribution de probabilité Densité de probabilité

pi = P(X = xi) fX application de R dans R, positivesuite de réels positifs tels que continue par morceaux telle que

i

pi = 1∞∫

−∞fX(x) dx = 1

Si A est un intervalle de R

P(X ∈ A) =∑

i | xi∈A

pi P(X ∈ A) =∫

A

fX(t) dt

Si A = ]−∞, x [ , on obtient la fonction de répartition : FX(x) = P(X < x)

FX(x) =∑

i |xi<x

pi FX(x) =x∫

−∞fX(t) dt

Fonction de répartition en escalier Fonction de répartition continuepi = FX(xi+1) − FX(xi) fX(x) = F ′

X(x)

P(a ≤ X < b) = FX(b) − FX(a)

Couple (X,Y ) de variables aléatoires

X(Ω) = x1, . . . , , xn, . . . Y (Ω) = y1, . . . , , yn, . . . X(Ω) et Y (Ω) sont des intervalles de R

Distribution de probabilité Densité de probabilité

pij = P(X = xi, Y = yj) f(X,Y ) application de R2 dans R, positive

suite double de réels positifs tels que continue par morceaux telle que∑

i,j

pij = 1∞∫

−∞

∞∫

−∞f(X,Y )(x, y) dxdy = 1

Fonction de répartition du couple : F(X,Y )(x, y) = P(X < x, Y < y)

F(X,Y )(x, y) =∑

(i,j) | xi<x,yj<y

pij F(X,Y )(x, y) =x∫

−∞

y∫

−∞f(X,Y )(u, v) dudv

Lois marginales d’un couple

Distribution de X Densité de X

pi. = P(X = xi) =∑

j

pij fX(x) =∞∫

−∞f(X,Y )(x, y) dy

Distribution de Y Densité de Y

p.j = P(Y = yj) =∑

i

pij fY (y) =∞∫

−∞f(X,Y )(x, y) dx

EH 99

DISCRETES CONTINUES

Espérance

E(X) =∑

i

xi pi E(X) =∞∫

−∞xfX(x) dx

si la série converge absolument si l’intégrale converge absolument

Si g est définie sur X(Ω) Si g est continue sur X(Ω)

E(g(X)) =∑

i

g(xi) pi E(g(X)) =∞∫

−∞g(x)fX(x) dx

Pour deux variables

E(g(X,Y )) =∑

i,j

g(xi, yj) pij E(g(X,Y )) =∞∫

−∞

∞∫

−∞g(x, y)f(X,Y )(x, y) dxdy

Moment d’ordre n

E(Xn) =∑

i

xni pi E(Xn) =

∞∫

−∞xnfX(x) dx

Variance

Var X = E(X2) − (E(X))2 = E((X − E(X))2)

Ecart-type

σX =√

Var X

Propriétés générales

E(aX + bY ) = a E(X) + b E(Y )

E(aX + b) = a E(X) + b

Var(aX + b) = a2 Var X

Y =X − E(X)

σXest centrée réduite

EH 100

DISCRETES CONTINUES

Variables aléatoires indépendantes

P(X ∈ A,Y ∈ B) = P(X ∈ A) P(Y ∈ B)

quels que soient les intervalles A et B

cela équivaut à

F(X,Y )(x, y) = FX(x)FY (y)

cela équivaut aussi àpij = pi. p.j f(X,Y )(x, y) = fX(x) fY (y)

quel que soit (i, j) quel que soit (x, y)

alors

E(X.Y ) = E(X) E(Y )

Var(X + Y ) = Var X + Var Y

EI 1

EI - EXERCICES DE PROBABILITES

CORRIGES

Notations

1) Les coefficients du binôme sont notés(n

p

).

2) Un arrangement de n objets pris p à p est noté Apn.

3) Si A est un ensemble fini, on notera |A| ou card A le nombre d’éléments de A.4) Si a et b sont des entiers tels que a ≤ b, on désigne par [[ a, b ]] l’ensemble [ a, b ] ∩ Z.5) Le symbole ⊥⊥ indique l’indépendance d’événements ou de variables aléatoires.6) On désigne par AC le complémentaire de A ou l’événement contraire de A.7) On note E et V respectivement, l’espérance et la variance d’une variable aléatoire.

0 - Dénombrement

1 Etablir les identitésa) Ak

n1An−k

n2

(nk

)= n!

(n1k

)(n2

n−k

)

b)(n

p

)( pm

)=(nm

)(n−mn−p

)

c)

(n1

k

)( n2

n−k

)

(n1+n2

n

) =

(nk

)(n1+n2−n

n1−k

)

(n1+n2

n1

)

Solution

a) On a

Akn1

An−kn2

(n

k

)

=n1!

(n1 − k)!

n2!

(n2 − n + k)!

n!

k! (n − k)!

= n!n1!

k! (n1 − k)!

n2!

(n − k)! (n2 − n + k)!

= n!

(n1

k

)(n2

n − k

)

.

b) On a(

n

p

)(p

m

)

=n!

p! (n − p)!

p!

m! (p − m)!=

n!

(n − p)!m! (p − m)!,

et (n

m

)(n − m

n − p

)

=n!

m! (n − m)!

(n − m)!

(n − p)! (p − m)!=

n!

(n − p)!m! (p − m)!,

EI 2

d’où l’égalité.

c) En calculant les deux membres on obtient le même résultat

n! (n1 + n2 − n)!n1!n2!

k! (n − k)! (n1 − k)!(n2 + k − n)! (n1 + n2)!.

2 Etablir l’identité (si 1 ≤ k ≤ n),

d)(nk

)=

n−1∑

j=k−1

(j

k − 1

)

.

Solution

On démontre par récurrence, que, pour n ≥ 1, on a la propriété suivante :

(Pn) : pour tout k compris entre 1 et n, on a(nk

)=

n−1∑

j=k−1

(j

k − 1

)

.

Initialisation : On prend n = 1, alors nécessairement k = 1, et on a

(1

1

)

=

0∑

j=0

(j

k − 1

)

=

(0

0

)

= 1 ,

et la propriété est vraie au rang 1.

Hérédité : Supposons la propriété vraie au rang n. Soit alors k tel que 1 ≤ k ≤ n + 1.

Il y a trois situations possibles.

Si k = n + 1 on a

1 =

(n + 1

n + 1

)

=

(n

n

)

=n∑

j=n

(j

n

)

.

Si 2 ≤ k ≤ n on peut écrire(

n + 1

k

)

=

(n

k

)

+

(n

k − 1

)

.

On a donc(

n

k

)

=

n−1∑

j=k−1

(j

k − 1

)

,

mais aussi 1 ≤ k − 1 ≤ n, donc(

n

k − 1

)

=n−1∑

j=k−2

(j

k − 2

)

,

EI 3

Alors

(n + 1

k

)

=

n−1∑

j=k−1

(j

k − 1

)

+

n−1∑

j=k−2

(j

k − 2

)

=

n−1∑

j=k−1

(

(j

k − 1

)

+

(j

k − 2

)

) +

(k − 2

k − 2

)

=

n−1∑

j=k−1

(j + 1

k − 1

)

+

(k − 2

k − 2

)

.

Par changement d’indice de sommation et puisque(k−2k−2

)=(k−1k−1

)= 1, on obtient

(n + 1

k

)

=

n∑

j=k

(j

k − 1

)

+

(k − 1

k − 1

)

=

n∑

j=k−1

(j

k − 1

)

.

Si k = 1(

n + 1

1

)

= n + 1 =

n∑

j=0

(j

0

)

.

On a donc démontré que, si 1 ≤ k ≤ n + 1

(n + 1

k

)

=

n∑

j=k−1

(j

k − 1

)

,

ce qui est la propriété au rang n + 1.La propriété Pn est donc vraie pour tout entier n ≥ 1.

3 Soient n1 et n2 deux nombres (réels ou complexes). Etablir les identités

e)n∑

k=0

k

(n

k

)

nk1n

n−k2 = nn1(n1 + n2)

n−1

f)

n∑

k=0

k(k − 1)

(n

k

)

nk1n

n−k2 = n(n − 1)n2

1(n1 + n2)n−2

Solution

Posons

f(x) = (x + n2)n =

n∑

k=0

(n

k

)

xknn−k2 .

En dérivant

f ′(x) = n(x + n2)n−1 =

n∑

k=1

k

(n

k

)

xk−1nn−k2 ,

EI 4

puis en dérivant une seconde fois

f ′′(x) = n(n − 1)(x + n2)n−2 =

n∑

k=2

k(k − 1)

(n

k

)

xk−2nn−k2 .

e) En particulier

f ′(n1) = n(n1 + n2)n−1 =

n∑

k=1

k

(n

k

)

nk−11 nn−k

2 ,

et en multipliant par n1

nn1(n1 + n2)n−1 =

n∑

k=0

k

(n

k

)

nk1n

n−k2 .

(La somme peut commencer à 0, puisque le premier terme est nul).

f) De même

f ′′(n1) = n(n − 1)(n1 + n2)n−2 =

n∑

k=2

k(k − 1)

(n

k

)

nk−21 nn−k

2 ,

et en multipliant par n21

n(n − 1)n21(n1 + n2)

n−2 =

n∑

k=0

k(k − 1)

(n

k

)

nk1n

n−k2 .

(La somme peut commencer à 0, puisque les deux premiers termes sont nuls).

4 On note, pour j ∈ N et m ∈ N∗, Tj(m) =

m∑

k=1

kj .

1) Montrer que

m∑

k=1

[(k + 1)n+1 − kn+1] =

n∑

j=0

(n + 1

j

)

Tj(m).

2) En déduire une formule de récurrence sur n permettant de calculer Tn(m).

3) Montrer que, pour tout n ∈ N∗, Tn(m) est un polynôme de degré n + 1 en m qui s’annule en 0 et

−1.

Solution

1) En utilisant la formule du binôme, on a

(k + 1)n+1 =

n+1∑

j=0

(n + 1

j

)

kj ,

donc

(k + 1)n+1 − kn+1 =

n∑

j=0

(n + 1

j

)

kj ,

EI 5

etm∑

k=1

[(k + 1)n+1 − kn+1] =m∑

k=1

n∑

j=0

(n + 1

j

)

kj

.

On peut intervertir les sommations et l’on obtient

m∑

k=1

[(k + 1)n+1 − kn+1] =n∑

j=0

(m∑

k=1

(n + 1

j

)

kj

)

=

n∑

j=0

(n + 1

j

)( m∑

k=1

kj

)

,

ce qui donne bienm∑

k=1

[(k + 1)n+1 − kn+1] =

n∑

j=0

(n + 1

j

)

Tj(m) .

2) La sommem∑

k=1

[(k + 1)n+1 − kn+1] étant télescopique, on obtient

m∑

k=1

[(k + 1)n+1 − kn+1] = (m + 1)n+1 − 1 ,

et on déduit alors de la question 1)

(m + 1)n+1 − 1 =

(n + 1

n

)

Tn(m) +

n−1∑

j=0

(n + 1

j

)

Tj(m) ,

et puisque(n+1

n

)= n + 1, on en tire

Tn(m) =1

n + 1

(m + 1)n+1 − 1 −n−1∑

j=0

(n + 1

j

)

Tj(m)

.

ceci permet de calculer Tn(m) en connaissant T0(m), . . . , Tn−1(m).

3) A partir de T0(m) = m, on retrouve facilement T1(m) =m(m + 1)

2.

La relation de récurrence peut encore s’écrire, si n ≥ 2,

Tn(m) =1

n + 1

(m + 1)n+1 − (m + 1) −n−1∑

j=1

(n + 1

j

)

Tj(m)

.

On montre alors par récurrence que Tn est un polynôme de degré n+1 admettant pour racines 0 et −1.

EI 6

C’est vrai si n = 1. Supposons la propriété vraie jusqu’à l’ordre n − 1. Alors les polynômes Tj s’an-nulent en 0 et −1 et le polynôme (m+1)n+1−(m+1) également. Donc Tn s’annule aussi en 0 et −1. Parailleurs tous ces polynômes sont de degrés distincts, et le plus grand des degrés est n+1. La somme estde degré n+1. La propriété est donc vraie au rang n. Il en résulte qu’elle est vraie pour tout entier n ≥ 1.

5 Etablir, pour n ∈ N∗, les identités :

g)

n∑

p=0

(n

p

)

(−1)p = 0

h)n∑

j=0

(n

2j

)

=n∑

j=0

(n

2j + 1

)

= 2n−1

Solution

En appliquant la formule du binôme au développement de (1−1)n, on trouve la relation g). En séparantdans cette relation les indices paires et les indices impaires, elle s’écrit

n∑

j=0

(n

2j

)

−n∑

j=0

(n

2j + 1

)

= 0 ,

Mais, en développant (1 + 1)n, et en séparant de même, on obtient

n∑

j=0

(n

2j

)

+

n∑

j=0

(n

2j + 1

)

= 2n ,

On en déduit donc quen∑

j=0

(n

2j

)

=

n∑

j=0

(n

2j + 1

)

= 2n−1 .

6 Etablir l’identité :

i)

(2n

n

)

=

n∑

k=0

[(n

k

)]2

Solution

On cherche le coefficient de Xn dans le polynôme (X + 1)2n. Par la formule du binôme, c’est(2n

n

).

Mais on peut écrire aussi

(X + 1)2n = (X + 1)n(X + 1)n =

(n∑

k=0

(n

k

)

Xk

)(n∑

k=0

(n

n − k

)

Xn−k

)

,

EI 7

et le coefficient de Xn dans le produit est la somme

n∑

k=0

(n

k

)(n

n − k

)

=

n∑

k=0

[(n

k

)]2

,

ce qui donne bien l’égalité(

2n

n

)

=n∑

k=0

[(n

k

)]2

.

7 Etablir, si 0 ≤ n ≤ n2, les identités :

j)

n∑

k=0

k

(n1

k

)(n2

n − k

)

= n1

(n1 + n2 − 1

n − 1

)

k)

n∑

k=0

k(k − 1)

(n1

k

)(n2

n − k

)

= n1(n1 − 1)

(n1 + n2 − 2

n − 2

)

Solution

j) On cherche le coefficient de Xn−1 dans le produit P (X) = [(X + 1)n1 ]′(X + 1)n2 . On a d’une part

P (X) = n1(X + 1)n1+n2−1 ,

donc le coefficient de Xn−1 est n1

(n1+n2−1

n−1

). D’autre part

P (X) =

(n1∑

k=1

k

(n1

k

)

Xk−1

)

n2∑

j=0

(n2

j

)

Xj

.

Le coefficient de Xn−1 est la somme des produits des coefficients de Xk−1 dans la première somme etde Xj dans la deuxième, lorsque k− 1+ j = n− 1, soit j = n− k. Cette valeur doit être positive, donck ≤ n, et la somme vaut

n∑

k=1

k

(n1

k

)(n2

n − k

)

,

d’où, puisque pour k = 0 le coefficient est nul, l’égalité

n∑

k=0

k

(n1

k

)(n2

n − k

)

= n1

(n1 + n2 − 1

n − 1

)

.

k) On cherche cette fois le coefficient de Xn−2 dans Q(X) = [(X + 1)n1 ]′′(X + 1)n2 . On a d’une part

Q(X) = n1(n1 − 1)(X + 1)n1+n2−2 ,

EI 8

donc le coefficient de Xn−2 est n1(n1 − 1)(n1+n2−2

n−2

). D’autre part

Q(X) =

(n1∑

k=2

k(k − 1)

(n1

k

)

Xk−2

)

n2∑

j=0

(n2

j

)

Xj

.

Le coefficient de Xn−2 est la somme des produits des coefficients de Xk−2 dans la première somme etde Xj dans la deuxième, lorsque k− 2+ j = n− 2, soit j = n− k. Cette valeur doit être positive, donck ≤ n, et la somme vaut

n∑

k=2

k(k − 1)

(n1

k

)(n2

n − k

)

,

d’où, puisque pour k = 0 et 1 les coefficients sont nuls, l’égalité

n∑

k=0

k(k − 1)

(n1

k

)(n2

n − k

)

= n1(n1 − 1)

(n1 + n2 − 2

n − 2

)

.

8 Quel est le nombre de chiffres du nombre entier card(P(P(0, 1, . . . , 9))) ?

Solution

Puisque card(0, 1, . . . , 9) = 10, on a tout d’abord l’égalité

card(P(0, 1, . . . , 9)) = 210 = 1024 ,

puiscard(P(P(0, 1, . . . , 9))) = 21024 .

Pour déterminer le nombre de chiffres de ce nombre, on utilise le logarithme décimal (noté log). Eneffet, si un nombre entier A s’écrit

A =

p∑

k=0

ak10k ,

avec ap 6= 0, et 0 ≤ ak ≤ 9 si k est compris entre 1 et p, alors, on peut écrire A = B · 10p où

B =

p∑

k=0

ak10k−p .

Mais

1 ≤ B ≤ 9

p∑

k=0

10k−p < 9∞∑

k=0

10−k =9

1 − 1/10< 10 ,

et donc0 ≤ log B < log 10 = 1 .

Et puisquelog A = p + log B ,

EI 9

il en résulte que p est la partie entière de log A, et le nombre de chiffres de A est

p + 1 = [log A] + 1 .

Ici, A = 21024, donc, le nombre de chiffres de A est [1024 log 2] + 1. Comme on a l’encadrement

0, 301 < log 2 < 0, 3011 ,

on en déduit en multipliant par 1024 que

308, 2 < log A < 308, 4 ,

et donc [1024 log 2] = 308. Le nombre cherché est 309.

9 Combien existe-il de grilles de mots croisés 10 × 10 ? (Une grille de mots croisés 10 × 10 est untableau carré de 100 cases dont certaines peuvent-être noires).

Solution

Une grille est déterminée par le sous-ensemble des cases blanches. Le nombre de grilles est donc lenombre de parties d’un ensemble à 100 éléments, soit 2100.

10 De combien de façons peut-on classer (sans ex aequo) les 40 étudiants d’un groupe de TD?

Solution

C’est le nombre de permutations de 40 éléments donc 40 !

11 On note Sjn le nombre de surjections différentes que l’on peut former d’un ensemble à n éléments

sur un ensemble à j éléments. Par convention, on pose Sjn = 0 si j /∈ N

∗ ou n /∈ N∗ ou j > n.

1) Que valent S1n et Sn

n ?

2) Montrer que, si n ≥ 2, on a S2n = 2n − 2.

3) a) Montrer sans calcul que (∀n ∈ N∗) (∀k ∈ [ 0, n ] ∩ N)

n∑

j=1

(k

j

)

Sjn = kn.

b) En déduire une formule de récurrence permettant de calculer les nombres Skn.

4) Etablir sans calcul la relation de récurrence (valable si n ≥ 2) Sjn = j(Sj

n−1 + Sj−1n−1).

5) a) On note f(x) = ex − 1. Montrer que, pour tout entier k ∈ [ 0, n ] , il existe des constantes réellesa1(k), . . . , ak(k) = Ak

n telles que

dn

dxn[fn(x)] =

dn−k

dxn−k

k∑

j=1

aj(k)fn−j(x)(f ′)j(x)

.

EI 10

b) En déduire quen∑

k=1

(−1)n−kkn

(n

k

)

= n!.

c) Montrer que Sjn =

j∑

k=1

(−1)j−kkn

(j

k

)

.

Solution

Notons Fp un ensemble à p éléments, et f une surjection de Fn dans Fj .

1) Il y a une seule application de Fn dans F1, et cette application est surjective, donc S1n = 1. Les

applications surjectives d’un ensemble à n éléments dans un autre sont bijectives. Le nombre de bijec-tions est aussi le nombre de permutations de n objets, donc Sn

n = n!.

2) Si F2 = y1, y2, choisir une surjection f de Fn dans F2, c’est choisir une partie C de Fn non videet différente de Fn qui sera l’ensemble f−1(y1). Alors f−1(y2) est le complémentaire de C dansFn. Il y a 2n − 2 telles parties C. Donc on a S2

n = 2n − 2.

3) a) Il y a kn applications de Fn dans Fk. Si f est une telle application, alors, c’est une surjection deFn sur f(Fn). On peut donc compter les applications de la manière suivante : On choisit une partieDj de Fk à j éléments. Il y a

(kj

)façons de le faire, et pour chaque choix de Dj , il y a Sj

n surjections

possibles. Donc, on ak∑

j=1

(k

j

)

Sjn applications possibles. On en déduit bien

k∑

j=1

(k

j

)

Sjn = kn .

b) En isolant le terme d’indice k dans la somme précédente, on en déduit

Skn = kn −

k−1∑

j=1

(k

j

)

Sjn .

4) Soit Fn = x1, . . . , xn−1, xn et Fn−1 = x1, . . . , xn−1 On peut obtenir une surjection de Fn dansFj , d’une des deux manières suivantes :

− on part d’une surjection de Fn−1 sur Fj et on envoie xn sur un des j éléments de Fj . Il y a donc

jSjn−1 surjections de ce type.

− on part d’une surjection de Fn−1 sur une partie à j−1 éléments de Fj , et on envoie xn sur l’élément

de Fj qui n’appartient pas à la partie choisie. Il y donc jSj−1n−1 surjections de ce type.

On a donc bien

Sjn = j(Sj

n−1 + Sj−1n−1) .

5) a) On démontre la propriété par récurrence sur k, le nombre n étant fixé.

EI 11

On a f ′(x) = ex, et (fn)′ = nfn−1f ′. Alors

dn

dxn[fn(x)] =

dn−1

dxn−1[nfn−1(x)f ′(x)] ,

si l’on pose a1(1) = n = A1n, on a la propriété au rang 1.

Supposons la propriété vraie au rang k. Donc

dn

dxn[fn(x)] =

dn−k

dxn−k

k∑

j=1

aj(k)fn−j(x)(f ′)j(x)

.

On peut écrire

dn

dxn[fn(x)] =

dn−k−1

dxn−k−1

d

dx

k∑

j=1

aj(k)fn−j(x)(f ′)j(x)

.

Or

d

dx

k∑

j=1

aj(k)fn−j(x)(f ′)j(x)

=

k∑

j=1

aj(k)((n − j)fn−j−1(x)(f ′)j+1(x) + jfn−j(x)(f ′)j(x))

=

k∑

j=1

aj(k)(n − j)fn−j−1(x)(f ′)j+1(x) +

k∑

j=1

aj(k)jfn−j(x)(f ′)j(x)

=

k+1∑

j=2

aj−1(k)(n − j + 1)fn−j(x)(f ′)j(x) +

k∑

j=1

aj(k)jfn−j(x)(f ′)j(x) .

Donc si l’on pose

aj(k + 1) =

a1(k) si j = 1(n − j + 1)aj−1(k) + jaj(k) si 2 ≤ j ≤ k(n − k)ak(k) si j = k + 1

On aura

dn

dxn[fn(x)] =

dn−k−1

dxn−k−1

k+1∑

j=1

aj(k + 1)fn−j(x)(f ′)j(x)

.

Avec de plusak+1(k + 1) = (n − k)ak(k) = (n − k)Ak

n = Ak+1n .

EI 12

La propriété est donc vraie au rang k + 1. Alors elle sera vraie si 1 ≤ k ≤ n.En particulier, si k = n,

dn

dxn[fn(x)] =

n∑

j=1

aj(n)(ex − 1)n−jejx .

Mais on a aussi, en développant par la formule du binôme,

f(x)n =

n∑

k=0

(n

k

)

(−1)n−kekx ,

doncdn

dxn[fn(x)] =

n∑

k=1

(n

k

)

(−1)n−kknekx .

En calculant la valeur en zéro dans les deux expressions précédentes, on obtient

n∑

k=1

(n

k

)

(−1)n−kkn = an(n) = Ann = n! ,

ce qui donne la formule voulue.

c) On démontre par récurrence, que si n ≥ 1, la propriété suivante est vraie :

(Pn) si 1 ≤ j ≤ n, Sjn =

j∑

k=1

(−1)j−kkn

(j

k

)

.

On a S11 = 1, et la propriété est vraie à l’ordre 1. Supposons la vraie à l’ordre n−1. On a tout d’abord,

d’après b),n∑

k=1

(−1)n−kkn

(n

k

)

= n! = Snn ,

et aussi1∑

k=1

(−1)1−kk1

(1

k

)

= 1 = S1n .

On suppose que 2 ≤ j ≤ n. Alors, en appliquant l’hypothèse de récurrence, on a

Sjn−1 =

j∑

k=1

(−1)j−kkn−1

(j

k

)

et Sj−1n−1 =

j−1∑

k=1

(−1)j−1−kkn−1

(j − 1

k

)

.

En appliquant la question 4),

Sjn = j

(j∑

k=1

(−1)j−kkn−1

(j

k

)

+

j−1∑

k=1

(−1)j−1−kkn−1

(j − 1

k

))

= j

(j−1∑

k=1

(−1)j−kkn−1(

(j

k

)

−(

j − 1

k

)

) + jn−1

)

.

EI 13

Mais (j

k

)

−(

j − 1

k

)

=

(j − 1

k − 1

)

,

donc

Sjn = j

j∑

k=1

(−1)j−kkn−1

(j − 1

k − 1

)

.

Par ailleurs

j

(j − 1

k − 1

)

= j(j − 1)!

(k − 1)!(j − k)!= k

j!

k!(j − k)!= k

(j

k

)

,

d’où

Sjn =

j∑

k=1

(−1)j−kkn

(j

k

)

.

On a donc montré que, si 1 ≤ k ≤ n,

Sjn =

j∑

k=1

(−1)j−kkn

(j

k

)

,

ce qui est la propriété au rang n. La propriété est donc vraie pour tout n ≥ 1.

12 Si n ≥ 1, on note αn le nombre de partitions d’un ensemble à n éléments (nombre de Bell), et onpose α0 = 1.

1) Montrer que si n ≥ 0, αn+1 =

n∑

i=0

(n

i

)

αi (formule de Dobinski).

2) En déduire que si n ≥ 1, αn =1

e

∞∑

k=1

kn

k!.

3) Montrer que si n ≥ 0, αn =n∑

k=0

βn−kkn

k!où βm =

m∑

j=0

(−1)j

j!.

(On pourra utiliser l’exercice 11, question 5c).

Solution

1) Soit Fn+1 = x1, . . . , xn, xn+1. On choisit une partie Aj à j éléments contenant xn+1. Cela revientà choisir les j−1 autres éléments parmi n. Il y a donc

( nj−1

)façons de le faire. Si l’on complète Aj avec

une partition quelconque de son complémentaire, on aura une partition de Fn+1. Comme ACj contient

n + 1− j éléments, on obtient ainsi( nj−1

)αn+1−j partitions différentes. On obtient toutes les partitions

de Fn+1 en faisant varier j de 1 à n + 1. Donc

αn+1 =

n+1∑

j=1

(n

j − 1

)

αn+1−j =

n+1∑

j=1

(n

n − j + 1

)

αn+1−j .

EI 14

En posant i = n + 1 − j on trouve,

αn+1 =

n∑

i=0

(n

i

)

αi .

2) Si n est fixé, la série de terme général uk = kn/k! converge. En effet

uk+1

uk=

(k + 1)n

(k + 1)!

k!

kn=

1

k + 1

(k + 1

k

)n

et ceci tend vers 0 quand k tend vers l’infini. La règle de d’Alembert assure donc la convergence de lasérie.

On démontre la formule par récurrence. Si n = 1, on a

∞∑

k=1

k

k!=

∞∑

k=1

1

(k − 1)!=

∞∑

k=0

1

k!= e ,

donc

α1 = 1 =1

e

∞∑

k=1

k

k!,

et la formule est vraie au rang 1.

Supposons la formule vraie jusqu’à l’ordre k. Alors

αn+1 =

n∑

i=0

(n

i

)

αi = 1 +

n∑

i=1

(n

i

)

αi ,

et en utilisant la relation aux rangs inférieurs à n,

αn+1 = 1 +

n∑

i=1

(n

i

)1

e

(∞∑

k=1

ki

k!

)

.

En intervertissant les sommations

αn+1 = 1 +1

e

∞∑

k=1

1

k!

(n∑

i=1

(n

i

)

ki

)

.

On reconnaît la formule du binôme, donc

αn+1 = 1 +1

e

∞∑

k=1

1

k!((k + 1)n − 1)

= 1 +1

e

∞∑

k=1

(k + 1)n

k!− 1

e

∞∑

k=1

1

k!.

EI 15

En changeant d’indice dans la première somme, et en remarquant que

∞∑

k=1

1

k!= e − 1 ,

on obtient

αn+1 = 1 +1

e

∞∑

k=2

kn

(k − 1)!− e − 1

e=

1

e

∞∑

k=2

kn

(k − 1)!+

1

e.

Finalement

αn+1 =1

e

∞∑

k=2

kn+1

k!+

1

e=

1

e

∞∑

k=1

kn+1

k!,

ce qui donne la formule au rang n. La propriété est donc vraie pour tout n ≥ 1.

3) Calculons la somme λn =

n∑

k=1

βn−kkn

k!. En remplaçant βn−k par sa valeur, on trouve

λn =

n∑

k=1

n−k∑

j=0

(−1)j

j!

kn

k!

=

n∑

k=1

n−k∑

j=0

(j + k

k

)(−1)jkn

(j + k)!

.

Effectuons le changement de variable i = k + j dans la somme interne. On obtient

λn =

n∑

k=1

[n∑

i=k

(i

k

)(−1)i−kkn

i!

]

.

On intervertit les sommations, ce qui donne

λn =

n∑

i=1

1

i!

[i∑

k=1

(i

k

)

(−1)i−kkn

]

.

Mais on reconnaît la formule 5c) de l’exercice précédent, et donc, finalement

λn =

n∑

i=1

Sin

i!.

Il reste à voir que Sin/i! est le nombre de partitions d’un ensemble à n éléments en i sous-ensembles

pour en conclure que λn = αn.

Pour cela on remarque qu’à toute partition A1, A2, . . . , Ai de Fn, on peut associer i! surjections fdistinctes de Fn sur Fi, puisque cela revient à choisir une permutation (y1, y2, . . . , yi) des éléments deFi, et à poser f(x) = yi si x est dans Ai. Le nombre de surjections de Fn dans Fi est donc i! fois lenombre de partitions de Fn en i sous-ensembles.

13 De combien de façons peut-on tirer, dans un jeu de 52 cartes, une main de 5 cartes qui comporteexactement 2 dames et 2 cœurs ?

EI 16

Solution

Il y a dans le jeu 13 cœurs et 4 dames, donc 13 + 4 − 1 = 16 dames ou cœurs. (La dame de cœurne doit être comptée qu’une fois). Le nombre de cartes qui ne sont ni dame ni cœur est donc 52−16 = 36.

Cherchons le nombre de mains contenant exactement 2 dames et 2 cœurs. Il y a deux cas possibles :

− la main contient la dame de cœur. Il reste à choisir une dame parmi 3, un cœur parmi 12, et ladernière carte parmi 36 ce qui fait 3 × 12 × 36 = 1296 possibilités.

− la main ne contient pas la dame de cœur. On choisit 2 dames parmi 3, ce qui fait(32

)= 3, possi-

bilités, 2 cœurs parmi 12, ce qui fait(12

2

)= 66 possibilités, et la dernière carte parmi 36. On a donc

3 × 66 × 36 = 7128 possibilités.

Le résultat final est donc 1296 + 7128 = 8424 possibilités.

14 De combien de façons peut-on tirer, dans un jeu de 52 cartes, une main de 5 cartes :

a) qui comporte un brelan, mais ni carré ni full ?

(un brelan est un ensemble de trois cartes de même valeurs, un carré est un ensemble de quatre cartesde même valeurs, un full est un ensemble de cinq cartes dont trois ont même valeur et les deux autresune même autre valeur).

b) qui ne comporte ni 2 cartes au moins de la même valeur, ni 5 cartes dont les valeurs se suivent, ni5 cartes d’une même des 4 couleurs ?

(les quatre couleurs sont : Pique, Cœur, Carreau et Trèfle).

Solution

a) La main est donc de la forme AAABC, où A,B,C sont des valeurs deux à deux distinctes.

On choisit tout d’abord la valeur A de la première carte. Il y a 13 possibilités. On choisit 3 cartes decette valeur parmi 4. Il y a

(43

)= 4 possibilités.

On choisit ensuite deux autres valeurs parmi les 12 restantes. Il y a(12

2

)= 66 possibilités. Il y a 4

cartes possibles pour chacune de ces deux valeurs, soit 16 possibilités.

Le nombre de mains est donc 13 × 4 × 66 × 16 = 54912.

b) On cherche donc parmi cinq cartes ayant des valeurs distinctes les mains où les cartes ne se suiventpas, et celles qui ne sont pas d’une même couleur. Si on appelle A l’ensemble des mains où les valeurssont distinctes, B l’ensemble de celles où les cartes se suivent, et C l’ensemble de celles où les cartessont de même couleur, les ensembles B et C sont inclus dans A, et donc

|A \ (B ∪ C)| = |A| − |B ∪ C| .

EI 17

D’autre part|B ∪ C| = |B| + |C| − |B ∩ C| .

Donc|A \ (B ∩ C)| = |A| + |B ∩ C| − |B| + |C| .

Pour A, on choisit 5 valeurs parmi 13. Il y a(135

)= 1287 possibilités. Puis pour chaque valeur, on

choisit une des quatre cartes de cette valeur, ce qui fait 45 possibilités. Donc

|A| = 45

(13

5

)

= 1317888 .

Pour B, on choisit la première carte de la série, il y a 9 possibilités (lorsque la première carte est le 9,la dernière est le roi). Puis pour chaque valeur, on choisit une des quatre cartes de cette valeur, ce quifait 45 possibilités. Donc

|B| = 45 × 9 = 9216 .

Pour C, on choisit la couleur, il y a 4 possibilités. Puis pour chaque couleur, on choisit 5 cartes de lacouleur obtenue, ce qui fait

(135

)possibilités. Donc

|C| = 4

(13

5

)

= 5148 .

L’ensemble B∩C est constitué des mains dans lesquelles les cartes se suivent et sont de même couleur.On choisit la couleur, il y a 4 possibilités, et pour chaque couleur il y a 9 possibilités. Donc

|B ∩ C| = 36 .

Finalement|A \ (B ∪ C)| = 1317888 + 36 − 9216 − 5148 = 1303560 .

15 On pose l’une à côté de l’autre, dans l’ordre de leur sortie, 5 cartes tirées l’une après l’autre etsans remise, dans un jeu de 32 cartes. Combien de figures différentes peut-on obtenir qui comportentexactement 3 trèfles ?

Solution

On choisit la place des trèfles parmi les cinq cartes. Il y a(53

)= 10 possibilités.

On choisit les trois trèfles parmi 8, en tenant compte de l’ordre. Il y a A38 = 8×7×6 = 336 possibilités.

On choisit les deux autres cartes parmi les 24 cartes qui ne sont pas de trèfles, en tenant compte del’ordre. Il y a A2

24 = 24 × 23 = 552 possibilités.

Le nombre cherché est donc 10 × 336 × 552 = 1854720.

16 Reprendre l’exercice 15 dans le cas où l’on tire les 5 cartes une à une et avec remise.

EI 18

Solution

On choisit la place des trèfles parmi les cinq cartes. Il y a(53

)= 10 possibilités.

On choisit les trois trèfles parmi 8. Chaque carte peut être une des 8 possibles. Il y a 83 = 512 possi-bilités.

On choisit les deux autres cartes parmi les 24 cartes qui ne sont pas de trèfles. Chaque carte peut êtreune des 24 possibles. Il y a 242 = 576 possibilités.

Le nombre cherché est cette fois 10 × 512 × 576 = 2949120.

17 Les nombres n et p étant deux entiers naturels non nuls, quel est le nombre de solutions dans Np

de l’équation d’inconnue (x1, x2, . . . , xp),

x1 + x2 + · · · + xp = n ?

Solution

Si l’on pose yi = xi + 1, l’équation équivaut à

y1 + y2 + · · · + yp = n + p ,

avec (y1, y2, . . . , yn) dans N∗p.

Le nombre cherché est aussi le nombre de solutions de l’équation

y1 + y2 + · · · + yp = n + p ,

avec (y1, y2, . . . , yn) dans N∗p.

En écrivant n + p comme la somme de n + p nombres 1,

n + p = 1 + 1 + 1 + · · · + 1 ,

trouver une solution de l’équation revient à choisir p− 1 signes + parmi les n + p− 1 signe + figurantdans la relation ci-dessus.

n + p = 1 + · · · + 1︸ ︷︷ ︸

y1

⊕ 1 + · · · + 1︸ ︷︷ ︸

y2

⊕ · · · ⊕ 1 + · · · + 1︸ ︷︷ ︸

yp

,

Le nombre cherché est donc (n + p − 1

p − 1

)

=

(n + p − 1

n

)

.

EI 19

18 De combien de façons peut-on choisir, dans un ensemble de n ≥ 2 éléments, une paire de sous-ensembles disjoints non vides ?

Solution

Si n ≥ 2, notons an le nombre de façons d’écrire un ensemble Fn de n éléments comme réunion de 2sous-ensembles disjoints non vides.

Il y a 2n − 2 façons de choisir une partie A non vide de Fn et distincte de Fn, et en prenant pour B lecomplémentaire de A on a une partition A,B de Fn. Mais toute partition est obtenue deux fois decette manière, il en résulte que an = 2n−1 − 1.

Si l’on veut maintenant choisir, dans Fn une paire de sous-ensembles disjoints non vides, on commencepar choisir un sous-ensemble de k éléments (k ≥ 2), ce qui donne

(nk

)possibilités, et pour chacun de

ces sous-ensembles, il y a ak façons de l’écrire comme réunion de deux sous-ensembles disjoints nonvides. Alors le nombre cherché est

bn =

n∑

k=2

(n

k

)

ak =

n∑

k=2

(n

k

)

(2k−1 − 1) .

Comme la somme est nulle si k = 1, on a encore

bn =

n∑

k=1

(n

k

)

(2k−1 − 1) =1

2

n∑

k=1

(n

k

)

2k −n∑

k=1

(n

k

)

.

Chaque somme se calcule par la formule du binôme, et l’on a

bn =1

2((2 + 1)n − 1) − ((1 + 1)n − 1) =

1

2(3n − 1) − (2n − 1) ,

ce qui donne finalement

bn =1

2(3n + 1) − 2n .

EI 20

I - Probabilités sur un ensemble fini

19 Quel est le plus probable ?

– Obtenir au moins un as en lançant 4 dés ?– Obtenir au moins deux as en lançant 10 dés ?– Obtenir au moins une paire d’as en lançant 25 paires de dés ?– Obtenir au moins deux as en lançant 50 dés ?

Solution

a) On prend comme univers Ω l’ensemble des quadruplets de nombres compris entre 1 et 6. La nombred’éléments de Ω est 64. On cherche la probabilité de l’événement :

A1 = « ne pas obtenir d’as en lançant 4 dés ».

Il y a 54 quadruplets ne contenant pas l’as, donc P(A1) =54

64. La probabilité d’obtenir au moins un as

est donc

p1 = P(AC1 ) = 1 − 54

64≈ 0, 5177 .

b) On prend comme univers Ω l’ensemble des 10−uplets de nombres compris entre 1 et 6. La nombred’éléments de Ω est 610.

La probabilité de l’événement : A2 = « ne pas obtenir d’as en lançant 4 dés », est, par le même

raisonnement que dans a), le nombre P(A2) =510

610.

Le nombre d’éléments de l’ensemble B2 des 10−uplets contenant exactement un as se calcule de lamanière suivante.

Il y a 10 positions possibles pour l’as, et chaque autre élément peut prendre 5 valeurs possibles. Celafait donc 10 · 59 éléments. Alors

P(B2) =10 · 59

610.

La probabilité qu’il y ait au plus un as est donc, puisque A2 et B2 sont disjoints,

P(A1 ∪ A2) = P(A2) + P(B2) =510

610+

10 · 59

610= 3

510

610.

La probabilité d’obtenir au moins deux as est alors,

p2 = P((A2 ∪ B2)C) = 1 − 3

510

610≈ 0, 5154 .

c) On prend comme univers Ω l’ensemble des 25−uplets de couples de deux nombres compris entre 1et 6. Il y a 36 couples, et chaque élément du 25−uplet peur prendre 36 valeurs. Donc Ω possède 3625

éléments.

EI 21

L’ensemble A3 des couples ne contenant pas 2 as possède 3525 éléments et sa probabilité vaut

P(A3) =3525

3625,

Alors la probabilité qu’il y ait au moins une paire d’as est

p3 = P(AC3 ) = 1 − 3525

3625≈ 0, 5055 .

d) On procède comme dans b). On prend comme univers Ω l’ensemble des 50−uplets de nombrescompris entre 1 et 6. La nombre d’éléments de Ω est 650.

La probabilité de l’événement : A4 = « ne pas obtenir d’as en lançant 4 dés », est P(A4) =550

650.

Le nombre d’éléments de l’ensemble B4 des 50−uplets contenant exactement un as se calcule de lamanière suivante.

Il y a 50 positions possibles pour l’as, et chaque autre élément peut prendre 5 valeurs possibles. Celafait donc 50 · 549 éléments. Alors

P(B4) =50 · 549

650.

La probabilité qu’il y ait au plus un as est donc

P(A1 ∪ A2) = P(A4) + P(B4) =550

650+

50 · 549

650= 11

550

650.

La probabilité d’obtenir au moins deux as est alors

p4 = P((A4 ∪ B4)C) = 1 − 11

550

650≈ 0, 9987 .

C’est ce dernier événement qui est donc le plus probable.

20 Déterminer, en précisant à partir de quelles hypothèses, la probabilité de trouver, dans un groupede n personnes choisies au hasard, au moins deux personnes ayant leur anniversaire le même jour.Calculer une valeur approchée de cette probabilité pour n = 23, n = 41 et n = 57.

Solution

On suppose que l’année a 365 jours, et on considère que les dates d’anniversaire sont réparties demanière équiprobable. Une personne donnée a donc une probabilité 1/365 d’avoir son anniversaire unjour donné.

Si n > 365, il y a plus de personnes que de jours de l’année, donc il y a toujours deux personnes ayantleur anniversaire le même jour.

On suppose maintenant n ≤ 365. On peut modéliser le problème en disant que chaque personne tireun nombre au hasard parmi les nombres entiers de 1 à 365.

EI 22

L’univers Ω est l’ensemble des n−uplets formés de tels nombres. Le nombre d’éléments de Ω est donc365n.

L’ensemble A des n−uplets dont tous les éléments sont différents est alors An365, et donc

P(A) =An

365

365n.

La probabilité pour que deux personnes au moins aient leur anniversaire le même jour est alors

P(AC) = 1 − An365

365n= 1 − 365!

(365 − n)! 365n.

En évaluant les factorielles à l’aide de la formule de Stirling, on peut approcher P(A), si n est petitdevant 365, par

an =(365e−1)365

√2 × 365π

((365 − n)e−1)365−n√

2 × (365 − n)π

1

365n= e−n

(365

365 − n

)365−n+1/2

.

On trouve les valeurs approchées de P(AC) suivantes :

n = 23 : P(AC) ≈ 0, 5n = 41 : P(AC) ≈ 0, 9n = 57 : P(AC) ≈ 0, 99 .

21 Quelle serait la probabilité de perdre si l’on pouvait jouer une seule grille au Loto National ?

Solution

On dispose d’une grille de 49 numéros. On en choisit 6. Le tirage a déterminé 6 numéros. On cherchela probabilité pour qu’il y ait k numéros communs aux deux tirages.

On choisit comme univers Ω l’ensemble des choix de 6 éléments parmi 49. Il y en a(49

6

).

Si 0 ≤ k ≤ 6, soit Ak l’ensemble des choix où k numéros exactement sont dans la liste déterminée parle tirage. Il y a

(6k

)façons de choisir k numéros parmi 6, et il reste

( 436−k

)façons de choisir les autres

numéros. Donc le nombre d’éléments de Ak est(6k

)( 436−k

). On a alors

P(Ak) =

(6k

)( 436−k

)

(496

) .

On perd si l’on a obtenu moins de 2 numéros communs avec le tirage. La probabilité de perdre estdonc

P(A0) + P(A1) + P(A2) =

(436

)+(61

)(435

)+(62

)(434

)

(496

) ≈ 0, 98 .

EI 23

22 Au bridge, si le mort et son partenaire ont chacun un as exactement, est-il plus probable que lesdeux autres as soient dans la même main adverse ou partagés ?

Solution

Chaque joueur ayant 13 cartes d’un jeu de 52 cartes, on a deux ensemble de 13 cartes fixés, contenantchacun un as. Cherchons le nombre d’éléments des ensembles suivants :

A = « les deux autres mains contiennent chacune un as »

B = « une des deux dernières mains contient deux as ».

Il suffit de choisir les cartes d’une main pour que les cartes de l’autre soient déterminées.

Pour A, on choisit un as parmi les deux restants, et 12 cartes parmi les 24 cartes restantes qui ne sontpas des as, donc

|A| = 2

(24

12

)

.

Pour B, ou bien la main contient les deux as, et il reste 11 cartes à choisir parmi 24, ou bien la mainne contient pas d’as, et il reste 13 cartes à choisir parmi 24. Donc

|B| =

(24

13

)

+

(24

11

)

= 2

(24

11

)

.

Regardons le quotient |A|/|B|.

|A||B| =

24!

(12!)211! 13!

24!=

11! 13!

(12!)2=

13

12.

CommeP(A)

P(B)=

|A||B| =

13

12> 1 ,

on en déduit que la probabilité de A est plus grande que celle de B.

23 On tire 5 cartes, sans remise, dans un jeu de 52 cartes. Comparer la probabilité des événementssuivants :

A = « une paire », c’est-à-dire 2 cartes de même valeur

B = « deux paires »

C = « un brelan », c’est-à-dire 3 cartes de même valeur

D = « un flush », c’est-à-dire 5 cartes de la même couleur

E = « un full », c’est-à-dire 3 cartes d’une même valeur, les 2 autres d’une même autre valeurF = « une quinte », c’est-à-dire 5 cartes dont les valeurs se suivent, l’ordre étant :

as, 2, 3,. . . ,10, valet, dame, roi, as

G = « un carré (ou poker) », c’est-à-dire 4 cartes de même valeur

EI 24

H = « une quinte flush ».

Solution

Remarque : les différents événements ont été interprétés en un sens strict précisé à chaque fois.

L’univers Ω est constitué des choix de 5 objets parmi 52, donc possède(525

)éléments.

A Les valeurs des cartes sont aabcd, où, a, b, c, d désignent des valeurs distinctes.

On choisit la valeur de a. Il y a 13 possibilités.

On choisit 3 autres valeurs parmi les 12 restantes. Il y a(12

3

)possibilités.

Pour la valeur a, on a deux cartes parmi 4, soit(42

)

Pour les valeurs de b, c, et d, il y a à chaque fois 4 possibilités. Donc

|A| = 13

(12

3

)(4

2

)

43 = 1098240 et P(A) ≈ 0, 422 .

B Les valeurs des cartes sont aabbc, où, a, b, c, désignent des valeurs distinctes.

On choisit 2 valeurs parmi 13. Il y a(13

2

)possibilités.

On choisit une autre valeur parmi les 11 restantes. Il y a 11 possibilités.

Pour la valeur a, on choisit deux cartes parmi 4, soit(42

), de même pour b

Pour la valeur de c il y a 4 possibilités. Donc

|B| =

(13

2

)

11

(4

2

)2

4 = 123552 et P(B) ≈ 0, 0475 .

C Les valeurs des cartes sont aaabc, où, a, b, c, désignent des valeurs distinctes.

On choisit la valeur de a. Il y a 13 possibilités.

On choisit 2 autres valeurs parmi les 12 restantes. Il y a(122

)possibilités.

Pour la valeur a, on choisit 3 cartes parmi 4, soit(43

)

Pour les valeurs de b et c, il y a à chaque fois 4 possibilités. Donc

|C| = 13

(12

2

)(4

3

)

42 = 54912 et P(C) ≈ 0, 021 .

D Les couleurs des cartes sont aaaaa.

On choisit la couleur a, il y a 4 possibilités.

Pour la couleur a, on choisit 5 cartes parmi 13, soit(13

5

), donc

|D| = 4

(13

5

)

= 5148 et P(D) ≈ 0, 002 .

EI 25

E Les valeurs des cartes sont aaabb, où, a, b, désignent des valeurs distinctes.

On choisit les valeurs de (a, b). Il y a A213 possibilités.

Pour la valeur a, on choisit 3 cartes parmi 4, soit(43

).

Pour la valeur b, on choisit deux cartes parmi 4, soit(42

). Donc

|E| = A213

(4

2

)(4

3

)

= 3744 et P(E) ≈ 0, 0014 .

F Il y a 10 successions possibles des 5 cartes.

Chacune des cartes d’une succession peut prendre 4 couleurs possibles. Donc

|F | = 10 · 45 = 10240 et P(F ) ≈ 0, 0039 .

G Les valeurs des cartes sont aaaab, où, a, b, désignent des valeurs distinctes.

On choisit les valeurs de (a, b). Il y a A213 possibilités.

Pour la valeur b, il y a 4 possibilités Donc

|G| = 4A213 = 624 et P(G) ≈ 0, 00024 .

H Il y a 10 successions possibles des 5 cartes.

Chacune des successions peut prendre 4 couleurs possibles. Donc

|H| = 40 et P(H) ≈ 0, 000015 .

24 Combien peut-on définir sur Ω = a, b, c de probabilités P

a) telles que P(a, b) =1

4?

b) telles que P(a, b) = P(b, c) =1

4?

c) telles que P(a, b) = P(b, c) =3

4?

Solution

a) On doit avoir

P(c) = 1 − P(a, b) =3

4,

et

P(a) + P(b) = P(a, b) =1

4.

Il y a donc une infinité de probabilités possibles.

EI 26

b) On doit avoir

P(a) + P(b) = P(b) + P(c) =1

4.

Donc P(a) = P(c).

On doit avoir également

P(c) = 1 − P(a, b) =3

4,

alors

Pb) = 1 − P(a) − P(c) = 1 − 3

2< 0 ,

ce qui n’est pas possible. Il n’existe pas de probabilité vérifiant les conditions données.

c) On doit avoir

P(a) + P(b) = P(b) + P(c) =3

4.

Donc P(a) = P(c).

On doit avoir également

P(c) = 1 − P(a, b) =1

4,

alors

Pb) = 1 − P(a) − P(c) = 1 − 1

2=

1

2.

Il y a une probabilité et une seule possible.

25 Soit A et B deux événements aléatoires associés à un espace probabilisé (Ω, P). Montrer que

P(A ∩ B) − P(A)P(B) = −P(AC ∩ B) + P(AC)P(B)

= −P(A ∩ BC) + P(A)P(BC)

= P(AC ∩ BC) − P(AC)P(BC) .

Solution

On aP(A) = P(A ∩ BC) + P(A ∩ B) et P(BC) = 1 − P(B) ,

donc

P(A ∩ B) − P(A)P(B) = P(A) − P(A ∩ BC) − P(A)P(B)

= P(A)(1 − P(B)) − P(A ∩ BC)

= P(A)P(BC) − P(A ∩ BC) .

En permutant les rôles de A et de B, on a aussi

P(A ∩ B) − P(A)P(B) = P(B)P(AC) − P(B ∩ AC) .

EI 27

Enfin, en appliquant cette formule à AC et BC ,

P(AC ∩ BC) − P(AC)P(BC) = P(BC)P(A) − P(BC ∩ A) .

Les quatre différences sont donc égales.

26 Monsieur Dubouchon pêche à la ligne dans son étang où ne vivent que trois carpes et sept tanches.Il a décidé de pêcher jusqu’à ce qu’il ait pris quatre poissons. En supposant que chacun des dix poissonsait la même probabilité de se faire prendre (dans n’importe quel ordre) et qu’ils aient tous des poidsdifférents, déterminer la probabilité de chacun des événements aléatoires suivants :

A = « l’un des quatre poissons pris est une carpe »

B = « l’un au moins des quatre poissons pris est une carpe »

C = « le premier poisson pris est une carpe »

D = « le second poisson pris est une carpe »

E = « les deux premiers poissons pris sont des carpes »

F = « au moins un des deux premiers poissons pris est une carpe »

G = « chacun des trois derniers poissons pris pèse plus que le précédent ».

Solution

a) Les événements A et B ne dépendent pas de l’ordre dans lequel les poissons sont pêchés. Si l’on notepk la probabilité d’avoir k carpes (exactement) parmi les 4 poissons, on a une loi hypergéométriqueH(4, 3, 10). En effet si l’on prend pour Ω les ensembles de 4 poissons pris parmi 10, on a

|Ω| =

(10

4

)

.

Si Ak est formé des ensembles contenant k carpes, on choisit k carpes parmi 3, et 4− k tanches parmi7, donc

|A| =

(3

k

)(7

4 − k

)

,

ce qui donne bien

pk =

(3k

)( 74−k

)

(104

) .

On a alors

P(A) = p1 =

(31

)(73

)

(104

) =1

2.

P(BC) = p0 =

(74

)

(104

) =1

6,

donc

P(B) = 1 − P(BC) =5

6.

EI 28

b) Pour les autres événements, on tient compte de l’ordre, et on va prendre pour Ω l’ensemble desquadruplets formés avec les 10 poissons. Donc

|Ω| = A410 .

Les événements C et D ont la même probabilité. Si i est la position étudiée, il y a 3 carpes possiblesen i, et pour les trois autres positions, il reste 3 poissons à choisir parmi 9. Donc

|C| = |D| = 3A39 ,

et

P(C) = P(D) =3A3

9

A410

=3

10.

On remarque que c’est tout simplement la probabilité de tirer 3 carpes parmi 10 poissons.

Pour E, il y a A23 façons de choisir les carpes et A2

8 façons de choisir les deux autres poissons, donc

|E| = A23A

28 ,

et

P(E) =A2

3A28

A410

=A2

3

A210

=1

15.

Là aussi on constate que c’est la probabilité de tirer deux carpes parmi 10 poissons.

Alors

P(F ) = P(C) + P(D) − P(C ∩ D) = P(C) + P(D) − P(E) =8

15.

c) Pour l’événement G, on cherche le nombre de cas où les poids des 4 poissons sont ordonnés encroissant.

Choisir un quadruplet de poissons, dont les éléments sont rangés suivant l’ordre croissant des poids,revient à choisir un ensemble de 4 poissons parmi 10 et à les ranger ensuite par ordre croissant depoids. (L’ordre croissant étant imposé, il y a une seule façon de ranger les poissons par ordre croissantde poids une fois que l’ensemble de 4 poissons est choisi). Cela revient à choisir 4 éléments parmi 10,donc

|G| =

(10

4

)

,

et

P(G) =

(104

)

A410

=1

4!=

1

24.

27 On tire au hasard un entier entre 1 et 900.

1) Quelle est la probabilité p0 qu’il soit pair, mais ni multiple de 4, ni multiple de 6 ?

2) Pour tout j diviseur de 900, calculer P(Ej), où Ej= « multiple de j ».

EI 29

3) Exprimer p0 en fonction des P(Ej) et vérifier ainsi la réponse à la question 1).

Solution

1) Parmi les nombres entiers compris entre 1 et 900, il y a450 nombres pairs,225 multiples de 4,150 multiples de 6,75 multiples de 12 (ce sont les multiples à la fois de 4 et 6).

Le nombre de nombres multiples de 4 ou 6 est donc

150 + 225 − 75 = 300 ,

et le nombre de nombres pairs ni multiple de 4, ni multiple de 6 est

450 − 300 = 150 .

Et donc

p0 =150

900=

1

6.

2) Il y a 900/j multiples de j donc P(Ej) = 1/j.

3) On a

p0 = P(E2) − P(E4) − P(E6) + P(E12) =1

2− 1

4− 1

6+

1

12=

1

6.

28 On tire trois cartes au hasard, l’une après l’autre et sans remise, dans un jeu de 32 cartes. Quelleest la probabilité que la troisième carte tirée soit une dame ?

Solution

Il y a 4 façons de choisir la dame et A231 façons de choisir les deux premières cartes. Alors la probabilité

vaut4A3

31

A232

=4 · 31!32!

=1

8.

On remarque que la probabilité est celle de choisir une dame parmi 32.

29 Le directeur d’un établissement où travaillent plus de mille personnes, dont la moitié de femmes,n’a pas réussi à susciter de candidatures volontaires pour constituer une commission de 10 personneschargée de proposer des amendements au règlement intérieur de l’établissement.Il décide de tirer au sort 10 noms sur la liste du personnel.Montrer qu’il y a environ une chance sur quatre de trouver autant d’hommes que de femmes dans lacommission ainsi formée.

EI 30

Solution

Si l’on note 2n le nombre d’employés, et Ω l’ensemble des choix de 10 employés parmi 2n, le nombred’éléments de Ω est

(2n10

).

Si A est l’ensemble des choix où il y a 5 hommes et 5 femmes, on choisit 5 femmes parmi n et 5 hommesparmi n. Alors le nombre d’éléments de A est

(n5

)2. La probabilité cherchée est donc

p(A) =

(n5

)2

(2n10

) .

Rappelons que l’on peut approcher(n1

k )( n2n−k)

(n1+n2n )

par(nk

)ak(1 − a)n−k, lorsque n et k sont fixes, n1 et n2

sont grands et n1n1+n2

proche de a. Ici n1 + n2 = 2n, et donc a = 1/2, n = 10 et k = 5, donc p(A)s’approche par

b =

(10

5

)1

210=

63

256≈ 0, 246 .

La probabilité de A est proche d’un quart.

30 1) On place dans une boîte six boules indiscernables au toucher et marquées des nombres 1 à 6.On y fait neuf tirages d’une boule, au hasard et avec remise. Quelle est la probabilité qu’un tiers aumoins des boules tirées portent un nombre pair ?

2) On lance neuf dés (équilibrés). Quelle est la probabilité qu’un tiers au moins des dés montrent unnombre pair de points ?

3) Même question si on lance neuf fois le même dé.

Solution

1) La probabilité de tirer un nombre pair est de 1/2. La probabilité pk d’avoir k nombres pairs en 9tirages est alors donnée par la loi binomiale B(9, 1/2) et vaut

pk =

(9k

)

29.

On veut que le nombre de boules ayant un numéro pair soit supérieur à 3. On cherche la probabilitéde l’événement contraire, c’est-à-dire que le nombre de boules soit inférieur à 2. On a donc

P(AC) = p0 + p1 + p2 =

(90

)+(91

)+(92

)

29=

23

28.

Donc

P(A) = 1 − 23

28=

233

256≈ 0, 91 .

2) et 3) Ce sont les mêmes problèmes que dans 1).

EI 31

31 On tire au hasard un 100−échantillon dans une population comportant 4% d’individus d’un typeA. Déterminer l’ordre de grandeur (ou de petitesse) de la probabilité que cet échantillon comporteexactement vingt individus de type A.

Solution

Soit N le nombre d’individus de la population totale et N1 le nombre de personnes ayant le type A.La probabilité d’avoir 20 individus de type A dans un 100-échantillon est donc (loi hypergéométrique)

p =

(N120

)(N−N1

80

)

(N100

) ,

Le rapport a = N1/N donne la proportion d’individus de type A et vaut donc 4/100. Alors on peutapprocher p par

q =

(100

20

)

a20(1 − a)80 ≈ 0, 2 · 10−8 .

32 J’ai dans ma poche trois jetons identiques au toucher : l’un a ses deux faces blanches, le seconda ses deux faces noires et le troisième a une face noire et l’autre blanche. Ayant sorti de ma poche unjeton choisi au hasard, je n’en vois qu’une seule face : elle est blanche. Quelle est la probabilité quel’autre face de ce jeton soit blanche également ?

Solution

Notons Bv (respectivement Bc) la probabilité d’avoir une face blanche visible (respectivement cachée).

Comme il y a trois faces blanches sur les six faces des pions on a P(Bv) = 1/2.

On cherche la probabilité conditionnelle de l’événement « tirer une face blanche » sachant que l’on adéjà une face blanche. C’est donc

p = P(Bc/Bv) =P(Bc ∩ Bv)

P(Bv).

L’événement Bv ∩ Bc est l’événement « tirer le pion blanc ». Sa probabilité est donc 1/3. Alors

p =1/3

1/2=

2

3.

33 Cette année, 29 des 79 étudiants de l’UE A 27 du Master d’astrologie numérique appliquée àl’économie sont des filles et 50 ont moins de 21 ans. D’autre part, dans cette UE, 64% des moins de 21ans sont des garçons.

Déterminer la proportion de moins de 21 ans parmi les filles.

EI 32

Solution

Traduisons les éléments donnés en terme de probabilité. Notons

F l’événement « être une fille »

G l’événement « être un garçon »

A l’événement « être âgé de moins de 21 ans ».

On a déjà

P(F ) =29

79et P(A) =

50

79.

Le fait que 64% des moins de 21 ans soient des garçons se traduit par une probabilité conditionnelle :

la probabilité d’être un garçon sachant que l’on a moins de 21 ans, est de 64%, c’est-à-dire

P(G/A) =64

100.

La probabilité que l’on cherche est une autre probabilité conditionnelle : c’est la probabilité d’avoirmoins de 21 ans, sachant que l’on est une fille, c’est-à-dire

P(A/F ) =P(A ∩ F )

P(F ).

Comme l’ensemble (G,F ) est un système complet d’événements aléatoires, on a

P(A ∩ F ) = P(A) − P(A ∩ G) = P(A) − P(G/A)P(A) = P(A)(1 − P(G/A)) ,

on obtient

P(A/F ) =50

79

(

1 − 64

100

)79

29=

18

29.

34 Soient A et B deux parties d’un ensemble Ω telles que

A ∩ B 6= ∅ , A ∩ BC 6= ∅ , AC ∩ B 6= ∅ , AC ∩ BC 6= ∅ .

Déterminer une condition nécessaire et suffisante sur le quadruplet (p, q, r, s) de ] 0, 1 [ 4, pour qu’ilexiste une probabilité P sur Ω telle que :

(P(A/B), P(A/BC), P(B/A), P(B/AC )) = (p, q, r, s) .

Solution

On se pose le problème suivant : donner une condition nécessaire et suffisante sous la forme d’une ouplusieurs relations reliant p, q, r, s, pour que l’on puisse trouver quatre nombres x, y, z, t dans ] 0, 1 [ ,dont la somme vaut 1 et vérifiant

P(A ∩ B) = x , P(A ∩ BC) = y , P(B ∩ AC) = z , P(AC ∩ BC) = t .

EI 33

On poseraP(A) = X , P(B) = Y et P = 1/p , Q = 1/q , R = 1/r , S = 1/s .

Les nombres P , Q, R, S, sont dans ] 1, +∞ [ , et la condition sera donnée sur P , Q, R, S.

En remplaçant P(A/B), etc... par leur valeur, on part donc du système

P(A ∩ B) = pP(B)P(A ∩ BC) = qP(BC)

P(A ∩ B) = rP(A)P(B ∩ AC) = sP(AC)

.

ou encore

P(A ∩ B) = pYP(A ∩ BC) = q(1 − Y )

P(A ∩ B) = rXP(B ∩ AC) = s(1 − X)

.

En soustrayant la première ligne de la troisième, on obtient

rX − pY = 0 .

En additionnant les deux premières lignes, on trouve

pY + q(1 − Y ) = P(A ∩ B) + P(A ∩ BC) = P(A) = X ,

et en additionnant les deux dernières

rX + s(1 − X) = P(A ∩ B) + P(B ∩ AC) = P(B) = Y ,

ce qui donne le système linéaire

rX − pY = 0X + (q − p)Y = q(s − r)X + Y = s

.

C’est un système de trois équations à deux inconnues. Si l’on introduit les vecteurs colonnes

U =

r1

s − r

, V =

−pq − p

1

et W =

0qs

,

on remarque que les deux premiers vecteurs ne sont pas colinéaires puisque le déterminant

δ =

∣∣∣∣

r −p1 q − p

∣∣∣∣= qr + p(1 − r) =

P + Q(R − 1)

PQR> 0 .

Alors dire que le système est vérifié équivaut à dire que W est combinaison linéaire de U et de V , etdonc que le déterminant

∆ =

∣∣∣∣∣∣

r −p 01 q − p q

s − r 1 s

∣∣∣∣∣∣

EI 34

est nul. On calcule ce déterminant en soustrayant la première ligne de la deuxième et en l’ajoutant àla troisième, puis en développant par rapport à la première ligne. On obtient

∆ =

∣∣∣∣∣∣

r −p 01 − r q q

s 1 − p s

∣∣∣∣∣∣

= rq(s + p − 1) + ps(1 − r − q) .

Si l’on divise par le produit pqrs, dire que ∆ est nul revient à la condition nécessaire

(C) P + S − PS − (Q + R − QR) = 0 ,

ou encore

(P − 1)(S − 1) = (Q − 1)(R − 1) .

Montrons que cette condition est suffisante.

Si la condition (C) est vérifiée, on peut alors résoudre le système en résolvant le système formé desdeux premières équations.

rX − pY = 0

X + (q − p)Y = q,

ce qui donne facilement

X =1

δ

∣∣∣∣

0 −pq q − p

∣∣∣∣=

pq

δ=

R

P + Q(R − 1),

et

Y =1

δ

∣∣∣∣

r 01 q

∣∣∣∣=

rq

δ=

P

P + Q(R − 1).

On constate en particulier, puisque δ est strictement positif, qu’il en est de même de X et de Y . D’autrepart on obtient également

1 − X =P + QR − Q − R

P + Q(R − 1),

et en utilisant la relation (C)

1 − X =(P − 1)S

P + Q(R − 1).

On a aussi

1 − Y =Q(R − 1)

P + Q(R − 1),

et 1 − X et 1 − Y sont strictement positifs. Donc X et Y appartiennent à l’intervalle ] 0, 1 [ .

Alors, on à immédiatement

x = pY =1

P + Q(R − 1)∈ ] 0, 1 [ ,

y = q(1 − Y ) =R − 1

P + Q(R − 1)et 1 − y =

P + (Q − 1)(R − 1)

P + Q(R − 1),

EI 35

donc y appartient à ] 0, 1 [ , puis

z = s(1 − X) =P − 1

P + Q(R − 1)et 1 − z =

1 + Q(R − 1)

P + Q(R − 1),

et z appartient aussi à ] 0, 1 [ . Alors

x + y + z =P + R − 1

P + Q(R − 1)> 0 ,

et enfin

t = 1 − (x + y + z) = 1 − P + R − 1

P + Q(R − 1)=

(Q − 1)(R − 1)

P + Q(R − 1)> 0 ,

et on a donc également t ∈ ] 0, 1 [ .

On a donc bien obtenu quatre nombres appartenant à ] 0, 1 [ et dont la somme vaut 1. On pourratrouver une probabilité P telle que

P(A ∩ B) = x , P(A ∩ BC) = y , P(B ∩ AC) = z , P(AC ∩ BC) = t .

35 A l’ouverture, le rayon boulangerie d’un magasin en libre service contient cent baguettes dont xseulement sont fraîches, les autres étant l’invendu de la veille.

1. Madame N est la première cliente de la journée. Elle choisit au hasard k baguettes. Quelle est laprobabilité pk(m) que m d’entre elles soient fraîches ?

2. Si Madame N n’est pas la première cliente de la journée et si n baguettes (n ≤ 100− k) ont déjà étéachetées (toutes choisies au hasard) avant son arrivée, qu’est devenue la probabilité pk(m) ?

3. Les clients du magasin ne connaissent évidemment pas le nombre x (ils le supposent nul !). En fait,ce nombre est aléatoire : il dépend chaque jour du nombre aléatoire de baguettes vendues la veille.Pour tout entier i de [ 1, 100 ] , on note Ai l’événement : i des 100 baguettes que contient le rayon àl’ouverture sont fraîches. Montrer que la probabilité p1(0) d’avoir du pain rassis quand on achète uneseule baguette ne dépend des P(Ai) qu’au travers du nombre

m1 =

100∑

i=1

iP(Ai) .

Solution

1. Il s’agit d’une loi H(k, x, 100), et

pk(m) =

(xm

)(100−xk−m

)

(100k

) .

EI 36

2. La probabilité ne change pas. Voici une démonstration.

On suppose que n baguettes ont déjà été achetées. Il reste donc 100 − n baguettes.La probabilité que j baguettes fraîches aient été achetées avant que Madame N n’arrive est

(xj

)(100−xn−j

)

(100n

) .

La probabilité que Madame N obtienne m baguettes fraîches sachant que j ont déjà été achetées est(x−j

m

)(100−n−x+jk−m

)

(100−nk

) .

Donc la probabilité p′k(m) que Madame N obtienne m baguettes fraîches est dans ce cas

p′k(m) =n∑

j=0

(x−jm

)(100−n−x+jk−m

)

(100−nk

)

(xj

)(100−xn−j

)

(100n

) =n∑

j=0

((x−j

m

)(xj

))((100−n−x+j

k−m

)(100−xn−j

))

(100−nk

)(100n

) .

Mais on a la relation (A − P

H

)(A

P

)

=

(A

H

)(A − H

P

)

,

(car ces deux nombres sont égaux àA!

H!P !(A − H − P )!), et on en déduit

(x − j

m

)(x

j

)

=

(x

m

)(x − m

j

)

,

(100 − n − x + j

k − m

)(100 − x

n − j

)

=

(100 − x

k − m

)(100 − x − k + m

n − j

)

,

(100 − n

k

)(100

n

)

=

(100

k

)(100 − k

n

)

.

ce qui donne

p′k(m) =

n∑

j=0

(x−m

j

)(100−x−k+m

n−j

)

(100−kn

)

(xm

)(100−xk−m

)

(100k

) .

Mais la sommen∑

j=0

(x−mj

)(100−x−k+mn−j

)

(100−kn

)

est celle des coefficients de la loi hypergéométrique H(n, x − m, 100 − k) et vaut donc 1. On retrouvealors p′k(m) = pk(m).

3. Soit B l’événement « obtenir une baguette rassise ». La probabilité d’obtenir une baguette rassiesachant que i des 100 baguettes sont fraîches est donc

P(B/Ai) =100 − i

100.

EI 37

la probabilité cherchée est alors

p1(0) =

100∑

i=0

P(Bi/Ai)P(Ai)

=

100∑

i=0

100 − i

100P(Ai)

=

100∑

i=0

P(Ai) −1

100

100∑

i=0

iP(Ai) .

Mais la première somme vaut 1, puisque Ai | i ∈ [[ 0, 100 ]] est un système complet d’événementsaléatoires. Donc

p1(0) = 1 − 1

100

100∑

i=0

iP(Ai) = 1 − m1

100.

36 Une enquête a montré qu’en Bordurie le cancer des bronches touchait une femme sur 10 000 et 4hommes sur 10 000. La même enquête prouvait que 70% des cancers bronchiques chez la femme appa-raissent chez une fumeuse, contre 30% seulement chez une non-fumeuse, alors que ces proportions chezl’homme sont respectivement 90% et 10%. Un journaliste de Klow matin concluait son commentairede l’enquête en affirmant qu’une fumeuse avait moins de risque de contracter ce cancer qu’un fumeur.

1. Sur quoi ce journaliste a-t-il vraisemblablement basé sa conclusion ?

2. Savoir que la proportion de fumeurs est six fois plus élevée chez l’homme que chez la femme permet-ilde conclure sérieusement ?

Solution

1. Il a sans doute comparé les proportions 70% et 90%.

2. Notons C l’événement « avoir un cancer », F « être une femme », H « être un homme », Φ « fumer »,et traduisons les pourcentages donnés sous forme de probabilités conditionnelles. On a

P(C/F ) =1

10 000, P(C/H) =

4

10 000, P(Φ/F ∩ C) =

7

10, P(Φ/H ∩ C) =

9

10.

De plus

P(Φ/H) = 6 P(Φ/F ) .

On veut comparer P(C/F ∩ Φ) et P(C/H ∩ Φ).

EI 38

On a

P(C/F ∩ Φ) =P(C ∩ F ∩ Φ)

P(F ∩ Φ)

=P(Φ/F ∩ C)P(F ∩ C)

P(F ∩ Φ)

=P(Φ/F ∩ C)P(C/F )P(F )

P(Φ/F )P(F )

=P(Φ/F ∩ C)P(C/F )

P(Φ/F ).

Le même calcul, donne

P(C/H ∩ Φ) =P(Φ/H ∩ C)P(C/H)

P(Φ/H).

D’oùP(C/F ∩ Φ)

P(C/H ∩ Φ)=

P(Φ/F ∩ C)

P(Φ/H ∩ C)

P(C/F )

P(C/H)

P(Φ/H)

P(Φ/F ).

On obtient doncP(C/F ∩ Φ)

P(C/H ∩ Φ)=

7

9

1

46 =

7

6> 1 .

Donc P(C/F ∩ Φ) est supérieur à P(C/H ∩ Φ).

37 Mr A et Mme B présentent tous deux des symptômes qui ne peuvent être provoqués que parl’une ou l’autre de deux maladies M1 et M2 qui se soignent de façons fort différentes (et que l’onsuppose incompatibles). La maladie M2 étant statistiquement 99 fois plus fréquente que la maladie M1

leurs médecins présument être en présence de M2. Pour confirmer leurs diagnostics, ils prescrivent uneanalyse. De par son principe même, cette analyse ne peut donner de certitude : son résultat est positifchez 74,25% des malades atteints de la maladie M1 et seulement chez 0,75% des malades atteints deM2 (et non pas 100% et 0%).

Les résultats sont négatifs pour Mr A et positifs pour Mme B. Faut-il en conclure que Mr A est atteintde M2 et Mme B de M1 ?

Solution

On traduit les hypothèse avec des probabilités. Notons T+ l’événement « le test est positif » et T−

l’événement « le texte est négatif ».

P(M2) = 0, 99 , P(M1) = 0, 01 , P(T+/M1) = 0, 7425 , P(T+/M2) = 0, 0075 .

On en déduit

P(T−/M1) = 1 − 0, 7425 = 0, 2575 et P(T−/M2) = 1 − 0, 0075 = 0, 9925 .

On calcule P(M2/T−) et P(M1/T+) par la formule de Bayes.

EI 39

P(M2/T−) =P(T−/M2)P(M2)

P(T−/M1)P(M1) + P(T−/M2)P(M2)≈ 0, 997 .

De même

P(M1/T+) =P(T+/M1)P(M1)

P(T+/M1)P(M1) + P(T+/M2)P(M2)≈ 0, 500 .

Si l’on peut conclure que Mr A est atteint de M2, l’incertitude reste pour Mme B.

38 Un vigile a, dans sa poche, dix clefs toutes différentes mais indiscernables au toucher. Il doitouvrir, dans la quasi-obscurité, une des portes de l’entrepôt qu’il surveille et, pour ce faire, il essaie lesclefs l’une après l’autre, au hasard.

Certaines nuits, il remet dans une autre poche toute clef essayée et qui n’a pas ouvert le porte. Lesautres nuits, il remet dans la même poche toute clef essayée sans succès.

Un cambrioleur probabiliste a remarqué que le vigile emploie la seconde méthode quand, et seulementquand, il a trop copieusement arrosé son repas, ce qui se produit de façon aléatoire avec une probabilité1/10 (et le rend inoffensif).

Déterminer la probabilité pk que le voleur, arrivé sur les lieux après le repas du vigile et très presséd’en finir, puisse agir sans risque quand il a constaté que la porte n’est pas encore ouverte après lak−ième tentative.

Solution

Si 1 ≤ k ≤ 9, notons Ak l’événement « utiliser k clefs sans avoir la bonne » et B l’événement « avoirtrop arrosé le repas ». La probabilité cherchée est

pk = P(B/Ak) ,

et on la calcule par la formule de Bayes.

pk =P(Ak/B)P (B)

P(Ak/B)P (B) + P(Ak/BC)P (BC).

On a P (B) = 1/10. Par ailleurs, lorsque le repas est bien arrosé, le vigile effectue un tirage avec remise,chacune des k clefs a la même probabilité 9/10 d’être mauvaise, donc

P (Ak/B) =

(9

10

)k

.

Si le vigile est sobre, il effectue un tirage sans remise, la j−ième clef a une probabilité (10 − j)/(10 − j + 1)d’être mauvaise, sachant que les j − 1 précédentes l’étaient, donc

P (Ak/BC) =

9

10

8

9· · · 10 − k

10 − k + 1=

10 − k

10.

EI 40

Alors

pk =

(9

10

)k 1

10(

9

10

)k 1

10+

10 − k

10

9

10

=9k−1

9k−1 + 10k − k10k−1.

39 Soit Ω et Ω′ deux univers, h une application de Ω sur Ω′ et B′kk∈K un système complet d’évé-

nements de P(Ω′). Montrer que h−1(B′k)k∈K est un système complet d’événements de P(Ω).

Solution

Soit ω dans Ω. Alors h(ω) appartient à Ω′. Comme

Ω′ =⋃

k∈K

B′k ,

il existe un indice k dans K tel que h(ω) appartienne à B′k. Il en résulte que ω appartient à h−1(B′

k).Donc

Ω =⋃

k∈K

h−1(B′k) .

Soit ω dans h−1(B′k)∩h−1(B′

j), où k 6= j. Alors h(ω) appartient à B′k ∩B′

j , mais cette intersection est

vide ce qui n’est pas possible. Il en résulte que h−1(B′k)∩ h−1(B′

j) est vide également. Les événements

h−1(B′k) sont deux à deux incompatibles.

Soit ω′ dans B′k. Comme h est surjective, il existe ω dans Ω tel que h(ω) = ω′. Alors ω appartient à

h−1(B′k) qui n’est donc pas vide.

On a donc bien montré que h−1(B′k)k∈K est un système complet d’événements de P(Ω).

40 1. Soit h une application d’un ensemble fini Ω dans un ensemble fini W , A et A′ deux partiesquelconques de W , (Ai)i∈I une famille quelconque de parties de W et (Bi)i∈I une famille de partiesde W deux à deux disjointes. Montrer que

EI 41

(a) h−1(AC) = (h−1(A))C

(b) h−1

(⋂

i∈I

Ai

)

=⋂

i∈I

h−1(Ai)

(c) h−1

(⋃

i∈I

Ai

)

=⋃

i∈I

h−1(Ai)

(d) h−1

j∈J

Bj

=⊎

j∈J

h−1(Bj)

(e) [A ⊂ A′] ⇒ [h−1(A) ⊂ h−1(A′)]

(f) h−1(A \ A′) = h−1(A) \ h−1(A′)

2. Démontrer que si (Ω, P) est un espace probabilisé, h une application définie sur Ω et Ω′ = h(Ω),alors l’application Ph définie sur P(Ω′) par

(∀A′ ∈ P(Ω′)) Ph(A′) = P(h−1(A′)) ,

est une probabilité sur Ω′.

Solution

(a) sera démontré plus loin comme application de (d).

(b) Soit x dans h−1

(⋂

i∈I

Ai

)

. Cela signifie que h(x) appartient à⋂

i∈I

Ai, ou encore, que pour tout i de

I, l’élément h(x) appartient à Ai. Cela veut dire que pour tout i de I, l’élément x est dans h−1(Ai)c’est-à-dire dans l’intersection de ces ensembles. On a donc bien

h−1

(⋂

i∈I

Ai

)

=⋂

i∈I

h−1(Ai) .

(c) Soit x dans h−1

(⋃

i∈I

Ai

)

. Cela signifie que h(x) appartient à⋃

i∈I

Ai, ou encore qu’il existe i de I, tel

que h(x) appartienne à Ai. Cela veut dire qu’il existe i de I, tel que x soit dans h−1(Ai) c’est-à-dire dansla réunion de ces ensembles. On a donc bien

h−1

(⋃

i∈I

Ai

)

=⋃

i∈I

h−1(Ai) .

EI 42

(d) Si l’on a, quels que soient les indices i et j de J distincts

Bi ∩ Bj = ∅ ,

alors d’après (b)

∅ = h−1(∅) = h−1(Bi ∩ Bj) = h−1(Bi) ∩ h−1(Bj) .

Donc les ensembles h−1(Bi) et h−1(Bj) sont disjoints.Par ailleurs d’après (c)

h−1

j∈J

Bj

=⋃

j∈J

h−1(Bj) .

Donc on a bien

h−1

j∈J

Bj

=⊎

j∈J

h−1(Bj) .

(a) AC est le complémentaire de A dans W . En écrivant

W = A ⊎ AC ,

et en appliquant (d), on a

Ω = h−1(W ) = h−1(A) ⊎ h−1(AC) .

Les ensembles h−1(A) et h−1(AC) sont disjoints et ont comme réunion Ω, donc h−1(AC) est le com-plémentaire dans Ω de h−1(A) c’est-à-dire

h−1(AC) = (h−1(A))C .

(e) Si A ⊂ A′ et si x est dans h−1(A), alors h(x) est dans A donc dans A′, donc x est dans h−1(A′),d’où l’inclusion

h−1(A) ⊂ h−1(A′) .

(f) On a

h−1(A \ A′) = h−1(A ∩ A′C) .

Donc en appliquant (b)

h−1(A \ A′) = h−1(A) ∩ h−1(A′C) ,

puis en appliquant (a)

h−1(A \ A′) = h−1(A) ∩ (h−1(A′))C ,

c’est-à-dire

h−1(A \ A′) = h−1(A) \ h−1(A′) .

2. Par définition de Ph et puisque P est une probabilité, l’application Ph est définie sur P(Ω′) et prendses valeurs dans R

+.

Puisque h−1(Ω′) = Ω, on a

Ph(Ω′) = P(h−1(Ω′)) = P(Ω) = 1 .

EI 43

Enfin si A et B sont deux sous-ensembles disjoints de Ω′,

Ph(A ⊎ B) = P(h−1(A ⊎ B)) = P(h−1(A)) ⊎ P(h−1(B)) .

Mais comme P est une probabilité c’est une fonction additive et

P(h−1(A)) ⊎ P(h−1(B)) = P(h−1(A)) + P(h−1(B) = Ph(A) + Ph(B) .

On a donc obtenuPh(A ⊎ B) = Ph(A) + Ph(B) ,

ce qui montre que Ph est additive.

L’application Ph est bien une probabilité sur Ω′.

41 Soit (Ω, P) un espace probabilisé.

1. Quelle est l’image de P par l’application constante k de Ω sur 0 ?

2. Si P est la loi uniforme sur −1, 0, 1, quelle est l’image de P par l’application h définie sur Ω parh(ω) = ω2 ?

Solution

1. Il n’y a que deux sous-ensembles de 0, et l’on a

Pk(0) = 1 et Pk(∅) = 0 .

2. On aΩ′ = h(Ω) = 0, 1 .

Puisque la loi est uniforme sur Ω, on a

P(−1) = P(0) = P(1) =1

3.

Alors

Ph(0) = P(h−1(0)) = P(0) =1

3,

et

Ph(1) = P(h−1(1)) = P(−1, 1) =2

3.

EI 44

II - Variables aléatoires

42 On tire au hasard, l’une après l’autre et sans remise, deux cartes d’un jeu de cartes dans lequelon n’a laissé que les dames et les rois. On choisit comme univers l’ensemble Ω de tous les arrangementsde deux cartes possibles et on note X1 (resp. X2) la variable aléatoire « couleur de la première (resp.seconde) carte tirée ».

1. L’événement aléatoire X1 est rouge est-il la paire cœur,carreau ?

2. Préciser les événements aléatoire X2 = cœur, X1 = X2 = pique, X1 = X2.

Solution

1. La réponse est NON. L’événement X1 est rouge est constitué des couples (A,B) où A et un carreauou un cœur. Il y a donc 4 possibilités pour A, et B peut prendre une des 7 autres valeurs. L’événementcontient 28 éléments.

2. L’événement X2 = cœur est constitué des couples (A,B) où B et un cœur. Il y a donc 2 possibilitéspour B, et A peut prendre une des 7 autres valeurs. Donc le nombre d’éléments est 14. Ce sont leséléments suivants

(Dpique,Dcœur), (Dcarreau,Dcœur), (Dtrèfle,Dcœur), (Rpique,Dcœur),

(Rcarreau,Dcœur), (Rtrèfle,Dcœur), (Rcœur,Dcœur), (Dcœur, Rcœur),

(Dpique, Rcœur), (Dcarreau, Rcœur), (Dtrèfle, Rcœur),

(Rpique, Rcœur), (Rcarreau, Rcœur), (Rtrèfle, Rcœur) .

L’événement X1 = X2 = pique est constitué des éléments suivants

(Dpique, Rpique), (Rpique,Dpique) .

L’événement X1 = X2 est constitué des éléments suivants

(Dpique, Rpique), (Rpique,Dpique), (Dcœur, Rcœur), (Rcœur,Dcœur) .

(Dcarreau, Rcarreau), (Rcarreau,Dcarreau), (Dtrèfle, Rtrèfle), (Rtrèfle,Dtrèfle) .

43 Soit X et Y deux variables aléatoires réelles définies sur le même univers fini Ω muni de laprobabilité P, f une application de R dans R, A et B deux parties finies de R. Montrer que,

EI 45

1. P(X ≤ 0) + P(X ≥ 0) = 1 + P(X = 0)

2. P(X /∈ A) = 1 −∑

x∈A

P(X = x)

3. P(X ∈ A/X ∈ B) =

x∈A∩B

P(X = x)

x∈B

P(X = x)si P(X ∈ B) > 0

4. P(Y = y) =∑

x∈X(Ω)

P(X = x et Y = y)

5. P(X ∈ A et Y ∈ B) =∑

(x,y)∈A×B

P(X = x et Y = y)

6. P(Y ≥ X ≥ 0) =∑

x∈X(Ω)x≥0

y∈Y (Ω)y≥x

P(X = x et Y = y)

7. P(Y = f(X)) =∑

x∈X(Ω)P(X=x) 6=0

P(X = x)

y∈Y (Ω)y=f(x)

P(Y = y/X = x)

Solution

Notons X(Ω) = x1, . . . , xn et Y (Ω) = y1, . . . , yp.

Si U est inclus dans Ω, remarquons que puisque l’événement x ∈ U est la réunion disjointes desévénements X = x lorsque x appartient à U , on aura

P(X ∈ U) =∑

x∈U

P(X = x) .

On applique cette propriété dans ce qui suit.

On rappelle aussi que si U et V sont inclus dans Ω

P(X ∈ U et X ∈ V ) = P(X ∈ (U ∩ V )) ,

P(X ∈ U ou X ∈ V ) = P(X ∈ (U ∪ V )) ,

etc. . .

EI 46

1. On aP(X ≤ 0 ou X ≥ 0) = P(X ≤ 0) + P(X ≥ 0) − P(X ≤ 0 et X ≥ 0) ,

MaisX ≤ 0 ou X ≥ 0 = Ω et X ≤ 0 et X ≥ 0 = X = 0 ,

donc1 = P(X ≤ 0) + P(X ≥ 0) − P(X = 0) ,

ce qui donne la formule voulue.

2. En calculant la probabilité de l’événement contraire, on a

P(x /∈ A) = 1 − P(x ∈ A) = 1 −∑

x∈A

P(X = x) .

3. En utilisant la définition de la probabilité conditionnelle, et les propriétés rappelées au début, onobtient immédiatement

P(X ∈ A/X ∈ B) =P(X ∈ A et X ∈ B)

P(x ∈ B)=

P(X ∈ (A ∩ B))

P(X ∈ B)=

x∈A∩B

P(X = x)

x∈B

P(X = x)

4. Comme (X = xj)1≤i≤n est un système complet d’événements aléatoires, on a (formule des proba-bilités totales)

P(Y = y) =

n∑

j=1

P(X = xi et Y = y) =∑

x∈X(Ω)

P(X = x et Y = y) .

5. L’événement X ∈ A et Y ∈ B est la réunion disjointe des événements X = x et Y = y, pourtous les couples (x, y) d’éléments de A × B. Cela donne immédiatement 5.

6. On a par définitionY ≥ X ≥ 0 = ω ∈ Ω |Y (ω) ≥ X(ω) ≥ 0 .

On peut écrire cet ensemble comme réunion disjointe :

Y ≥ X ≥ 0 =⊎

i

j

ω ∈ Ω |X(ω) = xi , Y (ω) = yj , yj ≥ xi ≥ 0

,

c’est-à-dire encore

Y ≥ X ≥ 0 =⊎

xi≥0

yj≥xi

ω ∈ Ω |X(ω) = xi , Y (ω) = yj

=⊎

x∈X(Ω)x≥0

y∈Y (Ω)y≥x

X = x et Y = y

.

EI 47

Donc, puisque les réunions sont disjointes

P(Y ≥ X ≥ 0) =∑

x∈X(Ω)x≥0

y∈Y (Ω)y≥x

P(X = x et Y = y)

.

7. Comme dans 6. on a

f(X) = Y = ω ∈ Ω | f(X(ω)) = Y (ω)=

(i,j)

ω ∈ Ω |X(ω) = xi , Y (ω) = yj , f(xi) = yj

=⊎

x∈X(Ω)P(X=x) 6=0

y∈Y (Ω)y=f(x)

X = x et Y = y

.

Donc, puisque les réunions sont disjointes

P(f(X) = Y ) =∑

x∈X(Ω)P(X=x) 6=0

y∈Y (Ω)y=f(x)

P(X = x et Y = y)

et en utilisant les probabilités conditionnelles

P(f(X) = Y ) =∑

x∈X(Ω)P(X=x) 6=0

y∈Y (Ω)yf(x)

P(X = x)P(Y = y/X = x)

,

et finalement

P(Y = f(X)) =∑

x∈X(Ω)P(X=x) 6=0

P(X = x)

y∈Y (Ω)y=f(x)

P(Y = y/X = x)

.

44 On lance quatre fois une pièce juste et on note X la variable aléatoire « nombre de séquencespile-face obtenues ». Par exemple, si on obtient successivement : pile, pile, face, pile, X prend la valeur1 ; si on obtient successivement : face, pile, pile, pile, X prend la valeur 0 ; si on obtient successivement :pile, face, pile, face, X prend la valeur 2. Déterminer la loi de probabilité de X.

Solution

Si l’on prend pour Ω l’ensemble des quadruplets P,F4, on a

|Ω| = 24 .

On a alorsX(Ω) = 0, 1, 2 .

EI 48

Il y a une seule manière d’avoir deux séquences pile-face : X = 2 = PFPF. Donc

P(X = 2) =1

16.

Cherchons le nombre de manières de ne pas avoir de séquence pile-face. Les F doivent sortir avant lesP . Il y a 5 cas possibles

X = 0 = FFFF,FFFP,FFPP,FPPP,PPPP .

Donc

P(X = 0) =5

16.

Alors

P(X = 1) = 1 − P(X = 0) − P(X = 2) =5

8.

45 On lance un dé juste, au plus cinq fois, en s’arrêtant (éventuellement) dès qu’on a obtenu un 6.et on note Y la variable aléatoire « nombre de lancers effectués ». Déterminer la loi de probabilité de Y .

Solution

On a Y (Ω) = [[ 1, 5 ]] .

Pour i ∈ [[ 1, 4 ]] , l’événement Y = i est l’événement « tirer un 6 au i−ème lancer et ne pas tirer un6 aux (i − 1) lancers précédents », donc

P(Y = i) =1

6

(5

6

)i−1

=5i−1

6i.

L’événement Y = 5 est la réunion de deux événements incompatibles :

1) « tirer un 6 au 5−ième lancer et ne pas tirer un 6 aux 4 lancers précédents » de probabilité54

65,

2) « ne pas avoir obtenu de 6 sur les 5 lancers », de probabilité55

65.

On a donc

P(Y = 5) =54

65+

55

65=

54

64.

On peut vérifier que la somme des probabilités trouvées vaut bien 1.

46 Une population E est constituée de N individus, N1 d’entre eux étant d’un type T .Après avoir fixé un entier n de [[ 1, N1 ]] , on fait des tirages au hasard dans E, sans remise (un individuà la fois) jusqu’à avoir obtenu n individus de type T , et on note Z la variable aléatoire « nombre detirages effectués ». Pour tout k de [[ n, N ]] , on note :

Ak−1 = « on obtient (n − 1) fois un T au cours des k − 1 premiers tirages » (si n ≥ 2)

EI 49

Bk = « on obtient un T au k−ième tirage ».

1. Déterminer P(Ak−1) et P|Ak−1(Bk) si k ≥ n ≥ 2.

2. Montrer que la variable Z a pour germe(

k,

(k−1n−1

)(N−kN1−n

)

(NN1

)

)

| k ∈ [[ n, N ]]

.

Solution

1. Pour calculer P(Ak−1), on se trouve dans la situation d’une loi hypergéométrique H(k − 1, N1, N)et donc

P(Ak−1) = pn−1 =

(N1n−1

)(N−N1k−n

)

( Nk−1

) .

Après k − 1 tirages, il reste N − k + 1 individus et N1 − n + 1 individus de type T , donc

P|Ak−1(Bk) =

N1 − n + 1

N − k + 1.

2. On a alors

P(Z = k) = P(Ak−1 ∩ Bk) = P|Ak−1(Bk)P(Ak−1)

=N1 − n + 1

N − k + 1

(N1n−1

)(N−N1k−n

)

( Nk−1

)

=N1!(N − N1)!(k − 1)!(N − k + 1)!(N1 − n + 1)

(n − 1)!(N1 − n + 1)!(k − n)!(N − N1 − k + n)!N !(N − k + 1)

=N1!(N − N1)!(k − 1)!(N − k)!

(n − 1)!(N1 − n)!(k − n)!(N − N1 − k + n)!N !

=

(k−1n−1

)(N−kN1−n

)

(NN1

) .

47 Soit n nombres réels y1, . . . , yn en progression arithmétique et X une variable aléatoire réelle quisuit une loi U(1, . . . , n).

Montrer qu’il existe un couple (a, b) de constantes réelles telles que la variable aléatoire réelle Y = aX+bsuive la loi U(y1, . . . , yn).

Solution

Si r est la raison de la suite arithmétique, on a, pour k ∈ [[ 1, n ]] ,

yk = r(k − 1) + y1 ,

EI 50

Donc, si l’on poseY = r(X − 1) + y1 = rX + y1 − r ,

on obtient une variable aléatoire Y telle que

Y (Ω) = y1, . . . , yn ,

avec

P(Y = yk) = P(rX + y1 − r = rk + y1 − r) = P(X = k) =1

n.

La variable aléatoire Y suit donc une loi uniforme sur y1 . . . , yn.

Comme r =yn − y1

n − 1, on a aussi

Y =yn − y1

n − 1X +

ny1 − yn

n − 1.

48 1. Déterminer la loi de probabilité de la variable T = n− S, où S est une variable aléatoire réellequi suit la loi B(n, a).

2. Déterminer la loi de probabilité de la variable V = n−U , où U est une variable aléatoire réelle quisuit la loi H(n,N1, N).

Solution

1. On a S(Ω) = [[ 0, n ]] , donc également T (Ω) = [[ 0, n ]] . Alors, si k ∈ [[ 0, n ]] , on a

P(T = k) = P(n − S = k) = P(S = n − k) =

(n

n − k

)

an−k(1 − a)k =

(n

k

)

(1 − a)kan−k .

Il en résulte que T suit la loi B(n, 1 − a).

2. On a U(Ω) = [[ 0, n ]] , donc également V (Ω) = [[ 0, n ]] . Alors, si k ∈ [[ 0, n ]] , on a

P(V = k) = P(U = n − k) =

( N1

n−k

)(N−N1

k

)

(Nn

) ,

et il en résulte que V suit une loi H(n,N − N1, N).

49 On tire au hasard, l’une après l’autre et avec remise, 14 cartes d’un jeu de 52 cartes. Déterminerla loi de probabilité de la variable aléatoire réelle T = « proportion de dames obtenues par rapport aunombre de cartes tirées ».

EI 51

Solution

Notons U la variable aléatoire « nombre de dames obtenues ». La variable U suit une loi B(14, 1/13)puisque la probabilité d’apparition d’une dame est 1/13. Donc, si k ∈ [[ 0, 14 ]] , on a

P (U = k) =

(14

k

)(1

13

)k (12

13

)14−k

=

(14

k

)1214−k

1314.

La proportion de dames est donc T = U/14, et T (Ω) = k/14 | k ∈ [[ 0, 14 ]] . Donc, si k ∈ [[ 0, 14 ]] ,

P(T = k/14) = P(U = k) =

(14

k

)1214−k

1314.

On remarquera que T ne suit pas une loi binomiale, puisqu’elle ne prend pas des valeurs entières.

50 Déterminer la loi de probabilité de la variable aléatoire Z = |W − n| où W est une variablealéatoire réelle qui suit la loi B(2n, 1/2).

Solution

Puisque W (Ω) = [[ 0, 2n ]] , on a (W − n)(Ω) = [[−n, n ]] , et Z(Ω) = [[ 0, n ]] .

Par ailleurs, si k appartient à [[ 0, 2n ]] , on a

P(W = k) =

(2nk

)

22n.

Donc

P(Z = 0) = P(|W − n| = 0) = P(W = n) =

(2nn

)

22n,

et si k appartient à [[ 1, n ]] ,

P(Z = k) = P(W − n = k ou W − n = −k) = P(W = n + k ou W = n − k) ,

et comme la réunion est disjointe

P(Z = k) = P(W = n + k) + P(W = n − k) =

( 2nn+k

)+( 2nn−k

)

22n=

2 ·( 2nn+k

)

22n=

( 2nn+k

)

22n−1.

51 Une série de trente mesures d’un caractère quantitatif a donné les résultats suivants :

xi = valeur observée 0,00261 0,00262 0,00263 0,00264 0,00265

ni = nombre d’observations de xi 3 9 13 4 1

Déterminer la valeur moyenne m de cette série statistique :1. par un calcul direct

EI 52

2. en utilisant la « moyenne provisoire » m0 = 0, 00263, c’est-à-dire en déduisant m de la valeurmoyenne m′ de la série statistique des x′

i = (xi − m0)105 où chaque x′

i aurait été observé ni fois.

Solution

1. On a 30 observations donc

m =1

30

5∑

i=1

ni xi =0, 07881

30= 0, 002627 .

2. Le tableau obtenu pour x′i devient

x′i = valeur obtenue −2 −1 0 1 2

ni = nombre d’observations de x′i 3 9 13 4 1

et donc

m′ =

5∑

i=1

ni

30= − 9

30= (m − m0)10

5 .

On en déduit

m = m0 + m′10−5 = 0, 00263 − 3

1010−5 = 0, 00263 − 0, 000003 = 0, 002627 .

52 Déterminer la moyenne de la loi U(1, . . . , n).

Solution

Puisque P(X = i) = 1/n, pour i ∈ [[ 1, n ]] , on a

E(X) =

n∑

i=1

i

n=

1

n

n∑

i=1

i =1

n

n(n + 1)

2=

n + 1

2.

53 Que représente pour une série statistique la moyenne de la loi de probabilité associée ?

Solution

C’est la moyenne arithmétique des nombres figurant dans la série (comptés avec leur multiplicité).

54 Déterminer l’espérance mathématique de chacune des variables aléatoires réelles X, Y et Z desexercices 44, 45 et 46.

EI 53

Solution

Le tableau des valeurs de X (exercice 44) est

xi 0 1 2

pi 5/16 5/8 1/16

Donc

E(X) =

3∑

i=0

xi pi =3

4= 0, 75 .

Le tableau des valeurs de Y (exercice 45) est

yi 1 2 3 4 5

pi 1/6 5/62 52/63 53/64 54/64

Donc

E(Y ) =

5∑

i=1

yi pi =4651

1296≈ 3, 59 .

Pour Z (exercice 46), on a

E(Z) =N∑

k=n

k

(k−1n−1

)(N−kN1−n

)

(NN1

) .

Mais

k

(k − 1

n − 1

)

= n

(k

n

)

,

donc

E(Z) =

N∑

k=n

n

(kn

)(N−kN1−n

)

(NN1

) .

En effectuant le changement d’indice de sommation j = k + 1, on trouve

E(Z) =

N+1∑

j=n+1

n

(j−1n

)((N+1)−jN1−n

)

(NN1

) .

Posons N1 + 1 = M1, N + 1 = M et n + 1 = p. On a alors

E(Z) =n(

NN1

)

M∑

j=p

(j − 1

p − 1

)(M − j

M1 − p

)

.

Mais d’après l’exercice 46,

(

j,

(j−1p−1

)(M−jM1−p

)

(MM1

)

)

| j ∈ p, . . . ,M

est le germe d’une probabilité P,

doncM∑

j=p

(j−1p−1

)(M−jM1−p

)

(MM1

) = 1 ,

EI 54

soitM∑

j=p

(j − 1

p − 1

)(M − j

M1 − p

)

=

(M

M1

)

,

et il en résulte que

E(Z) =n(

MM1

)

(NN1

) =n(

N+1N1+1

)

(NN1

) =n(N + 1)

N1 + 1.

55 Soit S une variable aléatoire de loi B(n, a).

1. Déterminer l’espérance mathématique de la variable aléatoire U = S(S − 1) et en déduire celle dela variable aléatoire V = S2.

2. Déterminer l’espérance mathématique de la variable aléatoire W = S3.

Solution

1. On a

E(U) =n∑

k=0

k(k − 1)ak(1 − a)n−k .

Pour calculer cette somme posons

f(x) = (x + 1 − a)n =

n∑

k=0

(n

k

)

xk(1 − a)n−k .

En dérivant deux fois, on obtient

f ′(x) = n(x + 1 − a)n−1 =

n∑

k=1

k

(n

k

)

xk−1(1 − a)n−k ,

puis

f ′′(x) = n(n − 1)(x + 1 − a)n−2 =n∑

k=2

k(k − 1)

(n

k

)

xk−2(1 − a)n−k .

En particulier

f ′′(a) = n(n − 1) =

n∑

k=2

k(k − 1)

(n

k

)

ak−2(1 − a)n−k ,

et en multipliant par a2

n(n − 1)a2 =n∑

k=0

k(k − 1)

(n

k

)

ak(1 − a)n−k .

(La somme peut commencer à 0, puisque les deux premiers termes sont nuls).

EI 55

On en déduit doncE(U) = n(n − 1)a2 .

Alors, puisque V = S2 = U + S, on obtient

E(V ) = E(U) + E(S) = n(n − 1)a2 + na = n2a2 + na(1 − a) .

2. Introduisons T = S(S − 1)(S − 2). On a

E(T ) =n∑

k=0

k(k − 1)(k − 2)

(n

k

)

ak(1 − a)n−k .

En dérivant une troisième fois la fonction f , on obtient

f (3)(x) = n(n − 1)(n − 2)(x + 1 − a)n−3 =

n∑

k=3

k(k − 1)(k − 2)

(n

k

)

xk−3(1 − a)n−k ,

donc

f (3)(a) = n(n − 1)(n − 2) =

n∑

k=3

k(k − 1)(k − 2)

(n

k

)

ak−3(1 − a)n−k ,

et en multipliant par a3

n(n − 1)(n − 2)a3 =

n∑

k=0

k(k − 1)(k − 2)

(n

k

)

ak(1 − a)n−k .

(La somme peut commencer à 0, puisque les trois premiers termes sont nuls).

On en déduit doncE(T ) = n(n − 1)(n − 2)a3 .

MaisT = S(S − 1)(S − 2) = S3 − 3S2 + 2S = W − 3V + 2S ,

doncW = T + 3V − 2S ,

et l’on obtient

E(W ) = E(T ) + 3E(V ) − 2E(S) = n(n − 1)(n − 2)a3 + 3(n(n − 1)a2 + na) − 2na ,

d’oùE(W ) = n(n − 1)(n − 2)a3 + 3n(n − 1)a2 + na .

56 On tire au hasard des cartes d’un jeu de cartes. Déterminer l’espérance mathématique de lavariable aléatoire réelle X = « proportion de piques parmi les cartes tirées ».

EI 56

Solution

Notons 4N le nombre de cartes du jeu. Supposons tout d’abord que le nombre de cartes tirées est unnombre n fixé, et appelons Xn la variable « proportion de piques parmi les cartes tirées parmi n ».

Le nombre Yn de piques suit une loi hypergéométrique H(n,N, 4N), donc

E(Yn) = nN

4N=

n

4.

et on en déduit

E(Xn) =E(Yn)

n=

1

4.

Maintenant, le nombre de cartes tirées n’est plus fixé et suit une loi N . On a alors

P(X = k/n) = P((X = k/n) /N = n)P(N = n) = P(Xn = k/n)P(N = n) .

Alors

E(X) =∑

1≤k≤n≤4N

k

nP(Xn = k/n)P(N = n) .

ce que l’on peut écrire

E(X) =4N∑

n=1

n∑

k=1

k

nP(Xn = k/n)P(N = n) =

4N∑

n=1

P(N = n)

(n∑

k=1

k

nP(Xn = k/n)

)

.

Alors

E(X) =

4N∑

n=1

P(N = n)E(Xn) =1

4

4N∑

n=1

P(N = n) =1

4.

57 Soit Y une variable aléatoire réelle de loi U(y1, . . . , yn) où les yi forment une progression arith-métique. Déduire des exercices 47 et 52 une expression de E(Y ) en fonction de n, y1 et yn.

Solution

D’après l’exercice 47, on peut écrire

Y =yn − y1

n − 1X +

ny1 − yn

n − 1,

où X suit une loi U(1, . . . , n). Alors

E(Y ) =yn − y1

n − 1E(X) +

ny1 − yn

n − 1.

EI 57

Mais d’après l’exercice 52

E(X) =n + 1

2,

donc

E(Y ) =yn − y1

n − 1

n + 1

2+

ny1 − yn

n − 1=

y1 + yn

2.

58 On sait que si X est une variable aléatoire réelle et si ϕ est une fonction affine, E(ϕ(X)) = ϕ(E(X)).A quelle condition sur la loi de probabilité de X cette égalité reste-t’elle vraie si ϕ est la fonction définiepar ϕ(x) = x2 ?

Solution

On aE((X − E(X))2) = E(X2) − E(X)2 .

La relationE(ϕ(X)) = ϕ(E(X)) ,

est donc équivalente àE((X − E(X))2) = 0 .

Cela signifie que P(X 6= E(X)) = 0, c’est-à-dire que X est constante presque sûrement.

59 On a fait le relevé des notes ki des 38 étudiants d’un groupe A et celui des notes ℓj des 34 étudiantsd’un groupe B, et on a additionné au fur et à mesure ces notes et leurs carrés. On a obtenu :

38∑

i=1

ki = 520

34∑

j=1

ℓj = 384

38∑

i=1

k2i = 10792

34∑

j=1

ℓ2j = 5632 .

La distribution des notes autour de la note moyenne du groupe est-elle plus dispersée dans le groupeA ou dans le groupe B ?

Solution

Notons NA et NB les variables aléatoires donnant les notes des groupes A et B respectivement. Oncompare les variances de ces variables aléatoires. On a

E(NA) =520

38et E(N2

A) =10 792

38,

donc

V(NA) =10 792

38−(

520

38

)2

=34924

361≈ 96, 7 .

De même

E(NB) =384

34et E(N2

B) =5 632

34,

EI 58

donc

V(NB) =5 632

34−(

384

34

)2

=11008

289≈ 38, 1 .

On a donc

V(NB) < V(NA) .

La distribution des notes du groupe A est plus dispersée que celle du groupe B.

60 Déterminer la variance σ2 de la loi de probabilité associée à la série statistique de l’exercice 51 :

1. par un calcul direct : σ2 =5∑

i=1

(xi − m)2ni

30,

2. en utilisant la moyenne provisoire m0, c’est-à-dire en déduisant σ2 de la variance σ′2 de la loi deprobabilité associée à la série statistique des x′

1.

Solution

1. On a obtenu m = 0, 002627. Le calcul de la somme donne

σ2 =1

30

[3(0, 00261 − 0, 002627)2 + 9(0, 00262 − 0, 002627)2 + 13(0, 00263 − 0, 002627)2

+4(0, 00264 − 0, 002627)2 + (0, 00265 − 0, 002627)2]

≈ 0, 87666 × 10−10 .

2. Pour les x′i, de moyenne m′ = −9/30, on obtient

x′i −2 −1 0 1 2

x′i2 4 1 0 1 4

ni 3 9 13 4 1

On en déduit que

5∑

i=1

x′i2ni = 29, Donc

σ′2 =29

30−(

− 3

10

)2

=263

300.

Donc puisque xi = 10−5x′i + m0, on obtient

σ2 = 10−10σ′2 =263

30010−10 ≈ 0, 87666 × 10−10 .

61 Déterminer la variance de la loi de probabilité U(1, . . . , n).

EI 59

Solution

On a

E(X2) =n∑

k=1

k2

n=

1

n

n(n + 1)(2n + 1)

6=

(n + 1)(2n + 1)

6.

Donc

V(X) = E(X2) − E(X)2 =(n + 1)(2n + 1)

6−(

n + 1

2

)2

=n2 − 1

12.

62 Que représente pour une série statistique la variance de la loi de probabilité associée ?

Solution

Elle représente la moyenne arithmétique des carrés des écarts de la série statistique avec la moyennearithmétique de la série.

63 Déterminer la variance des variables aléatoires X, Y et Z des exercices 44, 45 et 46.

Solution

Le tableau des valeurs de X (exercice 44) est

xi 0 1 2

x2i 0 1 4

pi 5/16 5/8 1/16

Donc

E(X2) =3∑

i=0

x2i pi =

7

8.

et

V(X) = E(X2) − E(X)2 =7

8−(

3

4

)2

=5

16= 0, 3125 .

Le tableau des valeurs de Y (exercice 45) est

yi 1 2 3 4 5

y2i 1 4 9 16 25

pi 1/6 5/62 52/63 53/64 54/64

Donc

E(Y 2) =

5∑

i=1

y2i pi =

6637

432,

EI 60

et

V(Y ) = E(Y 2) − E(Y )2 =6637

432−(

4651

1296

)2

=4172855

1679616≈ 2, 48 .

Pour Z (exercice 46), on calcule tout d’abord E(Z(Z + 1)). On a

E(Z(Z + 1)) =

N∑

k=n

k(k + 1)

(k−1n−1

)(N−kN1−n

)

(NN1

) .

Mais

k(k + 1)

(k − 1

n − 1

)

= n(n + 1)

(k + 1

n + 1

)

,

donc

E(Z(Z + 1)) =N∑

k=n

n(n + 1)

(k+1n+1

)(N−kN1−n

)

(NN1

) .

En effectuant le changement d’indice de sommation j = k + 2, on trouve

E(Z(Z + 1)) =

N+2∑

j=n+2

n(n + 1)

(j−1n+1

)((N+2)−jN1−n

)

(NN1

) .

Posons N1 + 2 = M1, N + 2 = M et n + 2 = p. On a alors

E(Z(Z + 1)) =n(n + 1)(NN1

)

M∑

j=p

(j − 1

p − 1

)(M − j

M1 − p

)

.

Mais d’après l’exercice 46,

(

j,

(j−1p−1

)(M−jM1−p

)

(MM1

)

)

| j ∈ p, . . . ,M

,

est le germe d’une probabilité P, donc

M∑

j=p

(j−1p−1

)(M−jM1−p

)

(MM1

) = 1 ,

soitM∑

j=p

(j − 1

p − 1

)(M − j

M1 − p

)

=

(M

M1

)

,

et il en résulte que

E(Z(Z + 1)) =n(n + 1)

(MM1

)

(NN1

) =n(n + 1)

(N+2N1+2

)

(NN1

) =n(n + 1)(N + 1)(N + 2)

(N1 + 1)(N1 + 2).

EI 61

Il en résulte, en utilisant l’exercice 54, que

V(Z) = E(Z(Z + 1)) − E(Z) − E(Z)2

=n(n + 1)(N + 1)(N + 2)

(N1 + 1)(N1 + 2)− n(N + 1)

N1 + 1−(

n(N + 1)

N1 + 1

)2

=n(N + 1)

(N1 + 1)2(N1 + 2)(−nN + nN1 + NN1 + N − N1 − N2

1 )

On peut écrire le dernier facteur

−nN + nN1 + NN1 + N − N1 − N21 = −[N2

1 − ((n − 1) + N)N1 + N(n − 1)] .

ce qui fait apparaître un trinôme du second degré de racines N et n − 1, donc

−nN + nN1 + NN1 + N − N1 − N21 = −(N1 − N)(N1 − n + 1) .

Finalement

V(Z) =n(N + 1)(N − N1)(N1 − n + 1)

(N1 + 1)2(N1 + 2).

64 Soit Y une variable aléatoire réelle de loi U(y1, . . . , yn) où les réels yi forment une progressionarithmétique. Exprimer V(Y ) en fonction de n, y1 et yn.

Solution

D’après l’exercice 47, on peut écrire

Y =yn − y1

n − 1X +

ny1 − yn

n − 1,

où X suit une loi U(1, . . . , n).

Alors

V(Y ) =

(yn − y1

n − 1

)2

V(X) .

et donc d’après l’exercice 61

V(X) =

(yn − y1

n − 1

)2 n2 − 1

12=

(n + 1)(yn − y1)2

12(n − 1).

65 Déterminer la loi de probabilité de la variable aléatoire X∗, réduite d’une variable aléatoire réelleX de loi B(a).

EI 62

Solution

Si X suit une loi B(a), on a E(X) = a et V(X) = a(1 − a).Donc

X∗ =X − E(X)√

V(X)=

X − a√

a(1 − a).

Lorsque X = 1, on a X∗ =

1 − a

a= α

Lorsque X = 0, on a X∗ = −√

a

1 − a= − 1

α

Donc X∗(Ω) = −1/α, α, et

P(X∗ = α) = P(X = 1) = a et P(X∗ = −1/α) = P(X = 0) = 1 − a .

66 Calculer P(X = 30) si X est une variable aléatoire de loi H(n,N1, 100) telle que E(X) = 10, 2 etE(X2) = 108, 8.

Solution

Si l’on pose a = N1/N = N1/100, on a

E(X) = na = 10, 2 et V(X) = na(1 − a)100 − n

99= E(X2) − E(X)2 = 4, 76 .

En faisant le quotient on en déduit

(1 − a)100 − n

99=

4, 76

10, 2,

donc

100 − 100a − n + na = 994, 76

10, 2,

et en remplaçant na par sa valeur, on en tire

n + N1 = n + 100a = 100 + 10, 2 − 994, 76

10, 2= 64 .

par ailleurs

nN1 = 100na = 1020 .

Les nombres n et N1 sont racines du trinôme X2 − 64X + 1020. Or ce trinôme a pour racines 30 et 34.On a donc deux possibilités

n = 30 N1 = 34 ou n = 34 N1 = 30 .

EI 63

Si l’on calcule la probabilité P(X = 30) pour la loi H(30, 34, 100) et pour la loi H(34, 30, 100), onobtient le même résultat

P(X = 30) =

(3430

)

(10030

) =

(704

)

(10034

) =34! 70!

4! 100!≈ 0, 15 × 10−20 .

67 Calculer P(X < E(X)) si X est une variable aléatoire de loi binomiale telle que E(X) /∈ N etE(X) = 2V(X).

Solution

Si n est entier et si n/2 ne l’est pas, notons α la partie entière de n/2. Alors

α <n

2< α + 1 ,

donc

2α < n < 2α + 2 ,

et comme n est entier, on en déduit

n = 2α + 1 .

Si X suit une loi B(n, a), on a

E(X) = na et V(X) = na(1 − a) .

Donc l’égalité E(X) = 2V(X) se traduit par

na = 2na(1 − a) ,

d’où l’on tire a = 1/2 et E(X) = n/2. On a alors

P(X < n/2) =

α∑

k=0

(nk

)

2n.

En faisant le changement d’indice de sommation j = n − k, on a

P(X < n/2) =

n∑

j=n−α

( nn−j

)

2n,

mais puisque n − α = α + 1,

P(X < n/2) =n∑

j=α+1

(nj

)

2n.

On a donc

P(X < n/2) = P(X > n/2) ,

EI 64

et puisque P(X = n/2) est nulle, on a

P(X < n/2) + P(X > n/2) = 1 ,

et on en déduit

P(X < n/2) =1

2.

68 Soit P une loi de probabilité sur un univers fini Ω ⊂ R et p un entier supérieur ou égal à 2. Ondit que des réels f1, . . . , fp−1 sont des fractiles d’ordre p de P si

(∀j ∈ [[ 1, p − 1 ]] )∑

x≤fj

P(x) ≥ j

pet

x≥fj

P(x) ≥ p − j

p.

1. Montrer que f1 ≤ · · · ≤ fp−1.

2. Déterminer une médiane (fractile d’ordre 2) et trois quartiles (fractiles d’ordre 4) de la loi de pro-babilité associée à la série statistique de l’exercice 51.

Solution

1. Supposons que l’on ait fj > fj+1. Alors

P( ] fj+1, fj [ ) = 1 − P( ]−∞, fj+1 ] ∪ [ fj , +∞ [ ) = 1 − (P( ]−∞, fj+1 ] ) + P( [ fj, +∞ [ )) .

Mais

P( ]−∞, fj+1 ] ) =∑

x≤fj+1

P(x) ≥ j + 1

pet P( [ fj , +∞ [ ) =

x≥fj

P(x) ≥ p − j

p.

Alors

P( ]−∞, fj+1 ] ) ≤ 1 − j + 1

p− p − j

p= −1

p< 0 ,

ce qui est impossible.

2. Complétons le tableau de l’exercice 51 en y plaçant les nombres 1/4, 1/2 et 3/4.

xi 0,00261 0,00262 0,00263 0,00264 0,00265

ni 3 9 13 4 1

P(xi) 3/30 9/30 13/30 4/30 1/30

P( ]−∞, xi ] ) 3/30 1/4 12/30 1/2 3/4 25/30 29/30 1

P( [xi, +∞ [ ) 1 27/30 3/4 18/30 1/2 1/4 5/30 1/30

EI 65

Pour les quartiles, on cherche f1, f2, f3 tels que

P( ]−∞, f1 ] ) ≥ 1

4P( [ f1, ∞ [ ) ≥ 3

4,

P( ]−∞, f2 ] ) ≥ 1

2P( [ f2, ∞ [ ) ≥ 1

2,

P( ]−∞, f3 ] ) ≥ 3

4P( [ f3, ∞ [ ) ≥ 1

4.

On peut prendref1 = x2 et f2 = f3 = x3 .

De plus x3 est également une médiane.

EI 66

III - Vecteurs aléatoires

69 Soit P une probabilité sur un produit cartésien fini Ω′ × Ω′′.

1. Montrer que si P est l’équiprobabilité, les probabilités marginales P′ et P

′′ de P sont elles-mêmes deséquiprobabilités.

2. Les probabilités P′ et P

′′ peuvent-elles êtres des équiprobabilités sans que P le soit ?

Solution

1. Si l’on a, pour 1 ≤ i ≤ m et 1 ≤ j ≤ n,

P((ω′i, ω

′′j )) =

1

mn,

Alors

P′(ω′

i) =

n∑

j=1

P((ω′i, ω

′′j )) =

n

mn=

1

m,

et

P′′(ω′′

j ) =m∑

i=1

P((ω′i, ω

′′j )) =

m

mn=

1

n.

Les probabilités marginales sont bien des équiprobabilités.

2. La réponse est NON. Prenons Ω′ = Ω′′ = 1, 2, et posons

P((1, 1)) = t .

Si l’on veut que P′ et P

′′ soient des équiprobabilités, on devra avoir

P((1, 1)) + P((1, 2)) = P((1, 1)) + P((2, 1)) =1

2,

donc

P((1, 2)) =1

2− t et P((2, 1)) =

1

2− t ,

et alors

P((2, 2)) =1

2− P((1, 2)) = t .

Il suffit de choisir t dans ] 0, 1/2 [ \1/4 pour avoir des probabilités marginales équiprobables, sansque P le soit.

70 On lance un dé (juste) n fois, puis une pièce de monnaie (équilibrée) autant de fois qu’on a obtenud’as sur le dé et on compte le nombre de piles obtenus. On note : Ω = Ω′ × Ω′′ où Ω′ = Ω′′ = [[ 0, n ]] .Déterminer

EI 67

1. la probabilité P sur Ω qui modélise cette expérience,

2. les probabilités marginales P′ et P

′′,

3. les fonctions probabilités « conditionnelles » sur [[ 0, n ]] , P′|j et P

′′|i définies par :

∣∣∣∣∣

(∀j ∈ [[ 0, n ]] ) P′|j(i) = P|Ω′×j(i × Ω′′)

(∀i ∈ [[ 0, n ]] ) P′′|i(j) = P|i×Ω′′(Ω′ × j) .

Solution

1. L’événement (i, j) n’est possible que si 0 ≤ j ≤ i ≤ n. Donc, si 0 ≤ i < j ≤ n, on a

P((i, j)) = 0 .

Dans le cas où 0 ≤ j ≤ i ≤ n, notons

Ai l’événement « on a obtenu i as »

Bj l’événement « on a obtenu j piles ».

Alors

P(i, j) = P(Ai ∩ Bj) = P|Ai(Bj)P(Ai) .

Mais la probabilité P(Ai) est obtenue à partir d’une loi binomiale B(n, 1/6), et la probabilité P|Ai(Bj)

est obtenue à partir d’une loi binomiale B(i, 1/2). Donc

P((i, j)) =

(n

i

)5n−i

6n

(i

j

)1

2i.

2. On a tout d’abord, si 0 ≤ i ≤ n,

P′(i) =

i∑

j=0

P((i, j))

=

i∑

j=0

(n

i

)5n−i

6n

(i

j

)1

2i

=

(n

i

)5n−i

6n

i∑

j=0

(i

j

)1

2i.

Et comme la somme vaut 1,

P′(i) =

(n

i

)5n−i

6n,

ce qui était prévisible, puisque c’est la loi binomiale B(n, 1/6) déjà mentionnée.

EI 68

Si 0 ≤ j ≤ n,

P′′(j) =

n∑

i=j

P((i, j))

=

n∑

i=j

(n

i

)5n−i

6n

(i

j

)1

2i.

On a (n

i

)(i

j

)

=n!

j!(i − j)!(n − i)!=

(n

j

)(n − j

i − j

)

,

donc

P′′(j) =

n∑

i=j

(n

j

)5n−i

6n

(n − j

i − j

)1

2i

=

(n

j

) n∑

i=j

(n − j

i − j

)5n−i

6n

1

2i.

En posant k = i − j, la somme devient

P′′(j) =

(n

j

) n−j∑

k=0

(n − j

k

)5n−j−k

6n

1

2k+j

=

(n

j

)1

6n

1

2j

n−j∑

k=0

(n − j

k

)

5n−j−k 1

2k.

On reconnaît alors la formule du binôme, et l’on obtient

P′′(j) =

(n

j

)1

6n

1

2j

(1

2+ 5

)n−j

=

(n

j

)11n−j

12n.

On a donc cette fois une loi B(n, 1/12).

3. On a

P′|j(i) = P|Ω′×j(i × Ω′′) =

P((i × Ω′′) ∩ (Ω′ × j))P(Ω′ × j) =

P((i, j))P′′(j) .

Donc

P′|j(i) =

(n

i

)5n−i

6n

(i

j

)1

2i

(n

j

)11n−j

12n

.

Mais (n

i

)(i

j

)

=

(n − j

i − j

)(n

j

)

,

EI 69

donc

P′|j(i) =

(n − j

i − j

)2n−i5n−i

11n−j=

(n − j

i − j

)10n−i

11n−j.

On a de manière analogue

P′′|i(j) =

P((i, j))P′(i) .

Donc

P′′|i(j) =

(n

i

)5n−i

6n

(i

j

)1

2i

(n

i

)5n−i

6n

=

(i

j

)1

2i.

71 Une secrétaire très étourdie glisse trois lettres qui ne diffèrent que par le nom de leur destinatairedans les trois dernières enveloppes qui lui restent puis cachète les enveloppes avant d’y coller les timbreset les trois étiquettes portant les adresses.

On note :

E = « la lettre tapée la première parvient à son destinataire »

Fj = « exactement j lettres parviennent à leur destinataire ».

Pour chaque indice j, étudier :

1. l’incompatibilité des événements aléatoires E et Fj ,

2. l’indépendance des événements aléatoires E et Fj .

Solution

Numérotons 1, 2, 3 les lettres. Nous avons les six possibilités suivantes, notées sous forme de triplet,

Ω = (1, 2, 3), (1, 3, 2), (2, 3, 1), (2, 1, 3), (3, 1, 2), (3, 2, 1) .

Chaque événement élémentaire a comme probabilité 1/6. On a alors

E = (1, 2, 3), (1, 3, 2) P(E) = 1/3

F1 = (1, 3, 2), (3, 2, 1), (2, 1, 3) P(F1) = 1/2

F2 = ∅ P(F2) = 0

F3 = (1, 2, 3) P(F3) = 1/6

On a égalementE ∩ F1 = (1, 3, 2) P(E ∩ F1) = 1/6 P(E)P(F1) = 1/6

E ∩ F2 = ∅ P(E ∩ F2) = 0 P(E)P(F2) = 0

E ∩ F3 = (1, 2, 3) P(E ∩ F3) = 1/6 P(E)P(F3) = 1/18

1. On constate que E et F2 sont incompatibles, mais que E et F1 et E et F3 ne le sont pas.

2. On constate donc que E et F1 sont indépendants, ainsi que E et F2, mais pas E et F3.

EI 70

72 Soit A et B deux événements aléatoires associés à un espace probabilisé (Ω, P). Montrer que

A ⊥⊥ B ⇔ AC ⊥⊥ B ⇔ A ⊥⊥ BC ⇔ AC ⊥⊥ BC .

Solution

Cela résulte de l’exercice 25. On a toujours

P(A ∩ B) − P(A)P(B) = −P(AC ∩ B) + P(AC)P(B)

= −P(A ∩ BC) + P(A)P(BC)

= P(AC ∩ BC) − P(AC)P(BC) .

Donc, lorsque un de ces nombres est nul, les quatre le sont.

73 Soit A1, . . . , An des événements aléatoires. Montrer que

(∀J ⊂ [[ 1, n ]] ) ( A1, . . . , An ⊥⊥ ⇔ A′1, . . . , A

′n ⊥⊥ )

où l’on a noté, pour tout j,

A′j =

Aj si j ∈ JAC

j si j /∈ J .

Solution

On montre tout d’abord que

A1, . . . , An ⊥⊥ ⇒ A1, . . . , An−1, ACn ⊥⊥ .

Soit Ai1 , . . . , Aip un sous-ensemble non vide de A1, . . . , An−1. Alors

P(Ai1 ∩ · · · ∩ Aip ∩ ACn ) = P(Ai1 ∩ · · · ∩ Aip) − P(Ai1 ∩ · · · ∩ Aip ∩ An)

= P(Ai1) · · ·P(Aip) − P(Ai1) · · · P(Aip)P(An)

= (1 − P(An))P(Ai1) · · · P(Aip)

= P(Ai1) · · ·P(Aip)P(ACn ) .

Si maintenant Ai1 , . . . , Aip un sous-ensemble non vide de A1, . . . , An−1, alors

P(Ai1 ∩ · · · ∩ Aip) = P(Ai1) · · · P(Aip) .

Donc, pour tout sous ensemble non vide A′i1

, . . . , A′ip de A1, . . . , An−1, A

Cn , on a

P(A′i1 ∩ · · · ∩ A′

ip) = P(A′i1) · · · P(A′

ip) .

Ceci montre que A1, . . . , An−1, ACn ⊥⊥.

Donc si l’on a une famille d’événements indépendants, on obtient une nouvelle famille d’événementsindépendants en remplaçant un des événements par l’événement contraire. Une récurrence immédiate,

EI 71

montre que cela reste vrai si l’on remplace un nombre fini d’événements par les événements contraires.Donc

(∀J ⊂ [[ 1, n ]] ) ( A1, . . . , An ⊥⊥ ⇒ A′1, . . . , A

′n ⊥⊥ ) .

Et comme (AC)C = A, on a également

(∀J ⊂ [[ 1, n ]] ) ( A′1, . . . , A

′n ⊥⊥ ⇒ A1, . . . , An ⊥⊥ ) ,

d’où l’équivalence.

74 Lors de trois donnes (= répartition au hasard de 52 cartes entre les 4 joueurs) successives d’unepartie de bridge, on note Ik, R, S et T les événements aléatoires suivants :

Ik = « le nombre total de cartes rouges dans le jeu du donneur au cours de la k−ième donne estimpair »

R= « le nombre total de cartes rouges dans le jeu du donneur au cours des deux premières donnes estimpair »

S= « le nombre total de cartes rouges dans le jeu du donneur au cours des deux dernières donnes estimpair »

T= « le nombre total de cartes rouges dans le jeu du donneur au cours des première et dernière donnesest impair »

Les événements R, S et T sont-ils indépendants ? Sont-ils indépendants deux à deux ?

Solution

Calculons la probabilité de Ik (elle ne dépend pas de k). Cherchons tout d’abord la probabilité pr

d’obtenir r cartes rouges au cours d’une donne. Cela revient à choisir 13 cartes parmi 52 et à regarderle nombre de cartes rouges obtenu. On a donc une loi hypergéométrique H(13, 26, 52), et

pr =

(26r

)( 2613−r

)

(5213

) .

Alors

P(Ik) =

6∑

s=0

p2s+1 =

6∑

s=0

( 262s+1

)( 2612−2s

)

(5213

) .

Si l’on fait le changement d’indice de sommation t = 6 − s, on obtient

P(Ik) =

6∑

t=0

( 2613−2t

)(262t

)

(5213

) =

6∑

t=0

p2t = 1 − P(Ik) .

Il en résulte donc que P(Ik) = 1/2.

On a alors

R = (I1 ∩ IC2 ) ⊎ (I2 ∩ IC

1 ) ,

EI 72

et comme les événements I1 et I2 sont indépendants, on a

P(R) = P(I1)P(IC2 ) + P(I2)P(IC

1 ) =1

2.

On a pour les mêmes raisons

P(R) = P(S) = P(T ) =1

2.

Notons ni le nombre de cartes rouges obtenu à la i−ème donne. Il n’est pas possible que les troisnombres n1 + n2, n2 + n3 et n1 + n3 soient tous impairs. Si par exemple n1 + n2 et n2 + n3 étaientimpairs, alors la somme n1 + n3 + 2n2 serait paire et n1 + n3 aussi. Donc R ∩ S ∩ T = ∅, et

P(R ∩ S ∩ T ) = 0 6= P(R)P(S)P(T ) .

Les événements R,S, T ne sont pas indépendants.

On a

R ∩ S = (I1 ∩ IC2 ∩ I3) ⊎ (IC

1 ∩ I2 ∩ IC3 ) ,

et comme les événements I1, I2, I3 sont indépendants

P(R ∩ S) = P(I1)P(IC2 )P(I3) + P(IC

1 )P(I2)P(IC3 ) =

1

4= P(R)P(S) ,

ce qui montre que R et S sont indépendants. Par symétrie du problème, il en est de même pour lesévénements S, T et R,T .

75 Cent urnes, numérotées de 0 à 99, contiennent chacune 99 boules indiscernables au toucher. Pourtout i, l’urne numéro i contient i boules rouges et 99− i boules blanches. On choisit au hasard une descent urnes et on y fait cinq tirages au hasard d’une boule avec remise. On constate que les cinq tiragesont tous amené une boule rouge. Déterminer la probabilité p que les trois tirages suivants (toujoursavec remise) donnent également des boules rouges.

Solution

Soit X la variable aléatoire donnant le numéro de l’urne choisie. Le choix étant fait au hasard, on adonc

P(X = i) =1

100.

Soit Rk l’événement « le k−ième tirage donne une boule rouge », et Sk l’événement

Sk =

k⋂

j=1

Rj .

On cherche la probabilité

p = P|S5(R6 ∩ R7 ∩ R8) =

P(S8)

P(S5).

EI 73

Comme les événement X = i | 0 ≤ i ≤ 99 forment un système complet, on a, si 0 < k ≤ 99,

P(Sk) =

99∑

i=0

P|X=i(Sk)P(X = i) =1

100

99∑

i=0

P|X=i(Sk) .

Mais les événements Rk sont indépendants pour la probabilité P|X=i et ont la même probabilité.Sachant que l’urne contient i boules rouges, la probabilité de tirer une boule rouge est de i/99, donc

P|X=i(Rk) =i

99et P|X=i(Sk) =

(i

99

)k

.

Finalement

P(Sk) =

99∑

i=0

(i

99

)k

,

et

p =

99∑

i=1

(i

99

)8

99∑

i=1

(i

99

)5=

1

993

99∑

i=1

i8

99∑

i=1

i5

.

76 On dispose de r boules identiques qu’on répartit au hasard dans n boîtes dont n1 sont de couleurc1, n2 d’une autre couleur c2,. . . , nd d’une autre couleur cd (où n1 + · · · + nd = n).

1. On procède d’une façon qui garantisse l’équiprobabilité sur l’ensemble des répartitions possibles(deux répartitions ne diffèrent donc que par le nombre de boules dans au moins une des n boîtes).Pour tout k, on note Xk = « nombre de boules en tout dans des boîtes de la couleur ck ». Déterminerla loi de probabilité du vecteur aléatoire X = (X1, . . . ,Xd) ainsi que les lois marginales.

2. Mêmes questions si les boules sont numérotées de 1 à r (ce qui oblige à distinguer deux répartitionsqui ne diffèrent que par les numéros de boules).

3. Mêmes questions si chaque boîte ne peut contenir qu’une boule au plus. (Les boules sont encorenumérotées).

Solution

1. Si l’on place a boules dans q boîtes, et si l’on note ai le nombre de boules de la boîte i, on aa1 + · · · + aq = a. Le nombre de façon de placer les boules est le nombre de façons d’écrire l’entier acomme somme de q entiers naturels. Ce nombre vaut

(a+q−1

a

)(voir l’exercice 17).

L’univers Ω est constitué des façons de placer r boules dans n boîtes, et donc

card Ω =

(n + r − 1

n

)

,

EI 74

On cherche P(X1 = x1, . . . ,Xd = xd). Cette probabilité est nulle si x1 + · · · + xd est différent de r.Dans le cas contraire, cherchons le nombre de façons d’obtenir cet événement.

Il y a(nj+xj−1

nj

)façons de remplir les nj boîtes de couleur cj avec les xj boules. Donc

card(X1 = x1, . . . ,Xd = xd) =

d∏

j=1

(nj + xj − 1

nj

)

,

et on en déduit

P(X1 = x1, . . . ,Xd = xd) =

d∏

j=1

(nj + xj − 1

nj

)

(n + r − 1

r

) si x1 + · · · + xd = r

0 sinon

.

On étudie la variable Xi. Plutôt que de calculer

P(Xj = xj) =∑

x1,...,xj−1,xj+1,...,xd

P(X = (x1, . . . , xd)) ,

on peut obtenir directement le nombre d’éléments de l’événement Xj = xj. Il y a(nj+xj−1

nj−1

)façons

de placer xj boules dans les nj boîtes de couleur cj et(n−nj+r−xj−1

n−nj−1

)façons de placer les r−xj boules

restantes dans les n − nj autres boîtes. Donc

cardXj = xj =

(nj + xj − 1

nj − 1

)(n − nj + r − xj − 1

n − nj − 1

)

,

et

P(Xj = xj) =

(nj+xj−1nj−1

)(n−nj+r−xj−1n−nj−1

)

(n+r−1

n

) .

2. Remarquons tout d’abord que le nombre de façons de placer p objets distincts dans q boîtes est qp,puisque pour chaque objet, il y a q façons de choisir la boîte qui le contient.

L’univers Ω est encore constitué des choix de r boules parmi n boîtes mais cette fois les boules sontnumérotées. Donc

card Ω = nr .

De nouveau P(X1 = x1, . . . ,Xd = xd) = 0 si x1 + · · · + xd 6= r. Dans le cas contraire cherchonscard(X1 = x1, . . . ,Xd = xd). Pour la couleur c1, on choisit x1 boules parmi r, il y a donc

( rx1

)

possibilités, et pour chacun de ces choix, il y a nx11 manières de placer les x1 boules dans les n1 boîtes.

EI 75

Pour la couleur c2, on choisit x2 boules parmi les r − x1 restantes. Il y a donc(r−x1

x2

)possibilités, et

pour chacun de ces choix, il y a nx22 manières de placer les x2 boules dans les n2 boîtes.

De manière générale, pour la couleur cj , on choisit xj boules parmi les r − x1 − · · · − xj−1 restantes. Ily a donc

(r−x1−···−xj−1

xj

)possibilités, et pour chacun de ces choix, il y a n

xj

j manières de placer les xj

boules dans les nj boîtes. Donc

card(X1 = x1, . . . ,Xd = xd) =d∏

j=1

[(r − x1 − · · · − xj−1

xj

)

nxj

j

]

.

et

P(X1 = x1, . . . ,Xd = xd) =

d∏

j=1

[(r − x1 − · · · − xj−1

xj

)

nxj

j

]

nr

=

d∏

j=1

[(r − x1 − · · · − xj−1

xj

) (nj

n

)xj

]

.

Remarquons que

d∏

j=1

(r − x1 − · · · − xj−1

xj

)

=r!

x1! · · · xd!.

On note ce nombre( rx1,...,xd

)que l’on appelle un coefficient multinomial car il apparaît dans le déve-

loppement d’une somme de d termes à la puissance r

(a1 + . . . + ad)r =

x1,...,xdx1+···+xd=r

(r

x1, . . . , xd

)

ax11 · · · axd

d ,

qui généralise la formule du binôme.

Pour déterminer card(Xj = xj) on choisit xj boules parmi r, il y a donc(

rxj

)possibilités, et pour

chacun de ces choix, il y a nxj

j manières de placer les xj boules dans les nj boîtes, et il y a (n−nj)r−xj

manières de placer les r − xj autres boules parmi les n − nj autres boîtes. Donc

card(Xj = xj) =

(r

xj

)

nxj

j (n − nj)r−xj ,

et

P(Xj = xj) =

( rxj

)n

xj

j (n − nj)r−xj

nr=

(r

xj

)(nj

n

)xj(

1 − nj

n

)r−xj

.

Donc Xj suit une loi binomiale B(r, nj/n).

EI 76

3. Dans cette situation chaque boîte contient 0 ou 1 boule et, nécessairement r ≤ n. L’univers Ω estconstitué des différentes façons de choisir les r boîtes contenant une boule parmi n en tenant comptede l’ordre, donc

card Ω = Arn .

Cherchons maintenant card(X1 = x1, . . . ,Xd = xd), avec x1 ≤ n1, . . . , xd ≤ nd et x1 + · · ·+ xd = r.(Les probabilités sont nulles si une de ces relations n’est pas vérifiée). Pour la couleur c1, on choisit x1

boules parmi r, il y a donc( rx1

)possibilités, et pour chacun de ces choix, il y a Ax1

n1façons de choisir

les boîtes dans lesquelles iront ces x1 boules.

Pour la couleur c2, on choisit x2 boules parmi les r − x1 restantes. Il y a donc(r−x1x2

)possibilités, et

pour chacun de ces choix, il y a Ax2n2

façons de choisir les boîtes dans lesquelles iront ces x2 boules.

De manière générale, pour la couleur cj , on choisit xj boules parmi les r − x1 − · · · − xj−1 restantes. Ily a donc

(r−x1−···−xj−1xj

)possibilités, et pour chacun de ces choix, il y a A

xjnj façons de choisir les boîtes

dans lesquelles iront ces xj boules.

Alors

card(X1 = x1, . . . ,Xd = xd) =

d∏

j=1

[(r − x1 − · · · − xj−1

xj

)

Axjnj

]

.

et

P(X1 = x1, . . . ,Xd = xd) =

d∏

j=1

[(r − x1 − · · · − xj−1

xj

)

Axjnj

]

Arn

=r!

x1! · · · xd!

n1!

(n1 − x1)!· · · nd!

(nd − xd)!

(n − r)!

n!

=

(n1x1

)· · ·(nd

xd

)

(nr

) .

Pour déterminer card(Xj = xj) on choisit xj boules parmi r, il y a donc( rxj

)possibilités, et pour

chacun de ces choix, il y a Axjnj manières de choisir les boîtes qui les contiennent, et il y a A

r−xj

n−nj

manières de choisir parmi les n−nj autres boîtes celles qui contiennent les r − xj autres boules. Donc

card(Xj = xj) =

(r

xj

)

AxjnjA

r−xj

n−nj,

et

P(Xj = xj) =

( rxj

)A

xjnjA

r−xj

n−nj

Arn

=

(nj

xj

)(n−nj

r−xj

)

(nr

) .

Donc Xj suit une loi hypergéométrique H(r, nj , n).

EI 77

77 On lance simultanément deux pièces de monnaie (équilibrées) et on note(X,Y ) = (nombre de « piles » obtenus, nombre de « faces » obtenus).Déterminer la loi de probabilité de chacune des variables aléatoires réelles

X , Y , X + Y , X − Y ,X − Y + 2

2.

Solution

Le vecteur (X,Y ) est défini sur l’ensemble P,F×P,F et prend ses valeurs dans 0, 1, 2×0, 1, 2.On a

X = 0, Y = 2 = (F,F ) , X = 2, Y = 0 = (P,P ) ,

donc

P(X = 0, Y = 2) = P(X = 2, Y = 0) =1

4.

On a égalementX = 1, Y = 1 = (P,F ), (F,P ) ,

donc

P(X = 1, Y = 1) =1

2.

Les autres probabilités sont nulles.

On a alors X(Ω) = Y (Ω) = 0, 1, 2 et

P(X = 0) = P(Y = 2) = P(X = 0, Y = 2) = 1/4P(X = 1) = P(Y = 1) = P(X = 1, Y = 1) = 1/2P(X = 2) = P(Y = 0) = P(X = 2, Y = 0) = 1/4

On remarque que X +Y = 2, donc X +Y est une variable certaine. (Il en résulte que P(X +Y = 2) = 1et P(X + Y = k) = 0 si k 6= 2).

On a (X − Y )(Ω) = −2, 0, 2, et

P(X − Y = −2) = P(X = 0, Y = 2) = 1/4P(X − Y = 0) = P(X = 1, Y = 1) = 1/2P(X − Y = 2) = P(X = 2, Y = 0) = 1/4

On a ((X − Y + 2)/2)(Ω) = 0, 1, 2, et

P((X − Y + 2)/2 = 0) = P(X − Y = −2) = 1/4P((X − Y + 2)/2 = 1) = P(X − Y = 0) = 1/2P(X − Y + 2)/2 = 2) = P(X − Y = 2) = 1/4

On obtient la même loi que pour X, ce qui est normal puisque Y = 2 − X presque sûrement.

EI 78

78 On lance deux dés équilibrés, l’un noir et l’autre blanc, chacun ayant six faces dont deux sontmarquées d’un seul point, deux de deux points et les deux autres de trois points. On note

X = nombre de points sur le dé noir Y = nombre de points sur le dé blancR = |X − Y | S = X + YT = (X + Y )2 U = X2 + Y 2

V = min(X,Y ) W = max(X,Y )

1. Déterminer la loi de probabilité du couple aléatoire (X,Y ).

2. En déduire celle de V et W .3. Déduire la loi de (V,W ) de celle de (X,Y ) et retrouver les lois de V et W comme lois marginales.

4. Calculer E(V ) et E(W ) sans utiliser, les résultats des questions 2 et 3.5. Exprimer R et S en fonction de V et W et en déduire E(R) et E(S).

Solution

1. Si (i, j) appartient à [[ 1, 3 ]] × [[ 1, 3 ]] , les événements X = i et Y = j sont indépendants etont une probabilité égale à 1/3, donc

P(X = i, Y = j) =1

9.

2. On a

P(V = 1) = P(X = 1, Y = 1) + P(X = 1, Y = 2) + P(X = 1, Y = 3) +

P(X = 2, Y = 1) + P(X = 3, Y = 1) =5

9

puis

P(V = 2) = P(X = 2, Y = 2) + P(X = 3, Y = 2) + P(X = 2, Y = 3) =1

3

et enfin

P(V = 3) = P(X = 3, Y = 3) =1

9.

De même

P(W = 1) = P(X = 1, Y = 1) =1

9

puis

P(W = 2) = P(X = 2, Y = 2) + P(X = 1, Y = 2) + P(X = 2, Y = 1) =1

3

et enfin

P(W = 3) = P(X = 3, Y = 1) + P(X = 3, Y = 2) + P(X = 3, Y = 3) +

P(X = 2, Y = 3) + P(X = 1, Y = 3) =5

9

3. Comme on a V ≤ W , on a donc, si i > j, P(V = i,W = j) = 0.

EI 79

Supposons 1 ≤ i ≤ j ≤ 3. On a, si i 6= j,

P(V = i,W = j) = P(X = i, Y = j) + P(X = j, Y = i) =2

9,

et

P(V = i,W = i) = P(X = i, Y = i) =1

9.

La loi de (V,W ) peut se représenter par la matrice triangulaire

1/9 2/9 2/90 1/9 2/90 0 1/9

,

ce qui en sommant les lignes redonne la loi de W , et en sommant les colonnes celles de V .

1/9 2/9 2/90 1/9 2/90 0 1/9

5/93/91/9

[1/9 3/9 5/9

]

4. On a

E(V ) =∑

1≤j≤31≤i≤3

min(i, j) P(X = i, Y = j) =1

9

1≤j≤31≤i≤3

min(i, j) .

On décompose cette somme en trois morceaux suivant que i = j, i > j ou i < j.

E(V ) =1

9

3∑

i=1

i +

3∑

i=2

i−1∑

j=1

j +

3∑

j=2

j−1∑

i=1

i

,

les deux dernières sommes ont la même valeur par symétrie, et

E(V ) =1

9

(

6 +

3∑

i=2

i(i − 1)

)

=14

9.

On calcule de la même manière pour W .

E(W ) =∑

1≤j≤31≤i≤3

max(i, j) P(X = i, Y = j) =1

9

1≤j≤31≤i≤3

max(i, j) .

D’où

E(W ) =1

9

3∑

i=1

i +

3∑

i=2

i−1∑

j=1

i +

3∑

j=2

j−1∑

i=1

j

,

EI 80

les deux dernières sommes ont la même valeur par symétrie, et

E(W ) =1

9

(

6 + 23∑

i=2

i(i − 1)

)

=22

9.

5. On a R = W − V et S = W + V , d’où

E(R) = E(W ) − E(V ) =8

9et E(S) = E(W ) + E(V ) = 4 .

79 Soit n, N1, N trois entiers naturels non nuls tels que N1 ≤ N , (aj,k) une suite double de nombresréels choisis dans [ 0, 1 ] et X = (X1, . . . ,Xn) un vecteur aléatoire à valeurs dans 0, 1n tel que

PX1 = B(N1/N)(∀(j, k) ∈ N

∗ × N) P(Xj+1 = 1 | X1 + · · · + Xj = k) = aj,k .

1. Déterminer la loi de probabilité de X1 + · · · + Xn dans les deux cas suivants :

(a) aj,k =

N1

Nsi j ≥ 1 et 0 ≤ k ≤ j

0 sinon

(b) aj,k =

N1 − k

N − jsi 1 ≤ j ≤ N − 1 , 0 ≤ k ≤ N1 et 0 ≤ j − k ≤ N − N1

0 sinon

2. Retrouver ainsi les moyennes des lois B(n,N1/N) et H(n,N1, N).

Solution

Notons Zn = X1 + · · · + Xn. Comme les variables Xi ne prennent que les valeurs 0 et 1, on a

P(Zn = n) = P(Zn−1 = n − 1,Xn = 1)

= P(Xn = 1|Zn−1 = n − 1)P(Zn−1 = n − 1)

= an−1,n−1P(Zn−1 = n − 1) ,

puis

P(Zn = 0) = P(Zn−1 = 0,Xn = 0)

= P(Xn = 0|Zn−1 = 0)P(Zn−1 = 0)

= (1 − an−1,0)P(Zn−1 = 0) ,

et enfin, si 1 ≤ k ≤ n − 1

P(Zn = k) = P(Zn−1 = k − 1,Xn = 1) + P(Zn−1 = k,Xn = 0)

= P(Xn = 1|Zn−1 = k − 1)P(Zn−1 = k − 1) + P(Xn = 0|Zn−1 = k)P(Zn−1 = k)

= an−1,k−1P(Zn−1 = k − 1) + (1 − an−1,k)P(Zn−1 = k) .

EI 81

(a) Notons a = N1/N . On a donc les trois relations suivantes

P(Zn = n) = aP(Zn−1 = n − 1) (1)

P(Zn = 0) = (1 − a)P(Zn−1 = 0) (2)

P(Zn = k) = aP(Zn−1 = k − 1) + (1 − a)P(Zn−1 = k) (3)

On déduit immédiatement de (1) que

P(Zn = n) = an−1P(Z1 = 1) = an−1

P(X1 = 1) = an ,

et de (2)

P(Zn = 0) = (1 − a)n−1P(Z1 = 0) = (1 − a)n−1

P(X1 = 0) = (1 − a)n .

On montre par récurrence sur n, que, Zn suit une loi B(n, a) c’est-à-dire que pour tout k compris entre0 et n, on a

P(Zn = k) =

(n

k

)

ak(1 − a)n−k .

C’est vrai si n = 1, puisque Z1 = X1 suit une loi de Bernoulli B(a) = B(1, a).

Supposons la propriété vraie au rang n− 1, et montrons qu’elle est vraie au rang n. Alors en utilisant(3) et l’hypothèse de récurrence, on a, si k est compris entre 1 et n − 1,

P(Zn = k) = a

(n − 1

k − 1

)

ak−1(1 − a)n−k + (1 − a)

(n − 1

k

)

ak(1 − a)n−k−1

=

(n − 1

k − 1

)

ak(1 − a)n−k +

(n − 1

k

)

ak(1 − a)n−k

= ak(1 − a)n−k

((n − 1

k − 1

)

+

(n − 1

k

))

,

Mais en utilisant la formule de récurrence liant les coefficients binomiaux

P(Zn = k) = ak(1 − a)n−k

(n

k

)

.

Comme on a aussi

P(Zn = 0) = (1 − a)n et P(Zn = n) = an ,

on a donc bien montré que, pour tout k compris entre 0 et n,

P(Zn = k) =

(n

k

)

ak(1 − a)n−k .

et la propriété est vraie au rang n. Il en résulte qu’elle est vraie pour tout n ≥ 1.

EI 82

Donc Zn suit une loi B(n, a). Alors

P(Xi = 1) =

i−1∑

k=0

P(Xi = 1|Zi−1 = k)P(Zi−1 = k) ,

mais, puisque

P(Xi = 1|Zi−1 = k) = a ,

on obtient

P(Xi = 1) = a

i−1∑

k=0

P(Zi−1 = k) = a .

Les variables Xi suivent toutes des lois de Bernoulli B(a). Alors

E(Zn) =

n∑

j=1

E(Xj) =

n∑

j=1

a = na .

(b) On va montrer, là aussi par récurrence, que si 1 ≤ n ≤ N , alors Zn suit une loi H(n,N1, N).

C’est vrai si n = 1, puisque Z1 = X1 suit une loi de Bernoulli B(N1/N) = H(1, N1, N).

Supposons donc que 2 ≤ n ≤ N . Et étudions les différents cas suivants.

1 ≤ k ≤ N1, et 1 ≤ n − k ≤ N − N1

En utilisant la formule

P(Zn = k) = an−1,k−1P(Zn−1 = k − 1) + (1 − an−1,k)P(Zn−1 = k) ,

on obtient

P(Zn = k) =N1 − k + 1

N − n + 1

( N1

k−1

)(N−N1

n−k

)

( Nn−1

) +

(

1 − N1 − k

N − n + 1

) (N1

k

)(N−N1

n−1−k

)

( Nn−1

)

=N1 − k + 1

N − n + 1

( N1

k−1

)(N−N1

n−k

)

( Nn−1

) +N − n + 1 − N1 + k

N − n + 1

(N1

k

)(N−N1

n−1−k

)

( Nn−1

) .

Mais en utilisant la relation

(p − q)

(p

q

)

= (q + 1)

(p

q + 1

)

,

on en déduit

(N1 − k + 1)

(N1

k − 1

)

= k

(N1

k

)

(N − n + 1)

(N

n − 1

)

= n

(N

n

)

EI 83

(N − N1 − (n − k − 1))

(N − N1

n − k − 1

)

= (n − k)

(N − N1

n − k

)

d’où

P(Zn = k) =k(N1k

)(N−N1n−k

)

n(N

n

) +(n − k)

(N1k

)(N−N1n−k

)

n(N

n

)

=

(N1

k

)(N−N1

n−k

)

(Nn

) .

k = n

P (Zn = n) = an−1,n−1P(Zn−1 = n − 1) = an−1,n−1

( N1

n−1

)

( Nn−1

) ,

d’où l’on déduit, si n − 1 ≤ N1,

P(Zn = n) =N1 − n + 1

N − n + 1

( N1

n−1

)

(N

n−1

) =

(N1n

)

(Nn

) .

et si n − 1 > N1,

P(Zn = n) = 0 =

(N1n

)

(Nn

) .

k = 0

P(Zn = 0) = (1 − an−1,0)P(Zn−1 = 0) = (1 − an−1,0)

(N−N1n−1

)

( Nn−1

) ,

d’où l’on déduit, si n − 1 ≤ N − N1,

P(Zn = 0) =

(

1 − N1

N − n + 1

) (N−N1n−1

)

( Nn−1

)

=N − N1 − n + 1

N − n + 1

(N−N1

n−1

)

(N

n−1

)

=

(N−N1

n−N1

)

(Nn

) ,

et si n − 1 > N − N1,

P(Zn = 0) = 0 =

(N−N1

n−N1

)

(Nn

) .

EI 84

Finalement, pour tout k entre 0 et N1, tel que n − k soit compris entre 0 et N − N1 on a obtenu

P(Zn = k) =

(N1k

)(N−N1n−k

)

(Nn

) .

La variable Zn suit bien une loi H(n,N1, N), et la propriété est vraie au rang n. Elle est donc vraiepour tout n ≤ N .

Calculons maintenant P(Xi = 1). On a toujours

P(Xi = 1) =

i−1∑

k=0

P(Xi = 1|Zi−1 = k)P(Zi−1 = k) ,

donc

P(Xi = 1) =

i−1∑

k=0

N1 − k

N − i + 1

(N1k

)(N−N1i−1−k

)

( Ni−1

) .

Mais

(N1 − k)

(N1

k

)

= N1

(N1 − 1

k

)

et (N − (i − 1))

(N

i − 1

)

= N

(N − 1

i − 1

)

,

donc

P(Xi = 1) =

i−1∑

k=0

N1

N

(N1−1k

)(N−N1

i−1−k

)

(N−1i−1

) =N1

N

i−1∑

k=0

(N1−1k

)(N−N1

i−1−k

)

(N−1i−1

) .

Mais la somme qui apparaît est celle des coefficients d’une loi H(i−1, N1, N −1) et vaut 1. Finalement

P(Xi = 1) =N1

N.

Les variables Xi suivent toutes des lois de Bernoulli B(N1/N). Alors

E(Zn) =

n∑

j=1

E(Xj) =

n∑

j=1

N1

N=

nN1

N.

80 Soit (X,Y ) un couple de variables indépendantes de même loi U(z1, . . . , zn) où les zi sont nréels distincts. Déterminer P(X = Y ) et P(X ≥ Y ).

Solution

Puisque les variables sont indépendantes et de même loi uniforme,

P(X = zi, Y = zj) = P(X = zi)P(Y = zj) =1

n2.

On peut supposer que les nombres z1, . . . , zn sont ordonnés en croissant. On a alors

EI 85

P(X = Y ) =

n∑

k=1

P(X = zk, Y = zk) =

n∑

k=1

1

n2=

1

n,

puis

P(X ≥ Y ) =∑

1≤j≤i≤n

P(X = zi, Y = zj) =n∑

i=1

i∑

j=1

1

n2=

n∑

i=1

i

n2=

n(n + 1)

2n2=

n + 1

2n.

81 Soit (xi, pi) | i ∈ [[ 1, n ]] le germe d’une loi de probabilité V. Montrer que si (X,Y ) est uncouple de variables indépendants de loi V, alors

P(X 6= Y ) =

n∑

i=1

pi(1 − pi) .

Solution

Puisque les variables sont indépendantes et de même loi, on a

P(X = xi, Y = xj) = P(X = xi)P(Y = xj) = pipj .

Alors

P(X 6= Y ) = 1 − P(X = Y ) = 1 −n∑

i=1

P(X = xi, Y = xi) = 1 −n∑

i=1

p2i .

Mais

1 =

n∑

i=1

pi ,

donc

P(X 6= Y ) =n∑

i=1

pi −n∑

i=1

p2i =

n∑

i=1

pi(1 − pi) .

82 Peut-on truquer un dé (à six faces, marquées 1,2,. . . ,6) de façon que la somme des points obtenusen deux lancers de ce dé suive une loi uniforme ?

Solution

Il s’agit de voir si l’on peut trouver 6 nombres positifs pi pour 1 ≤ i ≤ 6 de somme 1, tels que, si Xet Y suivent la loi V de germe (i, pi) | i ∈ [[ 1, n ]] et sont indépendantes, alors X + Y suit une loiuniforme. On a (X + Y )(Ω) = [[ 2, 12 ]] et l’on voudrait que, si 2 ≤ k ≤ 12,

P(X + Y = k) =1

11.

EI 86

On a de plus

P(X = i, Y = j) = P(X = i)P(Y = j) = pipj .

En calculant P(X + Y = k), on obtient un système (non linéaire) de 11 équations à 6 inconnues.Ecrivons seulement les premières et les dernières.

P(X + Y = 2) = p21

P(X + Y = 3) = 2p1p2

P(X + Y = 4) = 2p1p3 + p22

P(X + Y = 10) = 2p4p6 + p25

P(X + Y = 11) = 2p5p6

P(X + Y = 12) = p26

En raison de la symétrie du problème, on aura pk = p7−k. On trouve successivement

p1 = p6 =1√11

,

p2 = p5 =1

2p1

1

11=

1

2√

11,

p3 = p4 =1

2p1

(1

11− p2

2

)

=

√11

2

(1

11− 1

44

)

=3

8√

11.

Mais alors6∑

k=1

pi = 2(p1 + p2 + p3) =15

4√

116= 1 ,

et donc le problème est impossible.

83 Conjecturer à l’aide d’une expérience aléatoire bien choisie, puis vérifier dans le cas général que

X ⊥⊥ X ′

PX = B(n, a)PX′ = B(n′, a)

⇒ PX+X′ = B(n + n′, a) .

Solution

On fait une expérience ne présentant que deux possibilités : réussite avec probabilité a et échec avecprobabilité 1 − a, la variable X compte le nombre de réussites sur n expériences et la variable X ′ lenombre de réussites sur n′ expériences. Alors la variables X + X ′ donne le nombre de réussites surn + n′ expériences. (Les expériences étant faites de manière indépendante).

EI 87

On a (X + X ′)(Ω) = [[ 0, n + n′ ]] . Si k appartient à [[ 0, n + n′ ]] , on cherche P(X + X ′ = k). On a

P(X + X ′ = k) =k∑

j=0

P(X = j,X ′ = k − j)

=k∑

j=0

P(X = j)P(X ′ = k − j)

=

k∑

j=0

(n

j

)

aj(1 − a)n−j

(n′

k − j

)

ak−j(1 − a)n′−k+j

= ak(1 − a)n+n′−kk∑

j=0

(n

j

)(n′

k − j

)

.

Maisk∑

j=0

(n

j

)(n′

k − j

)

=

(n + n′

k

)

,

donc

P(X + X ′ = k) =

(n + n′

k

)

ak(1 − a)n+n′−k .

Pour démontrer l’égaliték∑

j=0

(n

j

)(n′

k − j

)

=

(n + n′

k

)

,

on peut introduire le polynôme

(X + 1)n+n′

= (X + 1)n(X + 1)n′

,

et développer par la formule du binôme.

n+n′∑

k=0

(n + n′

k

)

Xk =

n∑

j=0

(n

j

)

Xj

(n∑

i=0

(n′

i

)

Xi

)

.

Si l’on cherche le coefficient de Xk, on a alors

(n + n′

k

)

=∑

i+j=k

(n

j

)(n′

i

)

=k∑

j=0

(n

j

)(n′

k − j

)

.

84 Soit (xi, yj), (xk, yℓ) deux valeurs probables (distinctes) d’un couple aléatoire (X,Y ). Montrerqu’il suffit que P((X,Y ) = (xi, yℓ)) = 0 ou P((X,Y ) = (xk, yj)) = 0 pour que X et Y ne soient pasindépendantes.

EI 88

Solution

Par hypothèse P((X,Y ) = (xi, yj)) et P((X,Y ) = (xk, yℓ)) ne sont pas nuls. Supposons que P((X,Y ) =(xi, yℓ)) = 0 et que X et Y sont indépendantes. Alors

P((X,Y ) = (xi, yℓ)) = P(X = xi)P(Y = yℓ) = 0 ,

et un des deux nombres P(X = xi), P(Y = yℓ) est nul. Mais alors

P((X,Y ) = (xi, yj)) = P(X = xi)P(Y = yj)

et

P((X,Y ) = (xk, yℓ)) = P(X = xk)P(Y = yℓ) ,

et un de ces deux nombres est nul, d’où une contradiction. Donc X et Y ne sont pas indépendantes.

Le raisonnement est analogue si c’est P((X,Y ) = (xk, yj)) qui est nul.

85 1. Quelle est la loi de probabilité de la variable aléatoire Y =n∑

i=1

10−iXi si (X1, . . . ,Xn) est un

n−uplet de variables aléatoires indépendantes suivant une loi U( [[ 0, n ]] ) ?

2. Soit n et r deux entiers naturels non nuls, z1, . . . , zr, des nombres réels tous distincts et p1, . . . , pr

des nombres rationnels de l’intervalle ] 0, 1 [ tels que

∀i ∈ [[ 1, r ]] , 10npi ∈ N

r∑

i=1

pi = 1

Déterminer la loi de probabilité de la variable aléatoire réelle Z définie sur le même univers Ω que lesXi par :

a) Z(ω) = z1 ⇔ Y (ω) < p1

b) ∀k ∈ [[ 2, r ]] Z(ω) = zk ⇔k−1∑

i=1

pi ≤ Y (ω) <

k∑

i=1

pi

Solution

1. Si l’on écrit

Y =

n∑

i=1

10−iXi =

n∑

i=1

10n−iXi

10n,

EI 89

la sommen∑

i=1

10n−iXi donne tous les nombres compris entre 0 et 10n − 1, (écriture décimale d’un

nombre entier), et tous ces nombres sont distincts.

DoncD = Y (Ω) = p10−n | 0 ≤ p ≤ 10n − 1 .

En particulier card(Ω) = 10n. L’ensemble D est l’ensemble des nombres décimaux ayant au plus 10chiffres après la virgule.

Si r est un nombre de [[ 0, 10n − 1 ]] , il s’écrit de manière unique

r =

n∑

i=1

10n−iri ,

où ri appartient à [[ 0, 9 ]] , donc

P(Y = p10−n) = P((X1, . . . ,Xn) = (r1, . . . , rn)) =n∏

i=1

P(Xi = ri) ,

et comme les lois sont uniformes, on a pour tout i,

P(Xi = ri) =1

10,

donc

P(Y = r10−n) =1

10n.

La variable Y suit un loi uniforme U(D).

2. Si pi = ri10−n, avec ri ∈ [[ 0, 10n − 1 ]] , on a

P(Z = z1) = P(Y < p1) =card( [ 0, p1 [∩D)

card D.

Maiscard( [ 0, p1 [∩D) = card( [ 0, r1 [ ) = r1 ,

et doncP(Z = z1) =

r1

10n= p1 .

De même

P(Z = zr) = P

(k−1∑

i=1

pi ≤ Y <

k∑

i=1

pi

)

=

card

([k−1∑

i=1

pi,k∑

i=1

pi

[

∩ D

)

card D.

Mais

card

([k−1∑

i=1

pi,

k∑

i=1

pi

[

∩ D

)

= card

([k−1∑

i=1

ri,

k∑

i=1

ri

[)

= rk ,

EI 90

et donc

P(Z = zr) =rk

10n= pk .

Donc Z suit une loi de probabilité de germe (zk, pk) | k ∈ [[ 1, r ]] .

86 Soit (X,Y) un couple de variables aléatoires réelles et (a, b, c, d) ∈ R4.

1. Exprimer Cov(aX + b, cY + d) en fonction de Cov(X,Y ).

Solution

On a

aX + b − E(aX + b) = a(X − E(X)) et cY + d − E(cY + d) = c(Y − E(Y )) ,

donc

Cov(aX + b, cY + d) = E((aX + b − E(aX + b))(cY + d − E(cY + d)))

= E(ac(X − E(X))(Y − E(Y )))

= acE((X − E(X))(Y − E(Y )))

= ac Cov(X,Y ) .

87 Soit X une variable aléatoire réelle à valeurs positives ou nulles et h une fonction croissante de] 0, +∞ [ dans ] 0, +∞ [ . Etablir l’inégalité de Markov : pour tout ε > 0

P(X ≥ ε) ≤ E(h(X))

h(ε).

Solution

Notons X(Ω) = x1, . . . , xn. On a alors

E(h(X)) =

n∑

i=1

h(xi)P(X = xi)

=∑

i | xi≥ε

h(xi)P(X = xi) +∑

i |xi<ε

h(xi)P(X = xi) .

Mais comme la deuxième somme est positive, on a

E(h(X)) ≥∑

i |xi≥ε

h(xi)P(X = xi) .

Comme la fonction h est croissante, si xi ≥ ε, on en déduit h(xi) ≥ h(ε), et donc

EI 91

E(h(X)) ≥∑

i |xi≥ε

h(ε)P(X = xi) = h(ε)∑

i |xi≥ε

P(X = xi) .

Mais∑

i |xi≥ε

P(X = xi) = P(X ≥ ε) ,

d’où

E(h(X)) ≥ h(ε)P(X ≥ ε) ,

ce qui donne l’inégalité voulue.

88 (suite de l’exercice 78)

6. Vérifier que les variables (R,S) ne sont pas indépendantes mais que leur coefficient de corrélationest nul cependant.

7. Représenter par un nuage de points la loi de probabilité du couple (T,U).

8. Doit-on s’attendre à une forte corrélation linéaire entre T et U ?

9. Déterminer une équation cartésienne de la droite de régression ∆U/T de U en T et la représentersur le diagramme de la question 7.

10. Déterminer le coefficient de corrélation linéaire r T,U .

La valeur obtenue est-elle en accord avec la réponse à la question 8 de l’exercice 18 ?

Solution

6. L’événement R = 0, S = 3 = X = Y,X + Y = 3 = X = Y = 3/2 est impossible. Donc

P(R = 0, S = 3) = 0 .

Mais

R = 0 = (1, 1), (2, 2), (3, 3) et S = 3 = (1, 2), (2, 1) .

Les probabilités P(R = 0) et P(S = 3) ne sont donc pas nulles et on n’a pas l’égalité

P(R = 0, S = 3) = P(R = 0)P(S = 3) ,

ce qui montre que R et S ne sont pas indépendantes.

On a RS = |X2 − Y 2| et cette variable aléatoire ne peut prendre que les valeurs 0, 3, 5, 8. On obtient

RS = 3 = (2, 1), (1, 2) , RS = 5 = (2, 3), (3, 2) , RS = 8 = (1, 3), (3, 1) ,

EI 92

donc

P(RS = 3) = P(RS = 5) = P(RS = 8) =2

9.

Alors

E(XY ) = (3 + 5 + 8)2

9=

32

9=

8

9× 4 = E(R)E(S) .

Le coefficient de corrélation de R et S sera donc nul.

7. Calculons les probabilités P(T = i, U = j). On a le tableau

(i, j) T = i, U = j P(T = i, U = j)

(4, 2) (1, 1) 1/9

(9, 5) (1, 2), (2, 1) 2/9

(16, 10) (1, 3), (3, 1) 2/9

(16, 8) (2, 2) 1/9

(25, 13) (2, 3), (3, 2) 2/9

(36, 18) (3, 3) 1/9

d’où l’ensemble de points, auquel on a ajouté la droite de régression de la question 9. :

-

6

t

u

8. On doit s’attendre à une forte corrélation entre T et U car le nuage de points est très effilé.

9. L’équation de la droite de régression ∆U/T est

u =Cov(T,U)

V(T )t +

(

E(U) − Cov(T,U)

V(T )E(T )

)

.

EI 93

On part des germes des variables T et U .

i 4 9 16 25 36

P(T = i) 1/9 2/9 3/9 2/9 1/9

j 2 5 8 10 13 18

P(U = j) 1/9 2/9 1/9 2/9 2/9 1/9

On obtient successivement

E(T ) =52

3, E(T 2) = 388 , V(T ) =

788

9.

puis

E(U) =28

3.

Enfin

E(TU) =1844

9, E(T )E(U) =

1456

9, Cov(T,U) =

388

9,

donc l’équation de la droite de régression de U par rapport à T est

u =97

197t +

472

591.

avec comme valeur approchéeu = 0, 5 t + 0, 8 .

10. On a

rT,U =Cov(T,U)

σUσT.

On a déjà obtenu

Cov(T,U) =388

9, V(T ) =

788

9, E(U) =

28

3.

On obtient aussi

E(U2) =980

9, V(U) =

196

9.

Donc

r T,U =3889

√7889

√1969

=97

1379

√197 ≈ 0, 987 .

Ce coefficient est proche de 1, ce qui montre que les variables U et T sont très liées comme l’indiquaitle nuage de points.

89 Soit (X,Y ) un couple de variables aléatoires réelles non dégénérées.Montrer que la mesure de l’angle aigu que font entre elles les deux droites de régression peut servir demesure de la corrélation linéaire entre X et Y .

EI 94

Solution

Les équations des droites de régression sont données par

∆Y/X : y =Cov(X,Y )

σ2X

x +

(

mY − Cov(X,Y )

σ2X

mX

)

∆X/Y : x =Cov(X,Y )

σ2Y

y +

(

mX − Cov(X,Y )

σ2Y

mY

)

donc si l’on exprime y en fonction de x pour la deuxième équation lorsque Cov(X,Y ) 6= 0, on obtient

y =σ2

Y

Cov(X,Y )x −

(

mXσ2

Y

Cov(X,Y )− mY

)

.

La tangente de l’angle que fait une droite avec Ox est le coefficient directeur de cette droite (en axesorthonormés), donc, pour la première droite

tan α =Cov(X,Y )

σ2X

,

et pour la seconde

tan β =σ2

Y

Cov(X,Y ).

Alors la tangente de l’angle formé par les deux droites vaut

tan(α − β) =tan α − tan β

1 + tan α tan β

=

Cov(X,Y )

σ2X

− σ2Y

Cov(X,Y )

1 +σ2

Y

σ2X

=Cov(X,Y )2 − σ2

Xσ2Y

Cov(X,Y )(σ2X + σ2

Y ),

et puisque

Cov(X,Y ) = rX,Y σXσY ,

on obtient finalement

tan(α − β) =σXσY

σ2X + σ2

Y

r2X,Y − 1

rX,Y=

σXσY

σ2X + σ2

Y

(

rX,Y − 1

rX,Y

)

.

L’angle aigu formé par les deux droites a une mesure (appartenant à l’intervalle [ 0, π/2 ] ) donnée par

θX,Y = arctan

∣∣∣∣

σXσY

σ2X + σ2

Y

(

rX,Y − 1

rX,Y

)∣∣∣∣

.

EI 95

On remarque que

0 ≤ σXσY

σ2X + σ2

Y

≤ 1

2,

ce qui résulte du fait que (σX − σY )2 ≥ 0. Ce quotient est donc borné, et θX,Y se comporte comme

rX,Y − 1

rX,Y.

Etudions les variations de la fonction f définie sur ] 0, 1 [ par

f(r) = r − 1/r .

On a

f ′(r) = 1 + 1/r2 ,

Donc f est strictement croissante et varie de −∞ à 0. Il en résulte que θX,Y est nul si et seulement sir2X,Y − 1 = 0, c’est-à-dire si et seulement si X et Y sont liés par une relation affine.

Si rX,Y est nul, alors ∆Y/X est parallèle à Ox, et ∆X/Y est parallèle à Oy. Donc θX,Y = π/2, ce quiest en particulier le cas lorsque X et Y sont indépendantes.

L’angle θX,Y se comporte comme r2X,Y − 1 et donne donc une autre façon de mesurer la corrélationentre X et Y .

90 Suite de l’exercice 86

2. On suppose que σXσY 6= 0 et ac 6= 0. Exprimer raX+b,cY +d en fonction de rX,Y .

Solution

On a

Cov(aX + b, cY + d) = ac Cov(X,Y ) , V(aX + b) = a2V(X) , V(cY + d) = c2

V(Y ) ,

donc

raX+b,cY +d =ac Cov(X,Y )

|a|σX |c|σY= sign(ac) rX,Y .

91 Déterminer la loi de probabilité d’un couple (X,Y ) de variables aléatoires suivant des lois deBernoulli de même paramètre a et telles que rX,Y = r, où r est dans [−1, 1 ] .

Solution

Notons u = P(X = 1, Y = 1). La variable XY suit aussi une loi de Bernoulli et

P(XY = 1) = P(X = 1, Y = 1) = u ,

EI 96

donc c’est une loi B(u). AlorsE(XY ) = u .

CommeE(X) = E(Y ) = a ,

on aCov(X,Y ) = u − a2 .

On a égalementV(X) = V(Y ) = a(1 − a) ,

donc

rX,Y =u − a2

a(1 − a)= r ,

et l’on en déduitu = ra(1 − a) + a2 .

AlorsP(X = 1, Y = 1) + P(X = 1, Y = 0) = P(X = 1) = a ,

doncP(X = 1, Y = 0) = a − ra(1 − a) − a2 = a(1 − a)(1 − r) ,

et de même par symétrie du problème

P(X = 0, Y = 1) = a(1 − a)(1 − r) .

Enfin

P(X = 0, Y = 0) = P(X = 0) − P(X = 0, Y = 1)

= 1 − a − a(1 − a)(1 − r)

= (1 − a)(1 − a(1 − r))

= ra(1 − a) + (1 − a)2 .

EI 97

IV - Probabilité sur un ensemble dénombrable

92 Soit Ω un univers dénombrable et P une application de P(Ω) dans R+.Montrer que P est σ−additive si et seulement si

(∀A ∈ P(Ω)) P(A) =∑

ω∈A

P(ω) .

Solution

Supposons que P est σ−additive. Si A n’est pas vide, on a

A =⊎

ω∈A

ω ,

et on on aura doncP(A) =

ω∈A

P(ω) .

Réciproquement, supposons que

(∀A ∈ P(Ω)) P(A) =∑

ω∈A

P(ω) ,

et soit (An)n∈N une suite de parties de Ω deux à deux disjointes. On a alors

n∈N

An =⊎

n∈N

ω∈An

ω .

Donc

P

(⊎

n∈N

An

)

=∑

n∈N

ω∈An

P(ω) .

Mais, pour tout entier n,∑

ω∈An

P(ω) = P(An) ,

donc

P

(⊎

n∈N

An

)

=∑

n∈N

P(An) ,

ce qui montre que P est σ−additive.

93 Soit Ω un univers dénombrable et P une application additive de P(Ω) dans R+, telle que P(Ω) = 1.Montrer que P est une probabilité si et seulement si, pour toute suite croissante (An)n∈N d’événements,

P

(⋃

n∈N

An

)

= limn→∞

P(An) .

EI 98

Solution

Si P est additive, c’est-à-dire si quels que soient A et B dans P(Ω) incompatibles,

P(A ⊎ B) = P(A) + P(B) ,

alors on déduit par récurrence que, si (A0, . . . , Ap) sont deux à deux disjoints,

P

(p⊎

n=0

An

)

=

p∑

n=0

P(An) .

C’est vrai de manière évidente si p = 0. Si la propriété est vraie au rang p alors

P

(p+1⊎

n=0

An

)

= P

([p⊎

n=0

An

]⊎

Ap+1

)

= P

(p⊎

n=0

An

)

+ P(Ap+1)

Mais l’hypothèse de récurrence donne

P

(p⊎

n=0

An

)

=

p∑

n=0

P(An) ,

donc

P

(p+1⊎

n=0

An

)

=

p+1∑

n=0

P(An) ,

et la propriété est vraie au rang p + 1 donc pour tout p ≥ 0.

Supposons que, pour toute suite croissante (An)n∈N de parties de Ω, on ait

P

(⋃

n∈N

An

)

= limn→∞

P(An) .

Si (An)n∈N est une suite de parties deux à deux disjointes de Ω, posons

Bp =

p⊎

n=0

An .

On obtient une suite croissante et∞⋃

n=0

An =

∞⋃

p=0

Bp .

Alors,

P

(∞⋃

n=0

An

)

= P

∞⋃

p=0

Bp

= limp→+∞

P(Bp) .

EI 99

Mais

P(Bp) =

p∑

n=0

P(An) ,

donc

P

(∞⋃

n=0

An

)

= limp→+∞

p∑

n=0

P(An) =∞∑

n=0

P(An) ,

et P est σ−additive. Ce sera donc une probabilité.

Inversement si P est une probabilité, et si (An)n∈N est une suite croissante de parties de Ω, alors posons,B0 = A0, et si n ≥ 1, Bn = An \ An−1. Les ensembles Bn sont alors deux à deux disjoints et

n∈N

An =⊎

n∈N

Bn .

Donc

P

(⋃

n∈N

An

)

=∑

n∈N

P(Bn) .

Mais, on a P(A0) = P(B0), et puisque An−1 ⊂ An, on a, si n ≥ 1

P(Bn) = P(An) − P(An−1) ,

et donc on obtient une série télescopique, d’où

P

(⋃

n∈N

An

)

=

∞∑

n=1

(P(An) − P(An−1)) + P(A0) = limn→+∞

P(An) .

94 Montrer que pour toute suite (En)n∈N d’événements aléatoires d’un même espace probabilisé(Ω, P),

P

(⋃

n∈N

En

)

≤∞∑

n=0

P(En) .

Solution

Montrons tout d’abord par récurrence que, pour tout entier p ≥ 0,

P

(p⋃

n=0

En

)

≤p∑

n=0

P(En) .

EI 100

C’est vrai si p = 0 car on a alors égalité. Supposons la formule vraie au rang p. Alors

P

(p+1⋃

n=0

En

)

= P

([p⋃

n=0

En

]⋃

Ep+1

)

= P

(p⋃

n=0

En

)

+ P(Ep+1) − P

([p⋃

n=0

En

]⋂

Ep+1

)

≤ P

(p⋃

n=0

En

)

+ P(Ep+1) .

Mais l’hypothèse de récurrence donne

P

(p⋃

n=0

En

)

≤p∑

n=0

P(En) ,

donc

P

(p+1⋃

n=0

En

)

≤p+1∑

n=0

P(En) ,

et la propriété est vraie au rang p + 1 donc pour tout p ≥ 0.

Comme la suite

(p⋃

n=0

En

)

p∈N

est une suite croissante, on a

limp→+∞

P

(p⋃

n=0

En

)

= P

(∞⋃

n=0

En

)

,

et comme la suite (P(En))n≥0 est positive, la suite

(p∑

n=0

P(En)

)

p≥0

est croissante et possède une limite

finie au non. Alors par passage à la limite dans les inégalités, on obtient, lorsque p tend vers l’infini

P

(∞⋃

n=0

En

)

≤∞∑

n=0

P(En) .

95 Montrer qu’il ne peut exister d’équiprobabilité sur un univers infini dénombrable.

Solution

Soit Ω = ω1, ω2, . . .. S’il existait une équiprobabilité sur Ω, il existerait un nombre a non nul, telque, pour tout entier i,

P(ωi) = a ,

EI 101

mais la série de terme général P(ωi) devrait converger, ce qui n’est possible que si a = 0. Donc iln’existe pas d’équiprobabilité sur Ω.

96 Montrer qu’à toute suite décroissante (an)n∈N de réels strictement positifs tels que limn→+∞

an = 0,

on peut associer une unique loi de probabilité P sur N telle que

(∀n ∈ N) P(n, n + 1, . . .) =an

a0.

Solution

Si une telle loi existe, on a alors, si n ≥ 0

P(n) = P(n, n + 1, . . .) − P(n + 1, . . .) =an − an+1

a0,

ce qui est un nombre positif. Alors

∞∑

n=0

P(n) =1

a0

∞∑

n=0

(an − an+1) ,

et on a une série télescopique donc

∞∑

n=0

P(n) =1

a0(a0 − lim

n→∞an) = 1 .

On a donc bien obtenu une loi de probabilité sur N, et c’est la seule possible.

Inversement si une loi de probabilité sur N est définie par

P(n) =an − an+1

a0,

on a bien

P(n, n + 1, . . .) =

∞∑

k=n

ak − ak+1

a0=

1

a0(an − lim

k→∞ak) =

an

a0.

97 1. Montrer que l’ensemble (0, 0)∪(i, 3π2i2

) | i ∈ Z∗ est le germe d’une loi de probabilité V sur Z.

2. Une variable aléatoire de loi V possède-t-elle une espérance mathématique ?

Solution

1. On sait que∞∑

n=1

1

n2=

π2

6.

EI 102

Alors∑

i∈Z∗

1

i2= 2

∞∑

n=1

1

n2=

π2

3,

et donc∑

i∈Z∗

3

π2i2= 1 ,

ce qui montre que l’on a bien le germe d’une probabilité.

2. Soit X une variable aléatoire de loi V. Comme la série de terme général 1/n diverge, il en résulte

que la somme∑

i∈Z∗

|i|P(X = i) est infinie, et donc que X n’a pas d’espérance.

Remarque : on a pour tout nn∑

i=−n

iP(X = i) = 0 ,

mais cela ne suffit pas pour dire que l’espérance est nulle. En fait elle n’existe pas.

98 Montrer que si une variable aléatoire réelle discrète X prend toutes ses valeurs dans un intervallefermé borné I de R, elle possède une espérance mathématique et E(X) appartient à I.

Solution

Si X(Ω) = a1, a2, . . . , an, . . ., et I = [m, M ] , on a donc, pour tout entier n,

m ≤ an ≤ M ,

et en particulier|an| ≤ K = max(|m|, |M |) ,

donc|an|P(X = an) ≤ K P(X = an) ,

et comme la série de terme général P(X = an) converge (sa somme vaut 1), la série de terme généralanP(X = an) converge absolument. Il en résulte que la variable aléatoire X a une espérance.

Alors, puisque l’on a pour tout n ≥ 0,

m P(X = an) ≤ anP(X = an) ≤ M P(X = an) ,

on obtient en sommant

m

∞∑

n=1

P(X = an) ≤∞∑

n=1

anP(X = an) ≤ M

∞∑

n=1

P(X = an) ,

c’est-à-direm ≤ E(X) ≤ M ,

EI 103

ce qui montre que E(X) est dans I.

99 1. Montrer que pour tout entier naturel non nul k, il existe un réel αk tel que l’ensemble(j, αk

1+j2k | j ∈ Z soit le germe d’une loi de probabilité W.

2. Etudier, selon les valeurs de k, l’existence de l’espérance mathématique et de la variance d’unevariable aléatoire réelle X de loi W.

Solution

La série de terme général 1/(1 + n2k) converge puisque

1

1 + n2k∼ 1

n2k,

et que la série de Riemann de terme général 1/n2k converge si 2k > 1, donc si k ≥ 1. Alors, si l’on pose

Sk =

∞∑

n=1

1

1 + n2k,

on a∑

j∈Z

1

1 + j2k= 1 + 2

∞∑

n=1

1

1 + n2k= 1 + 2Sk > 0 .

Par suite, si l’on poseαk = 1/(1 + 2Sk)

on a ∑

j∈Z

αk

1 + j2k= 1 .

2. L’espérance de X existe si et seulement si la somme∑

j∈Z

|j|P(X = j) est finie ou encore, si et

seulement si la série de terme généraln

1 + n2kconverge absolument. Mais

n

1 + n2k∼ 1

n2k−1,

et la série converge si et seulement si 2k − 1 > 1, soit k ≥ 2.

La variance de X existe si et seulement si l’espérance de X et celle de X2 existent. Celle de X2 existe

si et seulement si la somme∑

j∈Z

j2P(X = j) est finie ou encore, si et seulement si la série de terme

généraln2

1 + n2kconverge absolument. Mais

n2

1 + n2k∼ 1

n2k−2,

EI 104

et la série converge si et seulement si 2k − 2 > 1, soit encore k ≥ 2.

Donc si k = 1 l’espérance n’existe pas, et si k = 2, espérance et variance existent.

100 1. Montrer qu’une variable aléatoire X à valeurs dans N possède une espérance mathématique

si et seulement si∞∑

n=1

P(X ≥ n) < +∞ et que, si c’est le cas,

E(X) =

∞∑

n=1

P(X ≥ n) .

2. En déduire un calcul de la moyenne de la loi géométrique B−(a).

Solution

1. Puisque

P(X ≥ n) =

∞∑

p=n

P(X = p) ,

on a donc∞∑

n=1

P(X ≥ n) =

∞∑

n=1

∞∑

p=n

P(X = p) .

Comme tous les termes de cette somme sont positifs, on peut intervertir les sommations et

∞∑

n=1

P(X ≥ n) =∞∑

p=1

p∑

n=1

P(X = p) =∞∑

p=1

p P(X = p) .

On constate donc que∞∑

n=1

P(X ≥ n) est finie si et seulement si∞∑

p=1

p P(X = p) est finie, c’est-à-dire si

et seulement si X possède une espérance. Et dans ce cas les deux nombres sont égaux, donc

E(X) =

∞∑

n=1

P(X ≥ n) .

2. La loi B−(a) a pour germe (k, (1 − a)k−1a) | k ∈ N∗. On cherche donc à calculer

E(X) =

∞∑

k=1

P(X ≥ n) .

Or en utilisant les séries géométriques

P(X ≥ n) =

∞∑

j=n

a(1 − a)j−1 = a(1 − a)n−1 1

1 − (1 − a)= (1 − a)n−1 ,

EI 105

puis

E(X) =

∞∑

k=1

(1 − a)n−1 =1

1 − (1 − a)=

1

a.

101 On décide de jouer à pile ou face jusqu’à ce qu’on obtienne un résultat différent du précédent.1. Quel est le « nombre moyen » de lancers à effectuer ?2. Retrouver le résultat en utilisant les fonctions génératrices.

Solution

1. Notons X la variable aléatoire donnant le nombre de lancers effectués.

Si l’on s’est arrêté au n−ième lancé (n ≥ 2), c’est que l’on a obtenu une des deux situations suivantes

P1P2 . . . Pn−1Fn ou F1F2 . . . Fn−1Pn .

La probabilité de chacun de ces événements est de 1/2n, donc, si n ≥ 2, on a

P(X = n) =1

2n−1.

Mais alors X − 1 prend ses valeurs dans N∗ et

P(X − 1 = n) = P(X = n + 1) =1

2n=

(

1 − 1

2

)n−1 1

2.

Donc X − 1 suit une loi B−(a) avec a = 1/2 et son espérance vaut 1/a = 2. Donc

E(X) = 3 .

2. Posons

g(s) = E(sX) =∞∑

n=2

sn

2n−1.

On a

g(s) = s∞∑

n=2

(s

2

)n−1=

s2

2

1

1 − s2

=s2

2 − s,

donc

g′(s) =2s(2 − s) + s2

(2 − s)2=

4s − s2

(2 − s)2.

AlorsE(X) = g′(1) = 3 .

EI 106

102 Soit n ∈ N∗ et a ∈ ] 0, 1 [ .

1. Développer en série entière au voisinage de 0 la fonction s 7→[

as

1 − (1 − a)s

]n

.

2. En déduire que l’ensemble (k,(k−1n−1

)an(1 − a)k−n | k ∈ n, n + 1, . . . est le germe d’une loi de

probabilité sur l’ensemble des entiers supérieurs ou égaux à n. On note cette loi B−(n, a).

3. Si n ≥ 2, étudier l’existence et déterminer éventuellement la valeur de l’espérance mathématique den−1X−1 si X est une variable aléatoire qui suit la loi B−(n, a).

4. Soit E une expérience aléatoire qui ne peut que « réussir » avec la probabilité a ou « échouer » avecla probabilité 1− a. On décide de faire des répétitions indépendantes de E jusqu’à ce qu’on ait obtenun réussites.

Montrer que la variable aléatoire : X = « nombre d’essais nécessaires » suit la loi B−(n, a). (Noterqu’en particulier B−(a) = B−(1, a)).

5. (a) Conjecturer, à l’aide d’une expérience aléatoire bien choisie que

X ⊥⊥ X ′

PX = B−(n, a)PX′ = B−(n′, a)

⇒ PX+X′ = B−(n + n′, a) .

(b) En utilisant la relation suivante, vraie si 1 ≤ n ≤ m, (voir exercice 2)

(m

n

)

=

m−1∑

i=n−1

(i

n − 1

)

,

montrer queX ⊥⊥ YPX = B−(n, a)PY = B−(a)

⇒ PX+Y = B−(n + 1, a) ,

et en déduire la preuve de la conjecture précédente.

6. Montrer que si (X1, . . . ,Xn) est un n−uplet de variables aléatoires indépendantes de même loi B−(a)alors

PX1+···+Xn = B−(n, a) ,

et en déduire la moyenne et la variance de la loi B−(n, a).

7. Déterminer la fonction génératrice de la loi B−(n, a) et retrouver ainsi les résultats obtenus aux deuxitems précédents.

Solution

1. En utilisant le développement de Taylor, on a, pour |u| < 1,

EI 107

(1 − u)−n = 1 +

∞∑

j=1

−n(−n − 1) · · · (−n − j + 1)

j!(−u)j

= 1 +

∞∑

j=1

n(n + 1) · · · (n + j − 1)

j!uj

= 1 +∞∑

j=1

(n + j − 1

n − 1

)

uj

=

∞∑

j=0

(n + j − 1

n − 1

)

uj .

Alors [as

1 − (1 − a)s

]n

= ansn(1 − (1 − a)s)−n ,

et d’après le calcul précédent,

[as

1 − (1 − a)s

]n

=

∞∑

j=0

(n + j − 1

n − 1

)

ansn+j(1 − a)j .

En effectuant le changement d’indice k = n + j, on obtient le développement en série entière cherché :

[as

1 − (1 − a)s

]n

=

∞∑

k=n

(k − 1

n − 1

)

ansk(1 − a)k−n .

2. En particulier, si s = 1,

1 =

∞∑

k=n

(k − 1

n − 1

)

an(1 − a)k−n ,

ce qui montre que l’on a bien le germe d’une loi de probabilité.

3. On a

E

(n − 1

X − 1

)

=

∞∑

k=n

n − 1

k − 1

(k − 1

n − 1

)

an(1 − a)k−n =

∞∑

k=n

(k − 2

n − 2

)

an(1 − a)k−n .

Effectuons les changement de variable K = k − 1 et N = n − 1. On obtient

E

(n − 1

X − 1

)

=

∞∑

K=N

(K − 1

N − 1

)

aN+1(1 − a)K−N = a

∞∑

K=N

(K − 1

N − 1

)

aN (1 − a)K−N .

La somme qui apparaît à droite est la somme des coefficients de la loi B−(N, a) et vaut 1. On trouvedonc

E

(n − 1

X − 1

)

= a .

EI 108

4. On a nécessairement k ≥ n. Cherchons dans ce cas P(X = k). On a effectué k expériences, et n − 1ont réussies sur les k − 1 premières expériences (la n−ième est nécessairement réussie), si l’on choisitles positions des expériences réussies, on a donc

(k−1n−1

)possibilités. La probabilité d’un de ces choix est

an(1 − a)k−n, donc

P(X = k) =

(k − 1

n − 1

)

an(1 − a)k−n ,

et l’on a bien la loi B−(n, a).

5. (a) On veut avoir n réussites avec X et n′ avec X ′, donc X + X ′ donnera n + n′ réussites.

(b) Comme X prend ses valeurs dans [n, ∞ [∩N et Y dans [ 1, ∞ [∩N, la variable X + Y prend sesvaleurs dans [n + 1, ∞ [∩N. Soit donc k un entier plus grand que n + 1. Alors

P(X + Y = k) =

k−1∑

r=n

P(X = r, Y = k − r) ,

et puisque X ⊥⊥ Y ,

P(X + Y = k) =

k−1∑

r=n

P(X = r)P(Y = k − r)

=

k−1∑

r=n

[(r − 1

n − 1

)

an(1 − a)r−n

]

[a(1 − a)k−r−1]

= an+1(1 − a)k−(n+1)k−1∑

r=n

(r − 1

n − 1

)

.

Mais d’après la formule rappelée dans l’énoncé

(m

n

)

=

m−1∑

i=n−1

(i

n − 1

)

,

donc en posant m = k − 1 et en faisant le changement d’indice r = i + 1, on trouve

k−2∑

i=n−1

(i

n − 1

)

=

k−1∑

r=n

(r − 1

n − 1

)

=

(k − 1

n

)

,

d’où

P(X + Y = k) =

(k − 1

n

)

an+1(1 − a)k−(n+1) .

La variable aléatoire X + Y suit donc une loi B−(n + 1, a).

Supposons maintenant quePX = B−(n, a)PYi

= B−(a) si 1 ≤ i ≤ n′

X,Y1, . . . , Yn′ ⊥⊥.

EI 109

Une récurrence immédiate montre que Y = Y1 + · · · + Yn′ suit une loi B(n′, a) et que X + Y suit uneloi B(n + n′, a), ce qui démontre la conjecture.

6. Il résulte de la question précédente que la somme de n variables aléatoires indépendantes Xi suivantune loi B−(a) suit une loi B−(n, a). Comme on a pour une des variables Xi

E(Xi) =1

aet V(Xi) =

1 − a

a2,

on en déduit immédiatement

E(X1 + · · · + Xn) = E(X1) + · · ·E(Xn) =n

a,

et puisque les variables sont indépendantes

V(X1 + · · · + Xn) = V(X1) + · · ·V(Xn) =n(1 − a)

a2.

7. Le calcul de la question 1. a montré que si X suit une loi B−(n, a), on a

gX(s) = E(sX) =

[as

1 − (1 − a)s

]n

.

Si X ′ suit une loi B−(n′, a), on a de même

gX′(s) = E(sX′

) =

[as

1 − (1 − a)s

]n′

.

Alors, si X et X ′ sont indépendantes

gX+X′(s) = gX(s)gX′(s) =

[as

1 − (1 − a)s

]n+n′

,

et on retrouve que X + X ′ suit une loi B−(n + n′, a).

De même, si les variables Xi sont indépendantes et de même loi B−(a), on a

gXi(s) =

as

1 − (1 − a)s,

et comme les variables sont indépendantes

gX1+···+Xn(s) = gX1(s) · · · gXn(s) =

[as

1 − (1 − a)s

]n

,

donc X1 + · · · + Xn suit une loi B−(n, a).

Pour une variable X suivant une loi B−(n, a), on a

gX(s) =

[as

1 − (1 − a)s

]n

.

EI 110

donc en dérivant

g′X(s) = n

[as

1 − (1 − a)s

]n−1 a

(1 − (1 − a)s)2,

d’oùE(X) = g′X(1) =

n

a.

Puis en dérivant une seconde fois

g′′X(s) = n(n − 1)

[as

1 − (1 − a)s

]n−2 a2

(1 − (1 − a)s)4+ n

[as

1 − (1 − a)s

]n−1 2a(1 − a)

(1 − (1 − a)s)3,

d’où

g′′X(1) =n(n − 1)

a2+

2n(1 − a)

a2=

n2 + n − 2na

a2.

Alors

V(X) = g′′X(1) + g′X(1) − g′X(1)2 =n2 + n − 2na

a2+

n

a− n2

a2=

n − na

a2.

103 Soit X et Y deux variables aléatoires réelles indépendantes qui suivent la même loi géométrique.Calculer P(X ≥ j) et en déduire que la variable aléatoire min(X,Y ) suit, elle-aussi, une loi géométrique.

Solution

On a, si j ≥ 1,

P(X ≥ j) =

∞∑

k=j

P(X = k) =

∞∑

k=j

a(1 − a)k−1 = (1 − a)j−1 .

Posons Z = min(X,Y ). Si j ≥ 1, l’événement Z = j est la réunion des événements X = j, Y ≥ jet X ≥ j, Y = j, donc

P(Z = j) = P(X = j, Y ≥ j) + P(X ≥ j, Y = j) − P(X = j, Y = j) ,

et comme les variables X et Y sont indépendantes

P(Z = j) = P(X = j)P(Y ≥ j) + P(X ≥ j)P(Y = j) − P(X = j)P(Y = j) ,

ce qui donne en remplaçant

P(Z = j) = a(1 − a)j−1(1 − a)j−1 + a(1 − a)j−1(1 − a)j−1 − a(1 − a)j−1a(1 − a)j−1 ,

doncP(Z = j) = (2a − a2)(1 − a)2j−2 = (2a − a2)(1 − (2a − a2))j−1 .

La variable Z suit donc une loi B−(2a − a2).(On remarquera que l’on a bien 0 < 2a − a2 < 1).

EI 111

104 Soit X une variable aléatoire qui suit la loi de Poisson P(λ). On note Y une autre variablealéatoire à valeurs dans N, définie sur le même espace probabilisé (Ω, P) que X, telle que, pour toutentier naturel i,

P(Y = j |X = i) =

P(Y = j) si |i − j| ≥ 20 si |i − j| = 1λi−1[λ2 + λ(i + 1) + i(i + 1)]e−λ

(i + 1)!sinon

Montrer que Y suit, elle aussi, une loi de Poisson.

Solution

On a

P(Y = j) =∞∑

i=0

P(Y = j,X = i) =∞∑

i=0

P(Y = j|X = i)P(X = i) .

En distinguant suivant les valeurs de i

P(Y = j) =

j−2∑

i=0

P(Y = j|X = i)P(X = i)

+P(Y = j|X = j − 1)P(X = j − 1) + P(Y = j|X = j)P(X = j)

+P(Y = j|X = j + 1)P(X = j + 1) +∞∑

i=j+2

P(Y = j|X = i)P(X = i) .

D’après les propriétés données dans l’énoncé

P(Y = j) =

j−2∑

i=0

P(Y = j)P(X = i) + P(Y = j|X = j)P(X = j) +

∞∑

i=j+2

P(Y = j)P(X = i) .

Mais on a aussi

P(Y = j) =

∞∑

i=0

P(Y = j)P(X = i)

=

j−2∑

i=0

P(Y = j)P(X = i) + P(Y = j)

j+1∑

i=j−1

P(X = i) +

∞∑

i=j+2

P(Y = j)P(X = i) .

Donc en soustrayant les deux relations précédentes, on obtient

0 = P(Y = j|X = j)P(X = j) − P(Y = j)

j+1∑

i=j−1

P(X = i) ,

d’où l’on tire

P(Y = j) =P(Y = j|X = j)P(X = j)

j+1∑

i=j−1

P(X = i)

.

EI 112

Il reste à expliciter la formule précédente, qui va se simplifier et l’on obtient

P(Y = j) =

λj−1[λ2 + λ(j + 1) + j(j + 1)]e−λ

(j + 1)!

e−λλj

j!

e−λλj−1

(j − 1)!+

e−λλj

j!+

e−λλj+1

(j + 1)!

=e−λλj

j!.

Donc Y suit aussi une loi P(λ).

105 Calculer le moment centré d’ordre 3 d’une variable aléatoire X qui suit la loi de Poisson P(λ).

Solution

En utilisant la fonction caractéristique

g(s) =

∞∑

k=0

e−λ λksk

k!= eλ(s−1) ,

on obtient les dérivées successives dont on déduit les moments factoriels

g′(s) = λeλ(s−1) donc g′(1) = E(X) = λ

g′′(s) = λ2eλ(s−1) donc g′′(1) = E(X(X − 1)) = λ2

g′′′(s) = λ3eλ(s−1) donc g′′′(1) = E(X(X − 1)(X − 2)) = λ3

cela permet de calculerE(X2) = E(X(X − 1)) + E(X) = λ2 + λ ,

puisE(X3) = E(X(X − 1)(X − 2)) + 3E(X2) − 2E(X) = λ3 + 3λ2 + λ .

Or(X − E(X))3 = X3 − 3X2

E(X) + 3XE(X)2 − E(X)3 ,

DoncE((X − E(X))3) = E(X3) − 3E(X2)E(X) + 3E(X)E(X)2 − E(X)3 ,

ce qui donneE((X − E(X))3) = E(X3) − 3E(X2)E(X) + 2E(X)3 ,

doncE((X − E(X))3) = λ3 + 3λ2 + λ − 3(λ2 + λ)λ + 2λ3 = λ .

106 Soit d variables aléatoires indépendantes X1, . . . ,Xd suivant respectivement les lois de PoissonP(λ1),. . . , P(λd).

1. (a) Calculer P(X1 = j|X1 + X2 = k) si j et k sont deux entiers tels que k ≥ j ≥ 0.

EI 113

(b) Quelle est la loi de X1 conditionnellement à X1 + X2 = k ?

2. On note S =

d∑

i=1

Xi. Quelle est la loi du vecteur aléatoire (X1, . . . ,Xd) conditionnellement à S = n.

Solution

1. (a) Si 0 ≤ j ≤ k, on a, puisque X1 et X2 sont indépendantes,

P(X1 = j,X1 + X2 = k) = P(X1 = j,X2 = k − j) = P(X1 = j)P(X2 = k − j) ,

donc

P(X1 = j|X1 + X2 = k) =P(X1 = j)P(X2 = k − j)

P(X1 + X2 = k).

Comme X1 + X2 suit une loi P(λ1 + λ2), on trouve

P(X1 = j|X1 + X2 = k) =

e−λ1λj1

j!

e−λ2λk−j2

(k − j)!

e−(λ1+λ2)(λ1 + λ2)k

k!

=

(k

j

)(λ1

λ1 + λ2

)j ( λ2

λ1 + λ2

)k−j

.

(b) On obtient donc une loi B(k, λ1λ1+λ2

).

2. Si k1 + · · · + kd 6= n la probabilité P(X1 = k1, . . . ,Xd = kd|S = n) est nulle. Dans le cas contraire,on a

P(X1 = k1, . . . ,Xd = kd|S = n) =P(X1 = k1, . . . ,Xd = kd)

P(S = n)

=

e−λ1λk11

k1!· · · e−λdλkd

d

kd!

e−(λ1+···+λd)(λ1 + · · · + λd)n

n!

=

(n

k1, . . . , kd

)(λ1

λ1 + · · · + λd

)k1

· · ·(

λd

λ1 + · · · + λd

)kd

,

où (n

k1, . . . , kd

)

=n!

k1! · · · kd!.

107 Soit X et Y deux variables aléatoires indépendantes à valeurs entières naturelles paires quisuivent la même loi de probabilité telle que

(∀k ∈ N) P(X = 2k) = (k + 1)P(X = 2k + 2) .

EI 114

Déterminer la loi de probabilité de la variable aléatoire réelle Z =X + Y

2.

Solution

On déduit immédiatement de la relation

P(X = 2k + 2) =P(X = 2k)

k + 1,

que, si k ≥ 0,

P(X = 2k) =P(X = 0)

k!.

Alors, si X et Y prennent des valeurs paires positives, Z prend des valeurs entières positives et, sik ∈ N,

P(Z = k) = P(X + Y = 2k) =

k∑

i=0

P(X = 2i, Y = 2k − 2i) .

Comme les variables X et Y sont indépendantes et de même loi

P(Z = k) =

k∑

i=0

P(X = 0)

i!

P(X = 0)

(k − i)!= P(X = 0)2

k∑

i=0

1

i!(k − i)!.

Alors

P(Z = k) =P(X = 0)2

k!

k∑

i=0

(k

i

)

=P(X = 0)2

k!2k ,

et il en résulte que Z suit une loi de Poisson P(2), et que

P(X = 0)2 = e−2 ,

doncP(X = 0) = e−1 .

108 Soit (An)n∈N∗ une suite d’événements aléatoires d’un espace probabilisé (Ω, P) mutuellementindépendants de même probabilité non nulle.

1. On note B0 = Ω et pour tout entier naturel n, on note Bn+1 = Bn ∩ ACn+1.

Montrer que la suite de terme général P(Bn) décroît vers 0.

2. On note E l’événement aléatoire « au moins un des An se réalise ».Montrer que P(E) = 1.

Solution

1. Posons P(Ai) = p > 0.

EI 115

On obtient immédiatement par récurrence que, si n ≥ 1,

Bn = AC1 ∩ · · · ∩ AC

n .

Comme les événements A1, . . . , An sont indépendants, il en est de même de AC1 , . . . , AC

n et

P(Bn) = P(AC1 ) · · · P(AC

n ) = (1 − p)n ,

et comme 0 ≤ 1 − p < 1, on en déduit que

limn→+∞

P(Bn) = 0 .

Mais Bn+1 est inclus dans Bn, et la suite (Bn)n≥1 est une suite décroissante d’événements, donc

P( limn→+∞

Bn) = limn→+∞

P(Bn) = 0 .

2. On a

limn→+∞

Bn =⋂

n≥1

Bn =⋂

n≥1

ACn =

n≥1

An

C

= EC .

On a donc obtenu

P(EC) = limn→+∞

Bn = 0 ,

et par suite P(E) = 1.

109 Un casino propose le jeu suivant :

le joueur paie une mise de 14 € puis lance un dé autant de fois qu’il est nécessaire pour obtenir autrechose qu’un as. La banque lui verse alors (5k − 1) € où k est le nombre d’as obtenus.

1. Déterminer le gain net moyen du joueur et celui du casino.

2. Quelle est la probabilité que ce joueur fasse « sauter la banque » si les réserves du casino se montentà 107 €.

Solution

1. Le joueur gagne donc (5k − 1) − 14 € s’il a tiré k as puis une autre valeur.

Si X désigne la variable aléatoire donnant le gain du joueur, cette variable prend les valeurs 5k − 15pour k ≥ 0, et

P(X = 5k − 15) =1

6k

5

6,

puisque la probabilité de tirer un as est 1/6 et celle de tirer une autre valeur est 5/6, les événementsétant indépendants.

EI 116

Alors

E(X) =

∞∑

k=0

(5k − 15)P(X = 5k − 15) =

∞∑

k=0

(5k − 15)5

6k+1=

∞∑

k=0

(5

6

)k+1

− 75

∞∑

k=0

1

6k+1.

Comme on obtient des séries géométriques, on trouve immédiatement

E(X) =5

6

1

1 − 56

− 75

6

1

1 − 16

= 5 − 15 = −10 .

L’espérance de gain moyen du joueur est donc de −10 € et pour la banque de 10 € .

2. La probabilité de faire sauter la banque est P(X ≥ 107). Cherchons les valeurs de k pour lesquelles5k − 15 ≥ 107. On a

k ≥ ln(107 + 15)

ln 5≈ 10, 01 ,

donc k ≥ 11. Alors

P(X ≥ 107) =

∞∑

k=11

5

6k+1=

5

612

1

1 − 16

=1

611.

110 Un joueur J , passionné de « Pile ou face », a trouvé un « pigeon » P qui accepte, à conditionque J lui verse d’abord 5 €, de jouer contre lui selon la règle suivante :

– à chaque partie, P devra miser la même somme que J et le gagnant de la partie empochera la misedes deux joueurs ;

– le nombre de parties n’est pas fixé à l’avance : le match durera jusqu’à ce que J décide de l’arrêter.

La tactique choisie par J est la suivante : il commence par jouer 10 € sur pile. Si pile sort, il empocherales 20 € et cessera de jouer. Sinon il aura perdu sa mise mais fera un nouveau pari sur pile en misantle double de sa mise précédente et ainsi de suite jusqu’à ce que pile sorte.

1. Quel est le nombre moyen de paris ?2. Quelle est l’espérance de gain net de J ?3. Quelle somme doit engager P ? Quelle somme doit pouvoir engager J ?4. La fortune de J n’est pas infinie. Quelle est réellement l’espérance de gain de son gain ?

Solution

1. Si l’on note la suite des résultats des parties, J s’arrête au bout de n parties lorsque l’on a FF . . . FP ,où F est sortie n − 1 fois. On a donc

P(FF . . . FP ) =1

2n.

La variable aléatoire N donnant le nombres de parties vérifie donc, pour n ≥ 1,

P(N = n) =1

2n.

EI 117

On remarque que cette variable suit une loi B−(a), avec a = 1/2 et en particulier

∞∑

n=1

P(N = n) = 1 .

Donc la probabilité pour que le jeu ne s’éternise pas est nulle. Par ailleurs

E(N) =1

a= 2 .

2. Le gain de J en € est, dans le cas où n parties ont été effectuées,

GJ = 10(2n −n−1∑

k=0

2k) − 5 = 10(2n − (2n − 1)) − 5 = 5 .

On a donc une variable certaine et E(GJ) = 5.

3. Pour jouer la première partie P doit disposer de 5 €, puisqu’il a déjà 5 € donnés par J . Ensuite cequ’il gagne à chaque fois lui permet de couvrir la mise suivante de J . Par contre J doit disposer d’unesomme infinie s’il veut être sûr de pouvoir continuer le jeu.

4. Si J ne dispose pas d’une somme infinie. Le jeu s’arrête soit lorsque J a gagné, soit lorsque la sommequ’il doit miser dépasse ce qu’il possède.

Supposons que J puisse miser au plus p fois. Pour n ∈ [[ 1, p ]] , il peut avoir gagné 5 € après la n−ièmepartie, et ceci avec la probabilité 1/2n, mais il peut avoir perdu à chacune des p parties. Il a doncperdu, en €

5 + 10

p−1∑

k=0

2k = 5 + 10(2p − 1) = 10 2p − 5 ,

avec la probabilité P(FF . . . F ) où F figure p fois,

P(FF . . . F ) =1

2p,

Donc

E(GJ ) = 5

p∑

n=1

1

2n− (10 2p − 5)

1

2p

= 5

(

1 − 1

2p

)

− (10 2p − 5)1

2p

= −5

111 1. De quel événement le nombre g(1)+g(−1)2 est-il la probabilité si g est la fonction génératrice

d’une variable aléatoire X à valeurs dans N ?

EI 118

2. En effectuant mille répétitions indépendantes de l’expérience aléatoire « choisir au hasard (avecéquiprobabilité) une des lettres d’un même livre de cent pages », on a obtenu quatre fois la lettre z.

En déduire une estimation de la probabilité de trouver un nombre pair de z dans un paragraphe decent vingt cinq lettres choisi au hasard dans ce livre.

Solution

1. On a

g(s) =

∞∑

k=0

P(X = k)sk ,

alors

g(s) + g(−s)

2=

1

2

∞∑

k=0

P(X = k)(sk + (−s)k)

=1

2

∞∑

k=0

P(X = k)(1 + (−1)k)sk

=

∞∑

j=0

P(X = 2j)s2j .

Doncg(1) + g(−1)

2=

∞∑

j=0

P(X = 2j) = P(X pair) .

2. Soit N le nombre de lettres du livre, et N1 le nombre de lettres z. La probabilité de tirer un z enchoisissant une lettre au hasard est donc

p =N1

N.

Soit Zi la variable aléatoire valant 1 si z a été obtenu au i−ème tirage et 0, sinon. Les variables Zi

sont indépendantes et suivent une loi de Bernoulli B(p) d’espérance p. La loi des grands nombres nousdit que la variable Z définie par

Z =Z1 + · · ·Z1000

1000,

a pour limite p.

Or Z1 + · · · + Z1000 donne le nombre de lettres z obtenues sur 1000 tirages. Donc Z tend vers 4/1000,et on peut prendre

p =4

1000=

1

250.

Si X est la variable aléatoire donnant le nombre de lettres z obtenues dans un paragraphe de 125lettres, elle suit une loi H(125, N1, N), et peut s’approcher tout d’abord par une loi B(125, p), puis parune loi P(125p) = P(1/2), dont la fonction caractéristique g est définie par

g(s) = e(s−1)/2 .

EI 119

Donc la probabilité que le nombre de lettres z soit pair peut s’approcher par

g(1) + g(−1)

2=

1

2

(

1 +1

e

)

≈ 0, 68 .

112 1. Montrer que, pour tout p entier naturel

4p+1∑

i=1i impair

i = (2p + 1)2 .

2. Deux joueurs A et B jouent avec un dé truqué selon la règle suivante : A lance le dé, puis B lancele dé 3 fois, puis A le relance 5 fois, puis B 7 fois, etc. . . jusqu’à ce que l’un des deux obtienne un 6 etemporte ainsi la partie. On note a la probabilité que le dé montre un 6 et b = 1−a, et on suppose a 6= 0.

Montrer que :

P(A gagne) =∞∑

k=0

(−b)k2.

3. Le dé peut-il être truqué de façon à rendre le jeu équitable ?Indication : utiliser la formule d’Euler. Si |α| < 1, on a

k∈Z

αk2= lim

n→+∞

n∏

k=1

(1 − α2k)(1 + α2k−1)2 .

Solution

On écrit4p+1∑

i=1i impair

i =

2p∑

k=0

(2k + 1) = 2

2p∑

k=0

k + (2p + 1) ,

donc, en utilisant la sommes∑

k=0

k =s(s + 1)

2,

on obtient4p+1∑

i=1i impair

i = (2p)(2p + 1) + (2p + 1) = (2p + 1)2 .

2. Si l’on écrit les premières situations où A gagne avec les probabilités correspondantes, on a

A aABBBA b4aABBBAA b5aABBBAAA b6aABBBAAAA b7aABBBAAAAA b8aABBBAAAAABBBBBBBA b16a

EI 120

De manière générale, on constate que pour tout entier p positif et pour tout k compris entre (2p)2 et(2p + 1)2 − 1, A gagne après k parties perdues, donc avec une probabilité bka. Alors

P(A gagne) =

∞∑

p=0

(2p+1)2−1∑

k=(2p)2

abk .

Mais

(2p+1)2−1∑

k=(2p)2

abk =

(2p)2+4p∑

k=(2p)2

abk = ab(2p)24p∑

k=0

bk = b(2p)2(1 − b4p+1) = b(2p)2 − b(2p+1)2 .

Posonsun = (−b)n

2.

Les nombres n et n2 ayant la même parité, on a (−1)n = (−1)n2, et donc

un = (−1)nbn2,

et l’on peut alors remarquer que la série de terme général un converge, car c’est une série alternée.Donc

P(A gagne) =

∞∑

p=0

(b(2p)2 − b(2p+1)2) = limN→+∞

N∑

p=0

(b(2p)2 − b(2p+1)2)

= limN→+∞

N∑

p=0

((−b)(2p)2 + (−b)(2p+1)2)

= limN→+∞

N∑

p=0

(−b)(2p)2 +

N∑

p=0

(−b)(2p+1)2

= limN→+∞

2N+1∑

k=0

(−b)k =∞∑

n=0

(−b)n2.

3. En remarquant que (−b)k2

ne dépend pas du signe de k, tous les termes, sauf si k = 0, de la sommede gauche ci-dessous figurent deux fois ce qui permet d’écrire

k∈Z

(−b)k2

= 2∞∑

n=0

(−b)n2 − 1 .

Donc

P(A gagne) =1

2+

1

2

k∈Z

(−b)k2,

et en utilisant la formule d’Euler

P(A gagne) =1

2+

1

2lim

n→+∞

n∏

k=1

(1 − b2k)(1 − b2k−1)2 .

EI 121

Etudions la limite du produit

Pn =

n∏

k=1

(1 − b2k)(1 − b2k−1)2 .

On a

− ln(Pn) =

n∑

k=1

(− ln(1 − b2k) − 2 ln(1 − b2k−1)) .

Or, lorsque k tend vers l’infini,

− ln(1 − b2k) ∼ b2k et − ln(1 − b2k−1) ∼ b2k−1 .

Comme la série de terme général b2k et celle de terme général b2k−1 sont des séries convergentes etpositives, alors il en est de même de la série de terme général − ln(1− b2k) et de celle de terme général− ln(1 − b2k−1). Donc la série de terme général ln(Pn) converge. Notons S sa somme. On a alors

limn→+∞

Pn = limn→+∞

eln Pn = eS > 0 .

et

P(A gagne) =1

2+

eS

2>

1

2.

Le jeu ne sera jamais équitable.

113 Deux joueurs A et B disposent chacun d’une urne qui contient dix boules indiscernables autoucher. Il y a b ≥ 1 boules noires dans l’urne de B et a ≥ 2 boules noires dans celle de A.

A joue le premier. Pour cela, il tire au hasard une boule dans son urne. Si celle-ci n’est pas noire, illa replace dans son urne et c’est au tour de B de jouer. Sinon A tire une seconde boule dans son urnesans avoir remis la première tirée. Si cette deuxième boule n’est pas noire, il replace les deux boulesdans son urne et c’est au tour de B de jouer. Sinon A a gagné la partie.

Pour jouer, B tire une boule de son urne. Si celle-ci n’est pas noire, il la replace dans son urne et c’estau tour de A de jouer. Sinon B a gagné la partie.

Peut-on choisir a et b pour que ce jeu soit équitable ?

Solution

Notons Ag, Ap, Bg, Bp, respectivement les événements : A tire une boule noire, A ne tire pas une boulenoire, B tire une boule noire, B ne tire pas une boule noire.

Le joueur A gagne la partie lorsque après un nombre fini de séquences AgAp ou Ap chacune suivie deBp on trouve une séquence AgAg.

On a

P(AgAp) =a

10

10 − a

9, P(Bp) =

10 − b

10, P(Ap) =

10 − a

10, P(AgAg) =

a

10

a − 1

9.

EI 122

Si l’on a k séquences AgAp et n−k séquences Ap, comme elles peuvent figurer dans un ordre quelconque,il y aura

(nk

)façons de les placer, par ailleurs Bp figure n fois. La probabilité que le jeu s’arrête après

n séquences est donc

pn =

(10 − b

10

)n n∑

k=0

(n

k

)(a

10

10 − a

9

)k (10 − a

10

)n−k

=

(10 − b

10

)n (10 − a

10

)n n∑

k=0

(n

k

)(a

9

)k.

Mais on peut alors appliquer la formule du binôme et l’on obtient

pn =

(10 − a

10

10 − b

10

)n (a

9+ 1)n

=

(10 − a

10

10 − b

10

9 + a

9

)n

.

La probabilité pour que A gagne est alors

P(A gagne) =a

10

a − 1

9

∞∑

n=0

pn ,

(il peut ne pas y avoir de séquence autre que AgAg et donc la sommation se fait à partir de n = 0).On trouve donc la somme d’une série géométrique

P(A gagne) =a

10

a − 1

9

∞∑

n=0

(10 − a

10

10 − b

10

9 + a

9

)n

=

a

10

a − 1

9

1 − 10 − a

10

10 − b

10

9 + a

9

.

Le joueur B gagne la partie lorsqu’après un certain nombre de séquences AgAp ou Ap entre lesquelless’intercale Bp, apparaît l’événement Bg.Le calcul est le même que dans le cas précédent, mais cette fois Bp figure n−1 fois. Donc la probabilitéque le jeu s’arrête après n séquences sera

qn =

(10 − b

10

)n−1 n∑

k=0

(n

k

)(a

10

10 − a

9

)k (10 − a

10

)n−k

=

(10 − b

10

)n−1 (10 − a

10

9 + a

9

)n

.

La probabilité pour que B gagne est alors

P(B gagne) =b

10

∞∑

n=1

qn ,

(il y a au moins une séquence AgAp ou Ap avant Bg et donc la sommation se fait à partir de n = 1).On trouve de nouveau la somme d’une série géométrique

P(B gagne) =b

10

10 − a

10

9 + a

9

∞∑

n=1

(10 − a

10

10 − b

10

9 + a

9

)n−1

=

b

10

10 − a

10

9 + a

9

1 − 10 − a

10

10 − b

10

9 + a

9

.

EI 123

On peut vérifier queP(A gagne) + P(B gagne) = 1 ,

ce qui montre que le la probabilité que le jeu s’éternise est nulle.

Les probabilités P(A gagne) et P(B gagne) sont égales si et seulement si

a

10

a − 1

9=

b

10

10 − a

10

9 + a

9,

ce qui équivaut à10a(a − 1) = (10 − a)(9 + a)b ,

soit à10a(a − 1) = (90 + a − a2)b ,

ou encore à10a(a − 1) = 90b − a(a − 1)b ,

puis àa(a − 1)(10 + b) = 90b ,

et finalement à

a(a − 1) =32 × 2 × 5 × b

10 + b.

Comme ce nombre est entier, et que les facteurs premiers du numérateur sont inférieurs à 10, cela exclules valeurs de b telles que 10 + b soit premier c’est-à-dire b = 1, 3, 7, 9. On peut aussi éliminer 4, car 90n’est pas divisible par 7, et 6, car le numérateur est divisible par 4 et non par 16. Il est facile de testerles valeurs restantes :− pour b = 2, l’équation devient a(a − 1) = 15 qui n’a pas de solution entière− pour b = 8, on trouve a(a − 1) = 40 qui n’a pas non plus de solution entière,− par contre, pour b = 5, on trouve

90b

10 + b= 30 = 6 × 5 = a(a − 1) ,

qui a comme solution entière a = 6.

114 Deux joueurs A et B, de fortunes initiales respectives nA et nB = N − nA € décident des’affronter en une suite de parties indépendantes.A chaque partie le joueur A peut gagner avec la probabilité a, sinon c’est B qui gagne avec la probabilitéb = 1 − a, le perdant donnant 1€ au gagnant. Le match ne s’arrête qu’avec la ruine d’un des deuxjoueurs.La fortune de chaque joueur varie donc au cour du match ; on note αn la probabilité que A gagne parruine de B quand on se place à une étape du match où la fortune de A s’élève à n €.

1. Montrer que ∀n ∈ [[ 1, N − 1 ]] αn = aαn+1 + (1 − a)αn−1.

2. On note u1 = α1 et, pour tout n dans [[ 1, N − 1 ]] , un−1 = αn−1 − αn.

EI 124

Montrer que ∀n ∈ [[ 1, N ]] , un = α1

(b

a

)n−1

.

3. En déduire que, si a 6= b, ∀n ∈ [[ 1, N ]]

αn =

1 −(

b

a

)n

1 −(

b

a

)N.

4. Que vaut αn si a = b.

Solution

1. Notons X la variable aléatoire donnant la fortune de A, et Ag l’événement « A gagne une partie ».On a donc

αn = P|X=n(A gagne) .

Alors

αn = P|X=n(A gagne|Ag)P(Ag) + P|X=n(A gagne|ACg )P(AC

g )

= aP(A gagne|Ag et X = n) + (1 − a)P(A gagne|ACg et X = n) .

Mais

Ag et X = n = X = n + 1 et ACg et X = n = X = n − 1 ,

donc

αn = aP(A gagne|X = n + 1) + (1 − a)P(A gagne|X = n − 1) = aαn+1 + (1 − a)αn−1 .

2. On peut écrire la relation de la question précédente

aαn + (1 − a)αn = aαn+1 + (1 − a)αn−1 ,

soit

a(αn+1 − αn) = (1 − a)(αn − αn−1) ,

et finalement

un+1 =b

aun ,

avec u1 = α1. On a donc une suite géométrique et il en résulte que, si 1 ≤ n ≤ N ,

un = α1

(b

a

)n−1

.

3. Alors, si 1 ≤ p ≤ n ≤ N ,

αp − αp−1 = α1

(b

a

)p−1

,

EI 125

et en sommant ces égalitésn∑

p=2

(αp − αp−1) = α1

n∑

p=2

(b

a

)p−1

,

on a à droite une somme télescopique, donc

αn − α1 = α1

n∑

p=2

(b

a

)p−1

,

ce qui donne

αn = α1

n∑

p=1

(b

a

)p−1

.

On a à gauche la somme des termes d’une suite géométrique, donc, si a 6= b,

αn = α1

1 −(

b

a

)n

1 − b

a

.

Mais, pour N

1 = αN = α1

1 −(

b

a

)N

1 − b

a

,

donc, en faisant le quotient

αn =

1 −(

b

a

)n

1 −(

b

a

)N.

Si a = b, on a

αn = α1

n∑

p=1

1 = nα1 .

et donc

1 = αN = Nα1 ,

ce qui donne

αn =n

N.

115 Trois joueurs A, B, C jouent à pile ou face selon la règle suivante : A et B jouent une premièrepartie. C remplace le perdant et le match se poursuit ainsi, le gagnant d’une partie jouant contre leperdant de la partie précédente, jusqu’à ce que l’un des trois gagne consécutivement contre les deuxautres.

EI 126

Montrer que la probabilité que le match s’éternise est nulle et déterminer la probabilité de gagner lematch pour chacun des trois joueurs.

Solution

A chaque partie un joueur a une probabilité 1/2 de gagner, et donc la probabilité de gain de troisjoueurs successifs est 1/8. Etudions la suite des gagnants, et regardons dans quelles situations chacundes joueurs emporte le match.

Le joueur A emporte le match lorsque l’on a un des deux cas suivants :

si A emporte la première partie ACB . . . ACB AA

si B emporte la première partie BCA . . .BCA A

dans le premier cas, si n est le nombre de triplets ACB, la probabilité sera pour tout n ≥ 0,

P(ACB . . . ACB AA) =1

4.8n,

dans le second cas si n est le nombre de triplets BCA, la probabilité sera pour tout n ≥ 1,

P(BCA . . .BCA A) =1

2.8n,

et donc la probabilité que A gagne le match est

P(A gagne) =∞∑

n=0

1

4.8n+

∞∑

n=1

1

2.8n,

ce qui se calcule facilement puisque l’on a des séries géométriques :

P(A gagne) =1

4

1

1 − 18

+1

16

1

1 − 18

=5

16

8

7=

5

14.

En raison de la symétrie du problème entre A et B, on a également

P(B gagne) =5

14.

Pour le joueur C on a un des deux cas suivants :

si A emporte la première partie ACB . . . ACB ACC

si B emporte la première partie BCA . . .BCA BCC

Si n ≥ 1 est le nombre de triplets, on a

P(C gagne) = 2

∞∑

n=1

1

8n=

1

4

1

1 − 18

=2

7.

EI 127

CommeP(A gagne) + P(B gagne) + P(C gagne) = 1 ,

la probabilité pour que le jeu s’éternise est donc nulle.

EI 128

V - Partiels et devoirs

116 On a représenté sur le diagramme c-dessous :– les probabilités des événements C et CC : P(C) = 0, 2– les probabilités conditionnelles sachant C des événements A et AC : P|C(AC) = 0, 6

– les probabilités conditionnelles sachant C ∩ AC des événements B et BC : P|C∩AC(B) = 0, 3– etc . . .

0,5

B

0,5

BC

????

???

0,4

A

0,6

AC

????

????

????

????

0,3

B

0,7

BC

????

???

0,2

C

0,8

CC

????

????

????

????

????

????

????

????

?

0,5

B

0,5

BC

????

???

0,5

A

0,5

AC

????

????

????

????

0,9

B

0,1

BC

????

???

EI 129

Compléter de la même façon, en justifiant tous les calculs, les deux diagrammes suivants :

B

BC

????

???

C

CC

????

????

????

????

B

BC

????

???

C

CC

????

???

A

AC

????

????

????

????

C

CC

????

???

B

BC

????

????

????

????

????

????

????

????

?

C

CC

????

???

A

AC

????

????

????

????

C

CC

????

???

EI 130

Solution

Puisque A,AC est un système complet d’événements, on a

P|C(B) = P|C(A ∩ B) + P|C(AC ∩ B)

= (P|C)|A(B)P|C(A) + (P|C)|AC (B)P|C(AC)

= P|C∩A(B)P|C(A) + P|C∩AC (B)P|C(AC)

= 0, 5 × 0, 4 + 0, 3 × 0, 6 = 0, 38 .

On en déduit

P|C(BC) = 1 − 0, 38 = 0, 62 .

De même

P|CC (B) = P|CC∩A(B)P|CC (A) + P|CC∩AC (B)P|CC (AC) = 0, 5 × 0, 5 + 0, 9 × 0, 5 = 0, 7 ,

et

P|CC (BC) = 1 − 0, 7 = 0, 3 .

D’où le diagramme

0,38

B

0,62

BC

????

???

0,2

C

0,8

CC

????

????

????

????

0,7

B

0,3

BC

????

???

Puisque C,CC est un système complet d’événements, on a,

P(B) = P|C(B)P(C) + P|CC (B)P(CC)

= 0, 38 × 0, 2 + 0, 7 × 0, 8 = 0, 636

Alors

P(BC) = 1 − 0, 636 = 0, 364 .

EI 131

Puisque C,CC est un système complet d’événements, on a, en utilisant la formule

P(U ∩ V ∩ W ) = PV ∩W (U)PW (V )P(W ) ,

la relation

P(A ∩ B) = P(A ∩ B ∩ C) + P(A ∩ B ∩ CC)

= P|C∩A(B)P|C(A)P(C) + P|CC∩A(B)P|CC (A)P(CC)

= 0, 5 × 0, 4 × 0, 2 + 0, 5 × 0, 5 × 0, 8 = 0, 24

d’où

P|B(A) =P(A ∩ B)

P(B)=

0, 24

0, 636=

20

53et P(AC) = 1 − 20

53=

33

53.

On a également

P(A ∩ BC) = P(A ∩ BC ∩ C) + P(A ∩ BC ∩ CC)

= P|C∩A(BC)P|C(A)P(C) + P|CC∩A(BC)P|CC (A)P(CC)

= 0, 5 × 0, 4 × 0, 2 + 0, 5 × 0, 5 × 0, 8 = 0, 24

d’où

P|BC (A) =P(A ∩ BC)

P(BC)=

0, 24

0, 364=

60

91et P(AC) = 1 − 60

91=

31

91.

Déterminons maintenant les dernières branches du diagramme.

P|A∩B(C) =P(A ∩ B ∩ C)

P(A ∩ B)=

P(C)P|C(A)PA∩C(B)

P(A ∩ B)=

0, 2 × 0, 4 × 0, 5

0, 24=

1

6.

Donc

P|A∩B(CC) = 1 − 1

6=

5

6.

De même

P|AC∩B(C) =P(AC ∩ B ∩ C)

P(AC ∩ B)=

P(C)P|C(AC)PAC∩C(B)

P(B) − P(A ∩ B)=

0, 2 × 0, 6 × 0, 3

0, 636 − 0, 24=

1

11.

Donc

P|AC∩B(CC) = 1 − 1

11=

10

11.

Ensuite

P|A∩BC (C) =P(A ∩ BC ∩ C)

P(A ∩ BC)=

P(C)P|C(A)PA∩C(BC)

P(A ∩ BC)=

0, 2 × 0, 4 × 0, 5

0, 24=

1

6.

Donc

P|A∩BC (CC) = 1 − 1

6=

5

6.

EI 132

Et enfin

P|AC∩BC (C) =P(AC ∩ BC ∩ C)

P(AC ∩ BC)=

P(C)P|C(AC)PAC∩C(BC)

P(BC) − P(A ∩ BC)=

0, 2 × 0, 6 × 0, 7

0, 364 − 0, 24=

21

31.

Donc

P|AC∩BC (CC) = 1 − 21

31=

10

31.

On obtient le diagramme suivant :

1/6

C

5/6

CC

????

???

20/53

A

33/53

AC

????

????

????

????

1/11

C

10/11

CC

????

???

0,636

B

0,364

BC

????

????

????

????

????

????

????

????

?

1/6

C

5/6

CC

????

???

60/91

A

31/91

AC

????

????

????

????

21/31

C

10/31

CC

????

???

117 Soit Ω un univers fini, A et B deux événements aléatoires et P une probabilité sur Ω telle que(∀ω ∈ Ω) P(ω) > 0. On note Φ l’application définie sur P(Ω) × P(Ω) par

Φ(E,F ) = P(E ∩ F ) − P(E)P(F ) .

EI 133

1) Montrer que

a) −1

4≤ Φ(A,AC) ≤ 0 ≤ Φ(A,A) ≤ 1

4,

b) si Φ(A,A) =1

4, alors P(A) = P(AC) .

2) a) Que vaut Φ(A,B) + Φ(AC , B) ?

b) Calculer P(A ∪ B) + P(AC)P(BC) et en déduire un encadrement de Φ(A,B).

3) A tout événement aléatoire K de P(Ω) on associe la variable aléatoire UK définie sur Ω par :

UK(ω) = 1lK(ω) − P(K) =

1 − P(K) si ω ∈ K−P(K) sinon

a) Calculer S(ω) =∑

ω∈Ω

UA(ω)UB(ω)P(ω).

b) Soit Q la fonction polynôme définie sur R par

Q(t) =∑

ω∈Ω

[tUA(ω) + UB(ω)]2P(ω) .

Etudier le signe de Q(t) et en déduire un autre encadrement de Φ(A,B).

4) a) Montrer que si |Φ(A,B)| = 1/4, alors P(A) = P(B).

b) Que vaut P(A∆ B) si |Φ(A,B)| = 1/4 ?

c) Qu’en déduit-on si Φ(A,B) = 1/4 ? si Φ(A,B) = −1/4 ?

Solution

1) a) On a

Φ(A,A) = P(A) − P(A)2 et Φ(A,AC) = −P(A)(1 − P(A)) = −Φ(A,A) .

Mais la fonction polynomiale ϕ définie par ϕ(x) = x − x2 est positive sur l’intervalle [ 0, 1 ] et atteintson maximum pour x = 1/2. Ce maximum vaut 1/4. On en déduit donc, en prenant x = P(A), que

0 ≤ Φ(A,A) ≤ 1

4.

Alors

−1

4≤ Φ(A,AC) = −P(A,A) ≤ 0 .

On peut également écrire

Φ(A,A) = P(A)(1 − P(A)) =1

4−(

1

2− P(A)

)2

,

EI 134

ce qui permet de retrouver l’inégalité

0 ≤ Φ(A,A) ≤ 1

4.

b) Comme la fonction ϕ atteint son maximum en 1/2 uniquement, on en déduit donc que l’égalité

Φ(A,A) = ϕ(P(A)) = 1/4

impliqueP(A) = 1/2 ,

et donc

P(AC) = 1 − P(A) =1

2= P(A) .

Cela se déduit également de l’égalité

P(A)(1 − P(A)) =1

4−(

1

2− P(A)

)2

=1

4,

2) a) On a

Φ(A,B) + Φ(AC , B) = P(A ∩ B) + P(AC ∩ B) − P(A)P(B) − P(AC)P(B) .

Mais, puisque le système A,AC est complet, on a

P(A ∩ B) + P(AC ∩ B) = P(B) ,

etP(A)P(B) + P(AC)P(B) = P(B)(P(A) + P(AC)) = P(B) .

On en déduit donc queΦ(A,B) + Φ(AC , B) = 0 .

b) On a

P(A ∪ B) + P(AC)P(BC) = P(A) + P(B) − P(A ∩ B) + (1 − P(A))(1 − P(B))

= 1 − P(A ∩ B) + P(A)P(B)

= 1 − Φ(A,B) .

Comme P(A ∪ B) + P(AC)P(BC) appartient à l’intervalle [ 0, 2 ] , on a donc

0 ≤ 1 − Φ(A,B) ≤ 2 ,

d’où−1 ≤ Φ(A,B) ≤ 1 .

3) Pour calculer la somme

S(ω) =∑

ω∈Ω

UA(ω)UB(ω)P(ω) ,

on peut procéder de plusieurs manières :

EI 135

Première méthode On utilise le fait que le système A ∩B,A∩BC , AC ∩B,AC ∩BC est complet,ce qui permet d’écrire

S(ω) =∑

ω∈A∩B

UA(ω)UB(ω)P(ω) +∑

ω∈A∩BC

UA(ω)UB(ω)P(ω)

+∑

ω∈AC∩B

UA(ω)UB(ω)P(ω) +∑

ω∈AC∩BC

UA(ω)UB(ω)P(ω) .

On calcule alors les quatre sommes qui apparaissent dans l’expression précédente. On a pour la première

S1(ω) =∑

ω∈A∩B

UA(ω)UB(ω)P(ω)

=∑

ω∈A∩B

(1 − P(A))(1 − P(B))P(ω)

= (1 − P(A))(1 − P(B))∑

ω∈A∩B

P(ω)

= (1 − P(A))(1 − P(B))P(A ∩ B) .

Pour la deuxième somme

S2(ω) =∑

ω∈A∩BC

UA(ω)UB(ω)P(ω)

=∑

ω∈A∩BC

(1 − P(A))(−P(B))P(ω)

= (1 − P(A))(−P(B))∑

ω∈A∩BC

P(ω)

= −P(B)(1 − P(A))P(A ∩ BC) ,

et de manière symétrique pour la troisième

S3(ω) =∑

ω∈AC∩B

UA(ω)UB(ω)P(ω) = −P(A)(1 − P(B))P(AC ∩ B) .

Enfin pour la quatrième somme

S4(ω) =∑

ω∈AC∩BC

UA(ω)UB(ω)P(ω)

=∑

ω∈AC∩BC

(−P(A))(−P(B))P(ω)

= P(A)P(B)∑

ω∈AC∩BC

P(ω)

= P(A)P(B)P(AC ∩ BC) .

EI 136

Alors, en développant

S(ω) = (1 − P(A))(1 − P(B))P(A ∩ B) − P(B)(1 − P(A))P(A ∩ BC)

−P(A)(1 − P(B))P(AC ∩ B) + P(A)P(B)P(AC ∩ BC)

= P(A)P(B)[P(A ∩ B) + P(A ∩ BC) + P(AC ∩ B) + P(AC ∩ BC)]

−P(A)[P(A ∩ B) + P(AC ∩ B)] − P(B)[P(A ∩ B) + P(A ∩ BC)] + P(A ∩ B)

= P(A)P(B) − P(A)P(B) − P(A)P(B) + P(A ∩ B)

= Φ(A,B) .

Deuxième méthode On utilise les propriétés de la fonction indicatrice 1lK

1lA 1lB = 1lA∩B

E(1lK) =∑

ω∈Ω

1lK(ω)P(ω) =∑

ω∈K

P(ω) = P(K)

On a alors en développant

S(ω) =∑

ω∈Ω

(1lA(ω) − P(A))(1lB(ω) − P(B))P(ω)

=∑

ω∈Ω

1lA(ω) 1lB(ω))P(ω) −∑

ω∈Ω

1lA(ω)P(B)P(ω)

−∑

ω∈Ω

1lB(ω)P(A)P(ω) +∑

ω∈Ω

P(A)P(B)P(ω)

=∑

ω∈Ω

1lA∩B(ω)P(ω) − P(B)∑

ω∈Ω

1lA(ω)P(ω)

−P(A)∑

ω∈Ω

1lB(ω)P(ω) + P(A)P(B)∑

ω∈Ω

P(ω)

= P(A ∩ B) − P(B)P(A) − P(A)P(B) + P(A)P(B)

= P(A ∩ B) − P(A)P(B) .

b) Le polynôme Q(t) est un trinôme du second degré positif. Il en résulte donc que son discriminantest négatif. On a, en développant,

Q(t) =∑

ω∈Ω

(t2UA(ω)2 + 2tUA(ω)UB(ω) + UB(ω)2)P(ω)

= t2

(∑

ω∈Ω

UA(ω)2P(ω))2

+ 2t∑

ω∈Ω

UA(ω)UB(ω)P(ω) +∑

ω∈Ω

UB(ω)2P(ω) .

Donc en utilisant le calcul de a)

Q(t) = t2Φ(A,A) + 2tΦ(A,B) + Φ(B,B) .

Alors le discriminantδ = 4(Φ(A,B)2 − Φ(A,A)Φ(B,B)) ,

EI 137

est négatif, d’où l’on déduitΦ(A,B)2 ≤ Φ(A,A)Φ(B,B) ,

Mais on a vu que Φ(A,A) et Φ(B,B) sont inférieurs à 1/4. Donc

Φ(A,B)2 ≤ 1

16,

et finalement

−1

4≤ Φ(A,B) ≤ 1

4.

4) a) Si |Φ(A,B)| = 1/4, on a1

16≤ Φ(A,A)Φ(B,B) ≤ 1

16,

et donc

Φ(A,A)Φ(B,B) =1

16.

Puisque Φ(A,A) et Φ(B,B) sont inférieurs à 1/4, ceci n’est possible que s’ils sont exactement égaux à1/4. (Si l’un était strictement inférieur à 1/4, l’autre serait strictement plus grand). Donc

Φ(A,A) = Φ(B,B) =1

4,

et d’après la question 1)b), cela implique

P(A) = P(B) =1

2.

b) On a A∆ B = (A ∪ B) \ (A ∩ B), avec A ∩ B ⊂ A ∪ B, donc

P(A∆ B) = P(A ∪ B) − P(A ∩ B) = P(A) + P(B) − 2P(A ∩ B) ,

ce que l’on peut exprimer en fonction de Φ(A,B),

P(A∆ B) = P(A) + P(B) − 2P(A)P(B) − 2Φ(A,B) ,

Lorsque |Φ(A,B)| = 1/4, on a donc

P(A∆ B) =1

2− 2Φ(A,B) .

c) Remarquons que, dire que P(ω) n’est jamais nul, implique pour un événement U que P(U) = 0 nepeut avoir lieu que si U = ∅, et que P(U) = 1 ne peut avoir lieu que si UC = ∅, c’est-à-dire que si U = Ω.

Si Φ(A,B) = 1/4. AlorsP(A∆ B) = 0 .

et doncA∆ B = (A ∪ B) \ (A ∩ B) = ∅ .

Il en résulte que A ∩ B = A ∪ B, c’est-à-dire que A = B.

EI 138

Si Φ(A,B) = −1/4. Alors

P(A∆ B) = 1 .

On en déduit queA∆ B = (A ∪ B) \ (A ∩ B) = Ω ,

et donc que A ∪ B = Ω et A ∩ B = ∅, c’est-à-dire que A = BC .

118 Soit P une loi de probabilité sur l’univers Ω = 0, 1, 2, 3, 4, 5, 6, 7, 8, de moyenne m = 4, 816.On connaît les nombres qk = P(Ω ∩ [ 0, k ] ) pour tout entier k compris entre 1 et 5 :

k 1 2 3 4 5

qk 0, 024 0, 056 0, 104 0, 304 0, 720

La variable aléatoire réelle X définie sur Ω par X(ω) = « reste de la division entière de ω par 4 », suitla loi binomiale B(3; 0, 4).

Déterminer l’espérance mathématique de la variable aléatoire réelle Y définie sur Ω parY (ω) = « reste de la division entière de ω par 3 »

Solution

La variable aléatoire Y est telle que Y (Ω) = 0, 1, 2, et donc

E(Y ) = P(Y = 1) + 2P(Y = 2) .

On aP(Y = 1) = P(1, 4, 7) et P(Y = 2) = P(2, 5, 8) .

Si l’on pose pk = P(k), on a

P(Y = 1) = p1 + p4 + p7 et P(Y = 2) = p2 + p5 + p8 .

Pour obtenir l’espérance de Y , il suffit de calculer les valeurs pk pour 0 ≤ k ≤ 8. Les données del’exercice vont fournir 11 équations à 9 inconnues.

Exprimons tout d’abord les probabilités qk. On obtient les 5 relations suivantes

q1 = P(0, 1) = p0 + p1

q2 = P(0, 1, 2) = p0 + p1 + p2

q3 = P(0, 1, 2, 3) = p0 + p1 + p2 + p3

q4 = P(0, 1, 2, 3, 4) = p0 + p1 + p2 + p3 + p4

q5 = P(0, 1, 2, 3, 4, 5) = p0 + p1 + p2 + p3 + p4 + p5

EI 139

Pour la loi de X, on a les quatre relations suivantes

P(X = 0) = P(0, 4, 8) = p0 + p4 + p8 = (0, 6)3 = 0, 216P(X = 1) = P(1, 5) = p1 + p5 = 3(0, 4)(0, 6)2 = 0, 432P(X = 2) = P(2, 6) = p2 + p6 = 3(0, 4)2(0, 6) = 0, 288P(X = 3) = P(3, 7) = p3 + p7 = (0, 4)3 = 0, 064

Le système obtenu est formé de 9 équations à 9 inconnues, auxquelles s’ajoutent deux équationsprovenant des égalités

P(Ω) =8∑

k=0

pk = 1 et m =8∑

k=0

kpk = 4, 816 .

On a donc le système

(S)

p0 + p1 = 0, 024p0 + p1 + p2 = 0, 056p0 + p1 + p2 + p3 = 0, 104p0 + p1 + p2 + p3 + p4 = 0, 304p0 + p1 + p2 + p3 + p4 + p5 = 0, 720p0 + p4 + p8 = 0, 216p1 + p5 = 0, 432p2 + p6 = 0, 288p3 + p7 = 0, 064p0 + p1 + p2 + p3 + p4 + p5 + p6 + p7 + p8 = 1p1 + 2p2 + 3p3 + 4p4 + 5p5 + 6p6 + 7p7 + 8p8 = 4, 816

On pourrait bien sûr résoudre ce système par la méthode du pivot, mais on peut le faire également parcombinaison de lignes dans le système. On rappelle qu’ajouter à une ligne d’un système d’équationslinéaires une combinaison linéaire des autres donne un système équivalent.

En soustrayant la ligne i de la ligne i + 1, pour i variant de 1 à 4, le système est équivalent à

(S1)

p0 + p1 = 0, 024p2 = 0, 032p3 = 0, 048p4 = 0, 2p5 = 0, 416p0 + p4 + p8 = 0, 216p1 + p5 = 0, 432p2 + p6 = 0, 288p3 + p7 = 0, 064p0 + p1 + p2 + p3 + p4 + p5 + p6 + p7 + p8 = 1p1 + 2p2 + 3p3 + 4p4 + 5p5 + 6p6 + 7p7 + 8p8 = 4, 816

EI 140

En soustrayant la ligne 2 de la ligne 8, la ligne 3 de la ligne 9 et la ligne 5 de la ligne 7, la système estéquivalent à

(S2)

p0 + p1 = 0, 024p2 = 0, 032p3 = 0, 048p4 = 0, 2p5 = 0, 416p0 + p4 + p8 = 0, 216p1 = 0, 016p6 = 0, 256p7 = 0, 016p0 + p1 + p2 + p3 + p4 + p5 + p6 + p7 + p8 = 1p1 + 2p2 + 3p3 + 4p4 + 5p5 + 6p6 + 7p7 + 8p8 = 4, 816

En soustrayant la ligne 9 de la ligne 1 la système est équivalent à

(S3)

p0 = 0, 008p2 = 0, 032p3 = 0, 048p4 = 0, 2p5 = 0, 416p0 + p4 + p8 = 0, 216p1 = 0, 016p6 = 0, 256p7 = 0, 016p0 + p1 + p2 + p3 + p4 + p5 + p6 + p7 + p8 = 1p1 + 2p2 + 3p3 + 4p4 + 5p5 + 6p6 + 7p7 + 8p8 = 4, 816

Enfin, en soustrayant de la ligne 6 la somme des lignes 4 et 1, on obtient

(S4)

p0 = 0, 008p2 = 0, 032p3 = 0, 048p4 = 0, 2p5 = 0, 416p8 = 0, 008p1 = 0, 016p6 = 0, 256p7 = 0, 016p0 + p1 + p2 + p3 + p4 + p5 + p6 + p7 + p8 = 1p1 + 2p2 + 3p3 + 4p4 + 5p5 + 6p6 + 7p7 + 8p8 = 4, 816

EI 141

On vérifie alors facilement que les deux dernières équations sont satisfaites et le système a donc commesolution

k 0 1 2 3 4 5 6 7 8

pi 0, 008 0, 016 0, 032 0, 048 0, 2 0, 416 0, 256 0, 016 0, 008

AlorsP(Y = 1) = p1 + p4 + p7 = 0, 232 et P(Y = 2) = p2 + p5 + p8 = 0, 456 ,

d’où

E(Y ) = P(Y = 1) + 2P(Y = 2) = 1, 144 .

EI 142

Partiel 1

119 La moitié des mails reçus par la messagerie d’une entreprise sont des SPAM (messages indési-rables). Le responsable du service informatique est intéressé par un logiciel qui met automatiquementau rebut les mails qu’il considère comme des SPAM. Ce logiciel est paramétrable : on peut régler de0 à 1 la probabilité p qu’un vrai SPAM soit mis au rebut mais la probabilité q qu’un mail soit mis aurebut alors que ce n’est pas un SPAM est liée à p par la relation q =

(p3

)2.

1. Exprimer en fonction de p la probabilité r(p) qu’un mail arrivant à la messagerie de cette entreprisesoit automatiquement mis au rebut.

2. La perte d’un mail qui n’est pas un SPAM coûte cinq fois plus cher à l’entreprise que la prise encompte et l’élimination d’un SPAM. Montrer que, si le paramétrage du logiciel anti-SPAM est réglépour minimiser le coût total, 49, 5% des mails seront automatiquement mis au rebut.

Solution

Notons S l’événement « le mail est un SPAM », et E l’événement « le mail est éliminé ».

1. Les données de l’énoncé se traduisent alors par

P(S) = P(SC) =1

2, P|S(E) = p , P|SC (E) = q .

Donc

r(p) = P(E) = P|S(E)P(S) + P|SC (E)P(SC) =1

2

[

p +(p

3

)2]

.

2. Notons c le coût d’un mail perdu, et b le bénéfice d’un SPAM éliminé. On a donc c = 5b.

Si N est le nombre de mails reçus, le coût total pour l’entreprise est donc

C(p) = cNP(E ∩ SC) − bNP(E ∩ S)

= Nb(5P(E ∩ SC) − NP(E ∩ S))

= Nb(5P|SC (E)P(SC) − P|S(E)P(S))

=Nb

2

[

5(p

3

)2− p

]

.

La fonction C est dérivable sur [ 0, 1 ] et son minimum est obtenu lorsque C ′(p) = 0. Or

C ′(p) =Nb

2

[10p

9− 1

]

.

Donc le minimum est obtenu pour

pm =9

10.

EI 143

Alors

r(pm) =1

2

[

9

10+

(3

10

)2]

=99

200=

49, 5

100.

Donc 49, 5% des mails seront mis au rebut.

120 Soit P une probabilité sur l’univers Ω = −2,−1, 0, 1, 2.

On définit sur Ω deux variables aléatoires réelles X et Y par :

X(k) =1

24(k2 − 2k)(−k2 + 8k + 17) + 1 et Y (k) = k2(5k2 − 17) .

X suit la loi binomiale B(2, 12 ) et Y suit la loi uniforme U(−12, 0, 12).

1. Pour tout k de Ω, on note pk = P(k).(a) Montrer que p−1 = 1

4 et p0 = 13 .

(b) En déduire les valeurs de p1 et p2.

2. Déterminer la moyenne m de la loi de probabilité P.

Solution

1. Calculons les valeurs de X et de Y sur Ω. On obtient le tableau suivant

k −2 −1 0 1 2

X(k) 0 2 1 0 1

Y (k) 12 −12 0 −12 12

Puisque X suit une loi B(2, 12), on a

P(X = 0) = P(X = 2) =1

4et P(X = 1) =

1

2,

et puisque Y suit une loi U(−12, 0, 12),

P(Y = −12) = P(Y = 0) = P(Y = 12) =1

3.

(a) On a donc en particulier,

X = 2 = −1 et Y = 0 = 0 ,

donc,

P(X = 2) = p−1 =1

4,

et

P(Y = 0) = p0 =1

3.

EI 144

(b) On a également

X = 0 = −2, 1 , X = 1 = 0, 2 , Y = −12 = −1, 1 , Y = 12 = −2, 2 .

ce qui donne tout d’abord

p0 + p2 =1

2et p1 + p−1 =

1

3,

d’où l’on tire immédiatement

p1 =1

12et p2 =

1

6.

Il reste les équations

p1 + p−2 =1

4et p2 + p−2 =

1

3,

qui donnent toutes les deux

p−2 =1

6.

On constate alors quep−2 + p−1 + p0 + p1 + p2 = 1 ,

ce qui montre que l’on a bien défini une probabilité sur Ω.

2. On en déduit la moyenne

m = −2p−2 − p−1 + p1 + 2p2 = −1

3− 1

4+

1

12+

1

3= −1

6.

121 La loi de probabilité d’une variable aléatoire réelle X est donnée par

P(X = k) =

(n−1k−1

)ak−1(1 − a)n−k si k ∈ [[ 1, n ]]

0 sinon

où n ∈ N∗ et a ∈ ] 0, 1 [ .

1. Calculer

n∑

k=1

(k − 1)P(X = k).

2. En déduire l’espérance mathématique de X.

Solution

1. Première méthode

Notons Sn la somme

n∑

k=1

(k − 1)P(X = k). Comme le premier terme est nul, on a, si n ≥ 2,

Sn =n∑

k=2

(k − 1)

(n − 1

k − 1

)

ak−1(1 − a)n−k .

EI 145

Mais

(k − 1)

(n − 1

k − 1

)

= (k − 1)(n − 1)!

(k − 1)!(n − k)!= (n − 1)

(n − 2)!

(k − 2)!(n − k)!= (n − 1)

(n − 2

k − 2

)

,

et donc

Sn =n∑

k=2

(n − 1)

(n − 2

k − 2

)

ak−1(1 − a)n−k .

Effectuons le changement d’indice de sommation j = k − 2. On obtient

Sn = (n − 1)

n−2∑

j=0

(n − 2

j

)

aj+1(1 − a)n−2−j ,

ou encore, en mettant a en facteur,

Sn = (n − 1)an−2∑

j=0

(n − 2

j

)

aj(1 − a)n−2−j .

Mais la somme est le développement de (a + (1 − a))n−2 et vaut 1 ; donc

Sn = (n − 1)a ,

ce qui reste vrai si n = 1 car la somme est nulle dans ce cas.

Deuxième méthode

Si n ≥ 2, soit f la fonction définie sur R par

f(x) = (x + 1 − a)n−1 .

On a, par la formule du binôme,

f(x) =n−1∑

j=0

(n − 1

j

)

xj(1 − a)n−1−j ,

et en dérivant

f ′(x) = (n − 1)(x + 1 − a)n−1 =

n−1∑

j=1

j

(n − 1

j

)

xj−1(1 − a)n−1−j .

Donc

f ′(a) = (n − 1) =

n−1∑

j=1

j

(n − 1

j

)

aj−1(1 − a)n−1−j .

Alors en multipliant par a,

a(n − 1) =

n−1∑

j=1

j

(n − 1

j

)

aj(1 − a)n−1−j .

EI 146

En faisant le changement d’indice de sommation k = j + 1, on obtient

a(n − 1) =

n∑

k=2

(k − 1)

(n − 1

k − 1

)

ak−1(1 − a)n−k = Sn .

2. On a

E(X) =

n∑

k=1

kP(X = k) =

n∑

k=1

(k − 1)P(X = k) +

n∑

k=1

P(X = k) = Sn + 1 .

DoncE(X) = (n − 1)a + 1 .

122 1. Montrer que, pour tout entier naturel non nul N ,

N∑

j=1

j =N(N + 1)

2et

N∑

j=1

j2 =N(N + 1)(2N + 1)

6.

2. Soit n un entier naturel non nul, et g la fonction réelle d’une variable réelle définie par

g(x) = n +1

2−∣∣∣∣x − n − 1

2

∣∣∣∣

.

On note X une variable aléatoire réelle telle que, pour tout k ∈ [[ 1, 2n ]] ,

P(X = k) =k

n(2n + 1).

et on définit une nouvelle variable aléatoire réelle par Y = g(X).

(a) i. Calculer E(X)ii. Calculer E(1/X).

(b) i. Déterminer la loi de probabilité de Y .ii. Calculer de deux manières l’espérance mathématique de Y .

Solution

1. Première méthode

On peut démontrer les deux formules par récurrence par exemple. Elles sont vraies pour N = 1.Supposons les deux formules vraies au rang N . Alors

N+1∑

j=1

j =

N∑

j=1

j + N + 1

=N(N + 1)

2+ N + 1

=N(N + 1) + 2(N + 1)

2

=(N + 1)(N + 2)

2.

EI 147

et de même

N+1∑

j=1

j2 =

N∑

j=1

j2 + (N + 1)2

=N(N + 1)(2N + 1)

6+ (N + 1)2

=N(N + 1)(2N + 1) + 6(N + 1)2

6

=(N + 1)(2N2 + N + 6N + 6)

6

=(N + 1)(2N2 + 7N + 6)

6

=(N + 1)(N + 2)(2N + 3)

6.

On obtient donc les formules au rang N +1. Il en résulte que les formules sont vraies pour tout N ≥ 1.

Deuxième méthode

On peut calculer les sommes directement. Notons S1(N) et S2(N) les deux sommes. Pour la première,on a

2S1(N) =

N∑

j=1

j +

N∑

j=1

(N + 1 − j) =

N∑

j=1

(N + 1) = N(N + 1) ,

ce qui redonne la valeur de S1(N).

On peut aussi écrire

(j + 1)2 − j2 = 2j + 1 ,

et en sommantN∑

j=0

((j + 1)2 − j2) = 2

N∑

j=0

j +

N∑

j=0

1 ,

ce qui donne

(N + 1)2 = 2S1(N) + N + 1 ,

d’où l’on tire

S1(N) =1

2((N + 1)2 − (N + 1)) =

N(N + 1)

2.

Pour la deuxième somme,

(j + 1)3 − j3 = 3j2 + 3j + 1 ,

doncN∑

j=0

((j + 1)3 − j3) = 3

N∑

j=0

j2 + 3

N∑

j=0

j +

N∑

j=0

1 ,

EI 148

ce qui donne

(N + 1)3 = 3S2(N) + 3S1(N) + N + 1 ,

et donc

S2(N) =(N + 1)3 − (N + 1)

3− N(N + 1)

2

=(N + 1)N(N + 2)

3− N(N + 1)

2

=N(N + 1)

6(2N + 4 − 3)

=N(N + 1)(2N + 1)

6.

2. On constate tout d’abord que l’on a

2n∑

k=1

k

n(2n + 1)=

1

n(2n + 1)

2n∑

k=1

k =1

n(2n + 1)

2n(2n + 1)

2= 1 ,

donc on peut effectivement avoir une variable aléatoire X telle que pour tout k ∈ [[ 1, 2n ]] ,

P(X = k) =k

n(2n + 1).

(a) i. On a alors

E(X) =

2n∑

k=1

k P(X = k) =1

n(2n + 1)

2n∑

k=1

k2 =1

n(2n + 1)

2n(2n + 1)(4n + 1)

6=

4n + 1

3.

ii. On a aussi

E(1/X) =2n∑

k=1

P(X = k)

k=

1

n(2n + 1)

2n∑

k=1

1 =2

2n + 1.

(b) i. Soit k dans [[ 1, n ]] . Alors k − n appartient à [[ 1 − n, 0 ]] et k − n − 1/2 est négatif. Donc

g(k) = n +1

2+

(

k − n − 1

2

)

= k ,

et g(k) se trouve dans [[ 1, n ]] .

Si k se trouve dans [[n + 1, 2n ]] , alors k − n appartient à [[ 1, n ]] et k − n − 1/2 est positif. Donc

g(k) = n +1

2−(

k − n − 1

2

)

= 2n − k + 1 ,

EI 149

et g(k) se trouve aussi dans [[ 1, n ]] .

Alors, si k appartient à [[ 1, n ]] , le nombre 2n + 1 − k appartient à [[ n + 1, 2n ]] , et

g(2n + 1 − k) = 2n − (2n + 1 − k) + 1 = k = g(k) .

Donc Y (Ω) = [[ 1, n ]] , et

Y = k = X = k ⊎ X = 2n + 1 − k .

On en déduit alors

P(Y = k) = P(X = k) + P(X = 2n + 1 − k) =k

n(2n + 1)+

2n + 1 − k

n(2n + 1)=

1

n.

La variable Y suit une loi uniforme U( [[ 1, n ]] ).

ii. On a tout d’abord

E(Y ) =

n∑

j=1

jP(Y = j) =

n∑

j=1

j

n=

n + 1

2.

Mais on a également

E(Y ) =

2n∑

k=1

g(k)P(X = k)

=

n∑

k=1

kg(k)

n(2n + 1)+

2n∑

k=n+1

kg(k)

n(2n + 1)

=

n∑

k=1

k2

n(2n + 1)+

2n∑

k=n+1

k(2n + 1 − k)

n(2n + 1).

Dans la deuxième somme, on fait le changement d’indice de sommation k′ = 2n + 1− k, et on obtient

E(Y ) =n∑

k=1

k2

n(2n + 1)+

n∑

k′=1

k′(2n + 1 − k′)

n(2n + 1)

=

n∑

k=1

k2 + k(2n + 1 − k)

n(2n + 1)

=

n∑

k=1

k(2n + 1)

n(2n + 1)=

n∑

k=1

k

n=

n + 1

2.

123 A toute variable aléatoire Z définie sur l’espace probabilisé (Ω, P) et à valeurs dans une partiefinie de N, on associe la fonction HZ définie sur R par

HZ(t) = P(Z ≥ t) .

EI 150

1. Montrer que(∀k ∈ N

∗) P(Z = k − 1) = HZ(k − 1) − HZ(k) .

2. Représenter HZ dans le cas particulier où la loi de Z est donnée par

k 0 1 3

P(Z = k) 1/6 1/2 1/3

3. Soit q la plus grande des valeurs que peut prendre Z.Montrer par récurrence (décroissante) sur n que

(∀n ∈ [[ 0, q ]] )

q∑

k=n

HZ(k) =

q∑

j=n

jP(Z = j) − (n − 1)HZ(n) .

4. Soit X et Y deux variables aléatoires définies sur (Ω, P) et à valeurs dans une partie finie de N.Démontrer l’implication

[ (∀t ∈ R) HX(t) ≥ HY (t) ] ⇒ (E(X) ≥ E(Y )) .

Solution

1. On aHZ(k − 1) − HZ(k) = P(Z ≥ k − 1) − P(Z ≥ k) .

Mais comme l’événement Z ≥ k est inclus dans Z ≥ k − 1, on en déduit

HZ(k − 1) − HZ(k) = P(Z ≥ k − 1 \ Z ≥ k)= P(Z ≥ k − 1 ∩ Z ≥ kC)

= P(Z ≥ k − 1 ∩ Z < k)= P(k − 1 ≤ Z < k) .

Mais puisque X est à valeurs entières, on a

k − 1 ≤ Z < k = Z = k − 1 ,

doncHZ(k − 1) − HZ(k) = P(Z = k − 1) .

2. On calcule les valeurs de HZ(t) suivant la position de t dans R.

Si t ∈ ] 3, ∞ [ , Z ≥ t = ∅ et HZ(t) = 0

Si t ∈ ] 1, 3 ] , Z ≥ t = 3 et HZ(t) =1

3

Si t ∈ ] 0, 1 ] , Z ≥ t = 1, 3 et HZ(t) =1

3+

1

2=

5

6

Si t ∈ ]−∞, 0 ] , Z ≥ t = Ω et HZ(t) = 1

EI 151

La fonction HZ est une fonction en escalier, dont le graphe est le suivant :

-

6

1

5/6

1/3

1 3 t

3. Puisque q est la plus grande valeur prise par Z, on a

Z ≥ q = Z = q ,

et doncHZ(q) = P(Z = q) ,

alorsqP(Z = q) − (q − 1)HZ(q) = qHZ(q) − (q − 1)HZ(q) = HZ(q) ,

et ceci donne la formule au rang q.

Supposons la formule vraie au rang n, où n est un entier compris entre 1 et q, et montrons qu’elle estvraie au rang n − 1. On a donc

q∑

k=n−1

HZ(k) =

q∑

k=n

HZ(k) + HZ(n − 1) ,

et, en utilisant l’hypothèse de récurrence,

q∑

k=n−1

HZ(k) =

q∑

j=n

jP(Z = j) − (n − 1)HZ(n) + HZ(n − 1) .

Alors d’après la question 1

q∑

k=n−1

HZ(k) =

q∑

j=n

jP(Z = j) − (n − 1)(HZ(n − 1) − P(Z = n − 1)) + HZ(n − 1) ,

EI 152

ce qui donne

q∑

k=n−1

HZ(k) =

q∑

j=n

jP(Z = j) + (n − 1)P(Z = n − 1) − (n − 2)HZ(n − 1) ,

et finalementq∑

k=n−1

HZ(k) =

q∑

j=n−1

jP(Z = j) − (n − 2)HZ(n − 1) ,

ce qui est la formule au rang n− 1. La formule est donc vraie pour tout entier n compris entre 0 et q.

4. En particulier, si n = 0,q∑

k=0

HZ(k) =

q∑

j=0

jP(Z = j) + HZ(0) ,

ou encore en soustrayant HZ(0) aux deux membres

q∑

k=1

HZ(k) =

q∑

j=0

jP(Z = j) .

On en déduit que

E(Z) =

q∑

k=1

HZ(k) .

Remarquons aussi que si k > q, on a HZ(k) = 0, donc, si N ≥ q

E(Z) =N∑

k=1

HZ(k) .

Soit N un nombre supérieur aux plus grandes valeurs prises par X et Y . Alors

E(X) − E(Y ) =N∑

k=1

HX(k) −N∑

k=1

HY (k) =N∑

k=1

(HX(k) − HY (k)) ,

mais comme on a, pour tout entier k,HX(k) ≥ HY (k) ,

tous les nombres HX(k) − HY (k) sont positifs, et leur somme E(X) − E(Y ) également. On en déduitque

E(X) ≥ E(Y ) .

EI 153

Partiel 2

124 Soit X et Y deux variables aléatoires définies sur le même espace probabilisé fini (Ω, P). Montrerl’équivalence des deux propriétés Q et R définies par

Q(X,Y ) : (∀x ∈ X(Ω)) (∀y ∈ Y (Ω)) X = x ⊥⊥ Y = y

R(X,Y ) : (∀A ⊂ X(Ω)) (∀B ⊂ Y (Ω)) X ∈ A ⊥⊥ Y ∈ BSolution

Si R(X,Y ) est vérifiée, et si x et y sont dans X(Ω), on peut prendre A = x et B = y, alorsX ∈ A ⊥⊥ Y ∈ B n’est autre que X = x ⊥⊥ Y = y, et Q(X,Y ) est vérifiée.

Si Q(X,Y ) est vérifiée, soit A et B deux sous-ensembles de Ω.

Si A ou B est vide on a

P(X ∈ A,Y ∈ B) = 0 = P(X ∈ A)P(X ∈ B) .

Si A et B ne sont pas vides, posons

A = a1, . . . , an et B = b1, . . . , bp .

Alors

P(X ∈ A,Y ∈ B) = P

n⊎

i=1

p⊎

j=1

X = ai, Y = bj

=

n∑

i=1

p∑

j=1

P(X = ai, Y = bj) ,

mais, puisque Q(X,Y ) est vérifiée, on a, quels que soient i et j,

P(X = ai, Y = bj) = P(X = ai)P(Y = bj) ,

donc

P(X ∈ A,Y ∈ B) =n∑

i=1

p∑

j=1

P(X = ai)(Y = bj) .

Cette somme n’est autre que le produit

(n∑

i=1

P(X = ai)

)

p∑

j=1

P(Y = bj)

= P(X ∈ A)P(X ∈ B) ,

et finalementP(X ∈ A,Y ∈ B) = P(X ∈ A)P(X ∈ B) .

On a donc bien montré que R(X,Y ) est vérifiée.

EI 154

125 Soit M1, . . . ,Mn des événements aléatoires deux à deux incompatibles ayant tous la mêmeprobabilité a non nulle et L un autre événement aléatoire associé au même espace probabilisé (Ω, P).

Exprimer la probabilité conditionnelle P

L |n⊎

j=1

Mj

en fonction des nombres pk = P(L | Mk).

Solution

On a, par définition d’une probabilité conditionnelle

P

(

L |n⊎

k=1

Mk

)

=

P

(

L ∩n⊎

k=1

Mk

)

P

(n⊎

k=1

Mk

) .

Mais, d’après la distributivité de l’intersection sur la réunion

L ∩n⋃

k=1

Mk =

n⋃

k=1

(L ∩ Mk) ,

et par ailleurs, si k 6= i,

(L ∩ Mk) ∩ (L ∩ Mi) = L ∩ (Mk ∩ Mi) = L ∩ ∅ = ∅ ,

donc

L ∩n⊎

k=1

Mk =

n⊎

k=1

(L ∩ Mk) ,

et

P

(

L ∩n⊎

k=1

Mk

)

= P

(n⊎

k=1

(L ∩ Mk)

)

=

n∑

k=1

P(L ∩ Mk) .

MaisP(L ∩ Mk) = P(Mk)P(L | Mk) = a pk ,

donc

P

(

L ∩n⊎

k=1

Mk

)

= an∑

k=1

pk .

Par ailleurs

P

(n⊎

k=1

Mk

)

=

n∑

k=1

P(Mk) =

n∑

k=1

a = na ,

d’où finalement

P

(

L |n⊎

k=1

Mk

)

=1

n

n∑

k=1

pk .

EI 155

126 Un dé équilibré porte un point sur une de ses six faces, deux points sur deux autres faces ettrois points sur chacune des trois dernières faces. On note X la variable aléatoire « nombre de pointsobtenus » lors d’un lancer de ce dé.

1. Calculer la variance de X.2. Après avoir lancé le dé, on lance une pièce de monnaie (équilibrée) autant de fois que le dé a

montré de points et on note Y la variable aléatoire « nombre de piles obtenus en tout ». Vérifier queP(Y = 2 | X = 3) = 3

8 .3. Déterminer la loi de probabilité du couple (X,Y ).4. En déduire la loi de probabilité de Y et vérifier que l’événement « Y est pair » et l’événement « Y

est impair » ont la même probabilité.

Solution

1. Soit i ∈ 1, 2, 3. Comme il y a i faces sur 6 de numéro i, la probabilité de sortie du numéro i estproportionnelle à i et vaut

P(X = i) =i

6.

Alors

E(X) =3∑

i=1

i2

6=

7

3et E(X2) =

3∑

i=1

i3

6= 6 ,

d’où

V(X) = E(X2) − E(X)2 = 6 − 49

9=

5

9.

2. Cherchons la probabilité P(Y = j | X = i), lorsque 0 ≤ j ≤ i et 1 ≤ i ≤ 3.

Sachant que X = i, on compte le nombre de piles parmi i lancers, où pile a une probabilité 1/2 desortir. On a donc une loi binomiale B(i, 1/2) et

P(Y = j | X = i) =

(i

j

)1

2i.

En particulier

P(Y = 2 | X = 3) =

(3

2

)1

23=

3

8.

3. On a, lorsque 0 ≤ j ≤ i et 1 ≤ i ≤ 3,

pij = P(X = i, Y = j) = P(Y = j | X = i)P(X = i) =

(i

j

)1

2i

i

6,

ce qui donne le tableau des pij :

j = 0 j = 1 j = 2 j = 3

i = 1 1/12 1/12 0 0i = 2 1/12 1/6 1/12 0i = 3 1/16 3/16 3/16 1/16

EI 156

4. En additionnant les lignes de ce tableau, on obtient la loi de Y :

P(Y = 0) =1

12+

1

12+

1

16=

11

48

P(Y = 1) =1

12+

1

6+

3

16=

7

16

P(Y = 2) =1

12+

3

16=

13

48

P(Y = 3) =1

16.

On constate que

P(Y pair) = P(Y = 0) + P(Y = 2) =11

48+

13

48=

1

2,

et

P(Y impair) = P(Y = 1) + P(Y = 3) =7

16+

1

16=

1

2,

donc ces deux probabilités sont bien égales.

127 1. Montrer que, si deux variables aléatoires ne sont pas indépendantes, aucune des deux n’estdégénérée.

2. Soit U , V , W des variables aléatoires réelles indépendantes qui ne peuvent prendre chacune qu’unnombre fini de valeurs distinctes.Calculer Cov(U + V, V + W ) et en déduire une condition nécessaire et suffisante d’indépendance desvariables aléatoires U + V et V + W .

Solution

1. Si A est un événement de probabilité 0 ou 1, il est indépendant de tout autre événement. En effet,si P(A) = 0, on a, quel que soit l’événement B,

A ∩ B ⊂ A ,

donc0 ≤ P(A ∩ B) ≤ P(A) = 0 ,

ce qui montre queP(A ∩ B) = 0 = P(A)P(B) .

Si P(A) = 1, alors P(AC) = 0, donc, B et AC sont indépendants, et

P(A ∩ B) = P(B) − P(AC ∩ B) = P(B) = P(A)P(B) .

On en déduit alors que toute variable aléatoire X dégénérée est indépendante d’une variable Y quel-conque définie sur le même espace (Ω, P). En effet, si a appartient à X(Ω), on a soit P(X = a) = 1,

EI 157

soit P(X = a) = 0, et donc, P(X = a) est indépendant de tout événement P(Y = b), ce qui montreque X et Y sont indépendantes.

Une variable dégénérée est donc indépendante de toute autre variable, et par suite si deux variablessont dépendantes, elles ne peuvent être dégénérées.

2. En utilisant la bilinéarité de la covariance, on a

Cov(U + V, V + W ) = Cov(U, V ) + Cov(U,W ) + Cov(V, V ) + Cov(V,W ) .

comme les variables sont indépendantes, donc indépendantes deux à deux, on a

Cov(U, V ) = Cov(U,W ) = Cov(V,W ) = 0 ,

doncCov(U + V, V + W ) = Cov(V, V ) = V(V ) .

Si les variables U + V et V + W sont indépendantes on a Cov(U + V, V + W ) = 0 donc V(V ) = 0 cequi implique que V est dégénérée.

Réciproquement, supposons que V est dégénérée, et soit v dans V (Ω) tel que P(V = v) = 1. L’événe-ment V = v est indépendant de tout autre événement. Alors, si u appartient à U(Ω) et w à W (Ω),on a

P(U + V = u, V + W = w) = P(U + V = u, V + W = w, V = v)

= P(U = u − v,W = w − v, V = v)

= P(U = u − v,W = w − v) ,

et de même

P(U + V = u) = P(U + V = u, V = v) = P(U = u − v, V = v) = P(U = u − v) ,

ainsi que

P(V + W = w) = P(V + W = w, V = v) = P(W = w − v, V = v) = P(W = w − v) .

Mais puisque U et W sont indépendantes

P(U = u − v,W = w − v) = P(U = u − v)P(W = w − v) ,

alors, il résulte de tout ce qui précède que

P(U + V = u,U + W = w) = P(U + V = u)P(V + W = w) ,

et donc U + V et V + W sont indépendantes.

128 Soit (Z,Z ′) un couple de variables aléatoires réelles non dégénérées qui ne peuvent prendrechacune qu’un nombre fini de valeurs distinctes, d’écart-type respectifs σ et σ′ = 2σ. Déterminer le

EI 158

coefficient de corrélation linéaire ρ du couple (Z + Z ′, Z − Z ′) si celui du couple (Z,Z ′) est r = −34 .

Solution

En utilisant la bilinéarité de la covariance, on a

Cov(Z + Z ′, Z − Z ′) = V(Z) − V(Z ′) = σ2 − σ′2 = −3σ2 .

On a également

σ2Z+Z′ = V(Z + Z ′) = V(Z) + 2 Cov(Z,Z ′) + V(Z ′) = σ2 + 2σσ′rZ,Z′ + σ′2 = σ2(1 + 4r + 4) = 2σ2 ,

et

σ2Z−Z′ = V(Z − Z ′) = V(Z) − 2 Cov(Z,Z ′) + V(Z ′) = σ2 − 2σσ′rZ,Z′ + σ′2 = σ2(1 − 4r + 4) = 8σ2 .

Alors

ρ =Cov(Z + Z ′, Z − Z ′)

σZ+Z′σZ−Z′

=−3σ2

4σ2= −3

4.

EI 159

Partiel 3

129 Soit (U, V ) un couple de variables aléatoires réelles indépendantes qui suivent la même loibinomiale B(2, 1/2).

1. On note S = (U − 1)2 + (V − 1)2.(a) Vérifier que S suit la même loi que U .(b) Calculer l’écart-type de S2.

2. On note T = (U − 1)(V − 1) + 1.(a) Calculer E(S(T − 1)).(b) Déterminer la loi de T .(c) Calculer le coefficient de corrélation linéaire du couple (S, T ).(d) Les variables aléatoires S et T sont-elles indépendantes ?

Solution

1. (a) Une variable X suivant une loi B(n, p) est telle que X(Ω) = [[ 0, 1 ]] et si k appartient à X(Ω),

P(X = k) =

(n

k

)

pk(1 − p)n−k .

Pour les variables U et V , on a donc

U(Ω) = V (Ω) = 0, 1, 2 ,

et

P(U = 0) = P(V = 0) = P(U = 2) = P(V = 2) =1

4et P(U = 1) = P(V = 1) =

1

2.

On remarque que (U − 1)(Ω) = −1, 0, 1 et donc (U − 1)2(Ω) = 0, 1. De plus

P((U − 1)2 = 0) = P(U = 1) =1

2,

donc

P((U − 1)2 = 1) = 1 − P((U − 1)2 = 0) =1

2,

et (U − 1)2 suit une loi de Bernoulli B(1/2). Il en est de même de (V − 1)2. Comme les variables Uet V sont indépendantes, les variables (U − 1)2 et (V − 1)2 le sont aussi, et S est la somme de deuxvariables de Bernoulli indépendantes de paramètre 1/2, elle suit alors une loi binomiale B(2, 1/2).

(b) Une variable suivant une loi binomiale B(n, p) a pour espérance np et pour variance npq, donc Uet V ont pour espérance 1, et pour variance 1/2.

Pour tout entier n on a (U − 1)2n = (U − 1)2, puisque (U − 1)2 ne prend que les valeurs 0 et 1, et donc

E((U − 1)2n) = E((V − 1)2n) = E((U − 1)2) = V(U) =1

2.

EI 160

AlorsE(S2) = E((U − 1)4 + 2(U − 1)2(V − 1)2 + (V − 1)4) ,

et en utilisant la linéarité de l’espérance

E(S2) = E((U − 1)4) + 2E((U − 1)2(V − 1)2) + E((V − 1)4) ,

et puisque (U − 1)2 et (V − 1)2 sont indépendantes

E((U − 1)2(V − 1)2) = E((U − 1)2)E((V − 1)2) .

Finalement

E(S2) = E((U − 1)4) + 2E((U − 1)2)E((V − 1)2) + E((V − 1)4) =3

2.

De même, par la formule du binôme de Newton,

E(S4) = E((U − 1)8 + 4(U − 1)6(V − 1)2 + 6(U − 1)4(V − 1)4 + 4(U − 1)2(V − 1)6 + (V − 1)8) ,

et en utilisant encore la linéarité de E et l’indépendance des variables U et V ,

E(S4) = E((U − 1)8) + 4E((U − 1)6)E((V − 1)2) + 6E((U − 1)4)E((V − 1)4)

+4E((U − 1)2)E((V − 1)6) + E((V − 1)8)

=1

2+ 4

1

2

1

2+ 6

1

2

1

2+ 4

1

2

1

2+

1

2

=9

2.

Alors

V(S2) = E(S4) − E(S2)2 =9

2− 9

4=

9

4,

et l’écart-type de S2 vaut 3/2.

2. (a) On aE(S(T − 1)) = E((U − 1)3(V − 1) + (U − 1)(V − 1)3) ,

et en utilisant toujours la linéarité de E et l’indépendance des variables U et V ,

E(S(T − 1)) = E((U − 1)3)E(V − 1) + E(U − 1)E((V − 1)3) .

MaisE(U − 1) = E(U) − 1 = 0 et E(V − 1) = E(V ) − 1 = 0 ,

doncE(S(T − 1)) = 0 .

(b) Faisons le tableau des valeurs de T .

U = 0 U = 1 U = 2

V = 0 T = 2 T = 1 T = 0

V = 1 T = 1 T = 1 T = 1

V = 2 T = 0 T = 1 T = 2

EI 161

Donc T (Ω) = 0, 1, 2, et

P(T = 0) = P(U = 0, V = 2) + P(U = 2, V = 0) ,

et puisque les variables U et V sont indépendantes

P(T = 0) = P(U = 0)P(V = 2) + P(U = 2)P(V = 0) =1

8.

De même

P(T = 2) = P(U = 0)P(V = 0) + P(U = 2)P(V = 2) =1

8.

Alors

P(T = 1) = 1 − P(T = 0) − P(T = 2) =3

4.

(c) On a déjàE(S) = E((U − 1)2) + E((V − 1)2) = 2V(U) = 1 .

On a aussi

E(T ) = P(T = 1) + 2P(T = 2) =3

4+

1

4= 1 .

Par ailleursE(S(T − 1)) = E(ST ) − E(S) = 0 ,

doncE(ST ) = E(S) = 1 .

Il en résulte queCov(S, T ) = E(ST ) − E(S)E(T ) = 0 .

(d) Regardons par exemple P(S = 0, T = 0). On a

S = 0 = U = 1, V = 1 .

Mais si U(ω) = V (ω) = 1, alors T (ω) = 1. Donc l’événement S = 0, T = 0 est impossible et saprobabilité est nulle, alors que P(S = 0)P(T = 0) n’est pas nul. Les variables aléatoires S et T ne sontpas indépendantes.

130 1. (a) Soit Z une variable aléatoire réelle qui possède une espérance mathématique m et unécart-type σ. Montrer que, si ε > σ, on peut majorer P(|Z − m| ≥ ε) par un nombre strictementinférieur à 1 qui ne dépend que de σ et ε.(b) Que peut-on en déduire pour P(−2 < Z < 4) si m = 1 et σ =

√2 ?

2. (a) On répète des lancers d’une pièce de monnaie (équilibrée) tant qu’elle montre « pile » et on noteN la variable aléatoire nombre de « piles » observés (N prend la valeur 0 si la pièce montre « face »au premier lancer).i. Calculer P(N = n) si n est un entier naturel.ii. En déduire la probabilité d’obtenir une infinité de « piles ».

EI 162

(b)

i. Déterminer la fonction génératrice gN .ii. En déduire que N possède des moments d’ordres 1, 2, 3 et calculer son espérance mathématique et

son écart-type.iii. Que vaut P(−2 < N < 4) ?iv. Calculer le moment centré d’ordre 3 de N .

Solution

1. (a) Si l’on applique l’inégalité de Tchebychev, on obtient

P(|Z − m| ≥ ε) ≤ σ2

ε2,

ce qui donne bien une majoration par un nombre strictement inférieur à 1.

(b) En particulier, si l’on prend ε = 3, m = 1 et σ =√

2, l’inégalité précédente donne

P(|Z − 1| ≥ 3) ≤ 2

9,

ou encore

1 − P(|Z − 1| < 3) ≤ 2

9,

Mais

|Z − 1| < 3 = −2 < Z < 4 ,

d’où

P(−2 < Z < 4) ≥ 1 − 2

9=

7

9.

2. (a) i. L’événement N = n est réalisé lorsque n piles puis un face ont été obtenus. Il y a donc N +1lancers de la pièce et chaque lancer a une probabilité 1/2 d’obtenir pile ou face. Donc

P(N = n) =1

2n+1.

ii. Soit Un l’événement « sur les n premiers lancers, on a obtenu n fois piles ». La probabilité de Un

vaut 2−n. L’intersection des événements Un est l’événement U « on observe une infinité de piles », etcomme la suite Un est décroissante (Un+1 ⊂ Un), on a

P(U) = limn→+∞

P(Un) = limn→+∞

2−n = 0 .

(b) i. On a

gN (s) = E(sN ) =∞∑

n=0

sn

2n+1=

1

2

∞∑

n=0

(s

2

)n,

donc

gN (s) =1

2

1

1 − s2

=1

2 − s= (2 − s)−1 .

EI 163

ii. La fonction obtenue est une série entière de rayon 2, car la série géométrique de raison s/2 convergesi et seulement si |s/2| < 1, soit |s| < 2. Comme le rayon de convergence est strictement plus grandque 1, la variable N possède donc des moments de tous ordres. On obtient en dérivant les momentsfactoriels. On a successivement

g′N (s) = E(NsN−1) = (2 − s)−2 , g′′N (s) = E(N(N − 1)sN−2) = 2 (2 − s)−3 ,

donc

E(N) = g′N (1) = 1 , V(N) = g′′N (1) + g′N (1) − g′N (1)2 = 2 + 1 − 1 = 2 ,

et l’écart-type de N vaut√

2.

iii. Comme N ne prend que des valeurs entières positives, on a

P(−2 < N < 4) = P(0 ≤ N ≤ 3) =3∑

n=0

1

2n+1=

1

2

3∑

n=0

1

2n=

1

2

1 − 124

1 − 12

= 1 − 1

24=

15

16.

Remarque : on vérifie bien que 15/16 ≥ 7/9.

iv. On a

g′′′N (s) = E(N(N − 1)(N − 2)sN−3) = 6 (2 − s)−4 ,

donc

E(N(N − 1)(N − 2)) = 6 .

Alors, en écrivant

N(N − 1)(N − 2) = (N − 1 + 1)(N − 1)(N − 1 − 1) = ((N − 1)2 − 1)(N − 1) = (N − 1)3 − (N − 1) ,

on obtient

E((N − 1)3) = E(N(N − 1)(N − 2)) + E(N − 1) = E(N(N − 1)(N − 2)) = 6 .

131 Soit (Ω, P) un espace probabilisé infini dénombrable. On note X une variable aléatoire nondégénérée définie sur Ω et à valeurs dans [ 0, 1 ] .

1. (a) Montrer que X possède des moments de tous ordres entiers naturels. (On pourra noter X(Ω) =x0, x1, . . .).(b) Montrer que 0 < E(X)2 < E(X2) ≤ E(X) < 1.

2. On suppose de plus que (∀n ∈ N∗) P(X < 1

n) =n + 2

2n + 3.

(a) Que vaut P(X = 1) ?

(b) Pour tout entier naturel non nul n, on note Dn = X < 1/n.Quel est l’événement aléatoire

n∈N∗

Dn ?

EI 164

(c) Montrer que P(0 < X < 1) = 0, 1.

3. On suppose en outre que les seules valeurs que peut prendre X sont 0 et les inverses des entiersnaturels : 1, 1/2, 1/3, . . .(a) Soit n un entier supérieur ou égal à 2.i. Quel est l’événement aléatoire 1

n ≤ X < 1n−1 ?

ii. Que vaut P(X = 1/n) ?(b)

i. Calculer

(1

2n − 1− 1

2n + 1

)

−(

1

2n + 1− 1

2n + 3

)

si n ∈ N∗.

ii. En déduire une valeur approchée de E(X) (on pourra encadrer 1/n par deux fractions ayant desdénominateurs impairs bien choisis).

Solution

1. (a) La série de terme général P(X = xn) est une série positive convergente (de somme 1). Or, puisquexn appartient à [ 0, 1 ] , on a, si k est un entier positif,

|xknP(X = xn)| ≤ P(X = xn) ,

et il en résulte que la série de terme général xknP(X = xn) converge absolument. Donc la variable X

possède des moments de tous ordres.

Comme on aV(X) = E(X2) − E(X)2 = E((X − E(X))2) ≥ 0 ,

on en déduit queE(X)2 ≤ E(X2) .

Si l’on avait égalité, alors V(X) serait nul et X serait dégénérée, ce qui n’est pas le cas. Donc cetteinégalité est stricte.

Comme X ≤ 1, on en déduit X2 ≤ X et E(X2) ≤ E(X).

On en déduit aussi E(X) ≤ E(1) = 1.

On a également E(X)2 ≥ 0.

En particulier on a obtenu l’encadrement

E(X)2 < E(X2) ≤ E(X) ,

doncE(X)2 < E(X) ,

et il est impossible d’avoir E(X) = 0 ou E(X) = 1. Donc

E(X) < 1 et 0 < E(X)2 .

EI 165

2. (a) On a P(X < 1) = 3/5. Alors, puisque X(Ω) est inclus dans [ 0, 1 ]

P(X = 1) = 1 − P(X < 1) =2

5.

(b) Si ω appartient à⋂

n∈N∗

Dn, on a pour tout n ≥ 0

0 ≤ X(ω) <1

n,

donc X(ω) = 0. On en déduit que⋂

n∈N∗

Dn ⊂ X = 0 .

Comme par ailleurs X = 0 est inclus dans tous les Dn on a l’inclusion inverse, d’où l’égalité

n∈N∗

Dn = X = 0 .

(c) Comme la suite Dn est décroissante, on obtient donc

P(X = 0) = limn→+∞

P(Dn) = limn→+∞

n + 1

2n + 3=

1

2.

Alors

P(0 < X < 1) = 1 − P(X = 0) − P(X = 1) = 1 − 1

2− 2

5=

1

10.

3. (a) i. Comme X ne prend comme valeurs non nulles que des inverses de nombre entier, on a

1

n≤ X <

1

n − 1

=

X =1

n

,

ce que l’on peut encore écrire

X <1

n − 1

\

X <1

n

=

X =1

n

.

ii. On a alors

P

(

X =1

n

)

= P

(1

n≤ X <

1

n − 1

)

= P

(

X <1

n − 1

)

− P

(

X <1

n

)

.

Donc

P

(

X =1

n

)

=n + 1

2n + 1− n + 2

2n + 3=

1

(2n + 1)(2n + 3).

(b) i. Posons

un =1

2n − 1− 1

2n + 1.

On obtient après calcul,

un − un+1 =

(1

2n − 1− 1

2n + 1

)

−(

1

2n + 1− 1

2n + 3

)

=8

(2n − 1)(2n + 1)(2n + 3).

EI 166

ii. On a

E(X) =

∞∑

n=1

1

nP

(

X =1

n

)

=

∞∑

n=1

1

n

1

(2n + 1)(2n + 3).

On peut alors écrire2

2n + 5≤ 1

n=

2

2n≤ 2

2n − 1,

et l’on en déduit

∞∑

n=1

2

(2n + 1)(2n + 3)(2n + 5)≤ E(X) ≤

∞∑

n=1

2

(2n − 1)(2n + 1)(2n + 3).

Mais d’après ce qui précède

2

(2n − 1)(2n + 1)(2n + 3)=

1

4(un − un+1) et

2

(2n + 1)(2n + 3)(2n + 5)=

1

4(un+1 − un+2) ,

donc1

4

∞∑

n=1

(un+1 − un+2) ≤ E(X) ≤ 1

4

∞∑

n=1

(un − un+1) .

Puisque l’on obtient des séries télescopiques dans les membres qui encadrent E(X) on peut effectuerla somme de ces séries, et puisque la suite (un) converge vers 0, on trouve finalement

1

4u2 ≤ E(X) ≤ 1

4u1 ,

soit1

30≤ E(X) ≤ 1

6.

On peut prendre comme valeur approchée de E(X) le milieu de l’intervalle [ 1/30, 1/6 ] , soit 1/10 = 0, 1avec une erreur inférieure à la moitié de la longueur de cet intervalle, soit

|E(X) − 0, 1| ≤ 1

15.